PSCAD-Resolving-Launching-Compiling-and-Running Issues
PSCAD-Resolving-Launching-Compiling-and-Running Issues
November 2, 2023
Revision 83
PSCAD - Resolving Launching, Compiling, and Running Issues
Contents
1. INTRODUCTION .............................................................................................................................. 7
1.1 OVERVIEW ..................................................................................................................................................................... 7
1.2 RELATED SUPPORT DOCUMENTS ........................................................................................................................................ 7
2. ISSUES WHEN LAUNCHING PSCAD .................................................................................................... 8
2.1 RECEIVING A “ROOT ELEMENT IS MISSING” ERROR MESSAGE ................................................................................................... 8
2.2 THE PSCAD LAUNCH IS PROLONGED ................................................................................................................................ 10
2.3 RECEIVING A “STOP RUNNING THIS SCRIPT?” ERROR MESSAGE .............................................................................................. 11
2.4 RECEIVING AN “UNABLE TO LOCATE THE MASTER LIBRARY FROM THE GIVEN FILE PATH ” ERROR MESSAGE ....................................... 12
2.5 RECEIVING A “SCRIPT ERROR” ERROR MESSAGE.................................................................................................................. 14
2.6 RECEIVING AN “MFC100.DLL IS MISSING” ERROR MESSAGE .................................................................................................. 16
2.7 RECEIVING A “THE PROCEDURE ENTRY POINT … DYNAMIC LINK LIBRARY SHELL32.DL” ERROR MESSAGE ....................................... 17
2.8 RECEIVING A “THE PROCEDURE ENTRY POINT …COULD NOT BE LOCATED IN THE DYNAMIC LINK LIBRARY ZSLIB2.DLL” ERROR MESSAGE 18
2.9 PSCAD CRASHES UPON STARTUP...................................................................................................................................... 19
2.10 THE PSCAD BETA EDITION CRASHES UPON STARTUP .......................................................................................................... 20
2.11 PSCAD V4.6.0 OR V4.6.1 64-BIT DOES NOT LAUNCH ......................................................................................................... 21
2.12 RECEIVING A “THE COMPUTER MUST BE TRUSTED FOR DELEGATION…” ERROR MESSAGE.............................................................. 22
2.13 HOW TO LAUNCH PSCAD WITHOUT WINDOWS ADMINISTRATOR PRIVILEGES ........................................................................... 22
2.14 RECEIVING A “THE APPLICATION WAS UNABLE TO START CORRECTLY (0XC00007B)” ERROR MESSAGE .......................................... 24
2.15 RECEIVING AN “ENTRY POINT NOT FOUND” ERROR MESSAGE ............................................................................................... 25
2.16 RECEIVING MISSING CRITICAL FILE MESSAGES WHEN TRYING TO LAUNCH PSCAD ..................................................................... 26
2.17 RECEIVING AN “UNABLE TO FIND OR CREATE THE REQUIRED VERSION REGISTRY KEYS ” ERROR MESSAGE ......................................... 28
2.18 RECEIVING AN “THE CODE EXECUTION CANNOT PROCEED BECAUSE MFC 140U.DLL WAS NOT FOUND” ERROR MESSAGE .................... 29
2.19 WHEN LAUNCHING PSCAD FROM A PSCAD PROJECT, CANNOT LAUNCH THE INTENDED VERSION............................................... 30
2.20 THE ERROR “THE END USER LICENSE AGREEMENT COULD NOT BE FOUND …” DISPLAYS UPON LAUNCHING ..................................... 31
3. ISSUES WHEN LICENSING PSCAD – LEGACY LOCK-BASED .................................................................... 32
4. ISSUES WHEN LICENSING PSCAD – CERTIFICATE-BASED...................................................................... 33
5. ISSUES WHEN LICENSING PSCAD – LEGACY LOCKLESS......................................................................... 34
6. ISSUES WHEN USING PSCAD............................................................................................................ 35
6.1 PSCAD CRASHES WHEN THE “ASSOCIATIONS” TAB IS SELECTED ............................................................................................. 35
6.2 THE COMPONENT WIZARD PANE IS NOT DISPLAYING CORRECTLY ........................................................................................... 36
6.3 RECEIVING A “THE PSCAD AUTOMATED EMAIL SYSTEM HAS FAILED TO SEND YOUR REQUEST ” MESSAGE ....................................... 38
6.4 RECEIVING AN “UNABLE TO DISPLAY START PAGE” ERROR MESSAGE........................................................................................ 40
6.5 COPYING A CONTROL WILL SWITCH ITS VALUE TO THE DEFAULT SETTING ................................................................................. 41
6.6 RECEIVING AN “UNABLE TO CONNECT TO MYCENTRE SERVER” ERROR MESSAGE ....................................................................... 42
6.7 RECEIVING A “REVOCATION INFORMATION FOR THE SECURITY CERTIFICATE FOR THIS SITE IS NOT AVAILABLE ” ERROR MESSAGE ........... 44
6.8 RECEIVING A “A PROGRAM IS TRYING TO SEND AN E-MAIL MESSAGE ON YOUR BEHALF” ERROR MESSAGE ....................................... 46
6.9 A PANE APPEARS TO BE MISSING ..................................................................................................................................... 47
6.10 GRAPHS DO NOT DISPLAY................................................................................................................................................ 49
6.11 ISSUES WITH DEBUGGING PSCAD .................................................................................................................................... 50
6.12 RECEIVING A “SERVER BUSY” ERROR MESSAGE................................................................................................................... 51
6.13 THE “OPEN EXAMPLES” MENU OPTION DOES NOT WORK ..................................................................................................... 52
6.14 CANNOT LOAD A PSCAD PROJECT.................................................................................................................................... 53
6.15 MATLAB IS NOT DETECTED BY PSCAD ............................................................................................................................... 54
6.16 RECEIVING A “SECURITY ALERT” MESSAGE ......................................................................................................................... 56
6.17 MENU TOOLS ARE NOT ACTIVE ........................................................................................................................................ 58
6.18 RECEIVING A “POWER SYSTEMS SIMULATOR HAS STOPPED WORKING” ERROR MESSAGE............................................................. 59
6.19 RECEIVING A “POWER SYSTEMS SIMULATOR HAS STOPPED WORKING” ERROR MESSAGE............................................................. 61
PSCAD - Resolving Launching, Compiling, and Running Issues
6.20 64-BIT EDITIONS OF INTEL FORTRAN ARE NOT DETECTED BY PSCAD ....................................................................................... 62
6.21 USER DOES NOT HAVE PERMISSIONS FOR LOCALAPPDATA FOLDER .......................................................................................... 63
6.22 INFORMATION IN THE PSCAD UPPER AND LOWER FRAMES IS NOT VISIBLE............................................................................... 64
6.23 SOME INFORMATION IN THE STATUS BAR IS NOT VISIBLE....................................................................................................... 65
6.24 RECEIVING A “THE SELECTED FOLDER ‘*’ DOES NOT EXIST…” ERROR MESSAGE .......................................................................... 66
6.25 RECEIVING ERROR “THIS FILE APPEARS TO BE FROM A LATER VERSION OF THE SOFTWARE…” WHEN ATTEMPTING TO OPEN A FILE IN
PSCAD X4 ............................................................................................................................................................................... 67
6.26 RECEIVING A “THE WORKSPACE ALREADY CONTAINS THE NAME SPACE …” ERROR MESSAGE WHEN TRYING TO CHANGE THE PROJECT
SETTINGS FOR A MULTIPLE SIMULATION SET.................................................................................................................................... 68
6.27 THE INSTALLED VERSION OF MATLAB IS NOT DETECTED IN THE PSCAD APPLICATION OPTIONS, AND THEREFORE CANNOT BE
SELECTED69
6.28 GRAPHS SHRINK AFTER REOPENING .................................................................................................................................. 70
6.29 SOME PARAMETER WINDOWS ARE BLANK ......................................................................................................................... 71
6.30 RECEIVING THE ERROR "SYSTEM.XML: MANAGED CODE EXCEPTION" WHEN TRYING TO LOAD A PSCAD PROJECT .......................... 72
6.31 “REQUEST SUPPORT” FEATURE – THE AUTO-ATTACH BUILD MESSAGES INCORRECTLY FORWARDS A TEXT FILE WITH STRINGS OF
NUMBERS ................................................................................................................................................................................. 73
7. ISSUES WHEN COMPILING CASES IN PSCAD ...................................................................................... 74
7.1 RECEIVING A “1 PROJECT(S) WAS UNSUCCESSFUL WHEN BUILDING” ERROR MESSAGE................................................................. 74
7.2 RECEIVING AN “UNRESOLVED EXTERNAL SYMBOL” ERROR MESSAGE........................................................................................ 76
7.3 RECEIVING A “FORTRAN COMPILER…IS NOT DETECTED” ERROR MESSAGE ................................................................................ 77
7.4 RECEIVING A " CANNOT EXECUTE COMMAND …ERROR 5" ERROR............................................................................................ 79
7.5 RECEIVING A "‘GFORTRAN.EXE: MAIN.F: INVALID ARGUMENT" ERROR..................................................................................... 81
7.6 RECEIVING AN “ERROR 1 – MULTIPLE DEFINITION OF ‘…’ - FIRST DEFINED HERE” ERROR ............................................................. 83
7.7 RECEIVING AN “ERROR 1 – SYNTAX ERROR IN ARGUMENT LIST AT (…)” ERROR .......................................................................... 84
7.8 RECEIVING A “MAKE FAILED TO GENERATE A SIMULATION EXECUTABLE FOR NAMESPACE ‘***’. BINARY FILE WAS NOT FOUND ” ERROR 85
7.9 RECEIVING A “MAKE: *** [SOMEFILE.EXE] ERROR 1” ERROR ................................................................................................ 89
7.10 RECEIVING AN "ERROR U1052 – FILE‘DATA.MAK’ NOT FOUND " ERROR .................................................................................. 90
7.11 RECEIVING A "MAKE FILE ERROR” ERROR .......................................................................................................................... 93
7.12 RECEIVING AN “UNABLE TO SOLVE LINE CONSTANTS” BUILD MESSAGE .................................................................................... 94
7.13 RECEIVING A “‘CL.EXE’ IS NOT RECOGNIZED AS AN INTERNAL OR EXTERNAL COMMAND ” BUILD MESSAGE......................................... 95
7.14 RECEIVING A “THE NUMBER OF PROJECTS INCLUDED IN THE SIMULATION LIST MUST NOT EXCEED %D” ERROR MESSAGE ................... 96
7.15 RECEIVING AN “UNABLE TO OPEN FILE…” ERROR MESSAGE................................................................................................... 97
7.16 RECEIVING A “COULD NOT READ SYMBOLS: MALFORMED ARCHIVE” ........................................................................................ 98
7.17 PSCAD STOPS UNEXPECTELY, WITH NO RUNTIME ERROR ..................................................................................................... 99
7.18 RECEIVING “ERROR CODE = 112” MESSAGE..................................................................................................................... 101
7.19 RECEIVING A “TRANSMITTER ‘*’ CANNOT BE ADDED” BUILD MESSAGE .................................................................................. 102
7.20 RECEIVING A “EMTDC: ERROR WHILE OPENING FILE FOR READ ” BUILD MESSAGE ................................................................... 103
7.21 RECEIVING A “MAKE: *** [STATION.O] ERROR 126” ERROR MESSAGE................................................................................. 104
7.22 RECEIVING A “WINSOCK ERROR #10048” ERROR MESSAGE ............................................................................................... 105
7.23 RECEIVING A “CONNECT FUNCTION FAILED WITH ERROR: 10061” ERROR MESSAGE ................................................................. 107
7.24 RECEIVING AN “UNABLE TO EXECUTE A SIMULATION EXECUTABLE FOR NAMESPACE ‘*’” ERROR MESSAGE..................................... 108
7.25 RECEIVING A “SIMULATION SET ‘*’ HAS XX PROJECT TASKS INCLUDED , EXCEEDING THE WORKSPACE SELECTED LIMIT OF * TASKS” ERROR
MESSAGE ............................................................................................................................................................................... 110
7.26 RUNNING A PSCAD V4.6 SIMULATION ON WINDOWS 10 IS SLOWER THAN RUNNING EARLIER VERSIONS OF PSCAD ..................... 111
7.27 RECEIVING AN “‘NMAKE’ IS NOT RECOGNIZED AS AN INTERNAL OR EXTERNAL COMMAND…” ERROR MESSAGE............................... 115
7.28 RECEIVING A “ ‘C:PROGRAM’ IS NOT A VALID INTERNAL OR EXTERNAL COMMAND ” ERROR MESSAGE........................................... 117
7.29 RECEIVING A “LIBENG – LIBRARY IS MISSING” ERROR MESSAGE ............................................................................................ 118
7.30 RECEIVING A “THE SIMULATION PROCESS HAS STOPPED UNEXPECTEDLY ” ERROR MESSAGE WHEN RUNNING ANY SIMULATION ........... 119
7.31 RECEIVING A “NO RULE TO MAKE TARGET ‘*.MAK’. STOP” ERROR MESSAGE ........................................................................... 121
7.32 RECEIVING A SYNTAX ERROR MESSAGE WHEN COMPILING WITH GFORTRAN ........................................................................... 122
7.33 RECEIVING A “SEVERE (41): INSUFFICIENT VIRTUAL MEMORY” ERROR MESSAGE...................................................................... 123
7.34 COMPILING ANY CASE CONTAINING MANY TRANSMISSION LINES IS EXTREMELY SLOW ............................................................. 124
PSCAD - Resolving Launching, Compiling, and Running Issues
7.35 RECEIVING A “GCC.EXE: C:PROGRA~…LIB: NO SUCH FILE OR DIRECTORY” ERROR MESSAGE .................................................... 125
7.36 RECEIVING “UNRESOLVED EXTERNAL SYMBOL _SPRINTF” AND “RETURN CODE '0X460'” ERROR MESSAGES .................................. 126
7.37 RECEIVING A “THE SIMULATION PROCESS HAS STOPPED UNEXPECTEDLY ” MESSAGE .................................................................. 132
7.38 RECEIVING A “MAIN.F(6) : FATAL ERROR C1034” ERROR MESSAGE ..................................................................................... 133
7.39 RECEIVING A “UNABLE TO GENERATE A SIMULATION” ERROR MESSAGE ................................................................................ 135
7.40 RECEIVING AN “EXCHANGE ERROR” MESSAGE .................................................................................................................. 136
7.41 RECEIVING AN “UNABLE TO GENERATE A SIMULATION EXECUTABLE FOR NAMESPACE ” ERROR MESSAGE....................................... 137
7.42 RECEIVING AN “ERROR LNK2005” ERROR MESSAGE......................................................................................................... 138
7.43 RECEIVING AN “ERROR LNK2001” ERROR MESSAGE......................................................................................................... 139
7.44 RECEIVING A “THE SIMULATION PROCESS HAS STOPPED UNEXPECTEDLY ” MESSAGE WHEN WINDOWS GOES TO SLEEP WHILE A CASE IS
RUNNING ................................................................................................................................................................................ 140
7.45 RECEIVING A “SYSTEM ERROR” MESSAGE WHEN COMPILING A PSCAD CASE........................................................................... 141
7.46 RECEIVING A “VISUAL STUDIO 2010, 2012 OR 11 IS NOT FOUND IN THE SYSTEM” ERROR MESSAGE .......................................... 142
7.47 RECEIVING A “‘MAKE’ IS NOT RECOGNIZED…” ERROR MESSAGE WHEN COMPILING WITH GFORTRAN............................................ 145
7.48 RECEIVING A “MAKE (E=5): ACCESS IS DENIED” BUILD ERROR .............................................................................................. 146
7.49 SECURITY SOFTWARE IS PREVENTING SIMULATIONS RESIDING OUTSIDE THE PROGRAM DIRECTORIES FROM RUNNING ...................... 147
7.50 RECEIVING AN “UNABLE TO GENERATE A SIMULATION EXECUTABLE FOR NAMESPACE ‘*’” ERROR MESSAGE .................................. 149
7.51 RECEIVING AN “UNABLE TO GENERATE A SIMULATION EXECUTABLE FOR NAMESPACE ‘*’” ERROR MESSAGE .................................. 150
7.52 RECEIVING A “MAKE FILE ERROR” MESSAGE WHEN COMPILING A PROJECT ............................................................................ 151
7.53 RECEIVING AN “UNABLE TO EXECUTE MAKE” ERROR MESSAGE............................................................................................. 152
7.54 RECEIVING A “…MACHINE TYPE ‘X86’ CONFLICTS WITH TARGET MACHINE TYPE ‘X64’” ERROR MESSAGE ..................................... 153
7.55 RECEIVING A “THIS VERSION OF *\GCC.EXE IS NOT COMPATIBLE WITH THE VERSION OF WINDOWS YOU’RE RUNNING” ERROR MESSAGE
154
7.56 RECEIVING A “VISUAL STUDIO <VERSIONS> IS NOT FOUND IN THE SYSTEM” ERROR MESSAGE .................................................... 155
7.57 RECEIVING A “CANNOT DETERMINE THE LOCATION OF THE VS COMMON TOOLS FOLDER” ERROR MESSAGE ................................. 156
7.58 RECEIVING AN “UNABLE TO GENERATE A SIMULATION EXECUTABLE FOR NAMESPACE ‘*’” ERROR MESSAGE .................................. 159
7.59 RECEIVING AN “ENGOPEN INVALIDCHECK MATLAB INSTALLATION!’” WARNING MESSAGE ........................................................ 160
7.60 RECEIVING A “MAKE: *** NO RULE TO MAKE TARGET ‘*.MAK’” ERROR MESSAGE ................................................................... 162
7.61 UNABLE TO COMPILE A PROJECT .................................................................................................................................... 163
7.62 UNABLE TO COMPILE A PROJECT .................................................................................................................................... 164
7.63 RECEIVING AN “UNABLE TO FIND MICROSOFT VISUAL C++ *** OR HIGHER” BUILD ERROR ....................................................... 165
7.64 UNABLE TO BUILD A PSCAD PROJECT USING GFORTRAN 4.2.1 OR GFORTRAN 4.6.2.............................................................. 166
7.65 UNABLE TO BUILD A PROJECT IN THE PSCAD FREE EDITION USING GFORTRAN 4.2.1 OR GFORTRAN 4.6.2 ................................. 167
7.66 RECEIVING AN “ERROR: VISUAL STUDIO 2013, 2015 OR 2017 IS NOT FOUND IN THE SYSTEM” MESSAGE ................................. 168
7.67 RECEIVING A “'C' IS NOT RECOGNIZED AS AN INTERNAL OR EXTERNAL COMMAND OPERABLE PROGRAM OR BATCH FILE ” MESSAGE ..... 169
7.68 RECEIVING AN “EXPORT ARGUMENT '*' CANNOT BE DECLARED SINCE IT IS ALREADY DECLARED AS A LOCAL SIGNAL ” ERROR MESSAGE 171
7.69 RECEIVING A “CONNECT FUNCTION FAILED WITH ERROR: 10060” ERROR MESSAGE ................................................................. 172
7.70 RECEIVING ERROR “MAKE: *** [<SOME FILE NAME>.EXE] ERROR 1” ERROR MESSAGE WITH AN ETRAN PRECOMPILED LIBRARY ..... 173
7.71 RECEIVING “ERROR 1” ................................................................................................................................................. 174
7.72 RECEIVING “THE SYSTEM CANNOT FIND THE PATH SPECIFIED ” ERROR MESSAGE ON A MACHINE WITH ANACONDA AND POWERSHELL175
7.73 RECEIVING A “LINK : FATAL ERROR LNK1181: CANNOT OPEN INPUT FILE 'WS2_32.LIB'” ERROR MESSAGE ................................. 176
7.74 RECEIVING A “FATAL ERROR: PARSE ERROR WHEN CHECKING MODULE VERSION FOR FILE 'NDDE.COM' OPENED AT (1)” ERROR MESSAGE
180
7.75 RECEIVING A “REACHED BUFFER LIMIT:” ERROR MESSAGE WHEN CALLING IN A BLACKBOXED MODULE ...................................... 181
7.76 RECEIVING A "CANNOT OPEN FILE 'LIBUCRT.LIB'" ERROR MESSAGE........................................................................................ 182
7.77 RECEIVING AN "UNRESOLVED EXTERNAL SYMBOL" ERROR MESSAGE ...................................................................................... 185
7.78 RECEIVING THE ERROR MESSAGE “WINDOWS IS NOT GENERATING SHORT (8.3) PATHNAMES” ERROR MESSAGE IN THE FORTRAN MEDIC
UTILITY 187
7.79 RECEIVING THE ERROR MESSAGE “” ERROR MESSAGE WHEN TRYING TO BUILD A PROJECT ........................................................ 188
7.80 BUILD START IS DELAYED .............................................................................................................................................. 189
7.81 MICROSOFT® VISUAL STUDIO IS NOT DETECTED ................................................................................................................ 190
7.82 RECEIVING THE ERROR “ONLY A SINGLE NAMESPACE IS INCLUDED IN THE SIMULATION SET” ........................................................ 191
7.83 RECEIVING “FATAL ERROR C1900” ERROR MESSAGE ......................................................................................................... 192
PSCAD - Resolving Launching, Compiling, and Running Issues
7.84 RECEIVING A “THE APPLICATION WAS UNABLE TO START CORRECTLY (0XC0000142)” ERROR MESSAGE ...................................... 193
7.85 RECEIVING AN “IFORTVARS.BAT...IS NOT RECOGNIZED AS AN INTERNAL OR EXTERNAL COMMAND ” ERROR MESSAGE ...................... 194
7.86 RECEIVING A “VCVARSQUERYREGISTRY.BAT”’ IS NOT RECOGNIZED…” ERROR MESSAGE .......................................................... 195
7.87 RECEIVING A “WINDOWS WAS UNEXPECTED AT THIS TIME ” ERROR MESSAGE ......................................................................... 196
7.88 RECEIVING AN “UNABLE TO LOAD C2.DLL” ERROR MESSAGE ................................................................................................ 197
7.89 RECEIVING A “INPUT FILE '\WINDOWS\EMTDC.CFG' DOES NOT EXIST” ERROR MESSAGE............................................................ 198
7.90 RECEIVING A “THE TOOLS FOR THE CONFIGURATION MIGHT NOT BE INSTALLED ” ERROR MESSAGE............................................... 199
7.91 RECEIVING A “NMAKE : FATAL ERROR U1073: DON’T’ KNOW HOW TO MAKE…” ERROR MESSAGE ........................................... 200
7.92 RECEIVING A “NMAKE : VISUAL STUDIO VERSION ERROR: UNKNOWN ERROR ENVIRONMENT CONFIGURED” ERROR MESSAGE ........ 201
7.93 RECEIVING A “LINK : FATAL ERROR LNK1104: CANNOT OPEN FILE 'IFCONSOL.LIB'” ERROR MESSAGE .......................................... 202
7.94 RECEIVING A “CANNOT OPEN OUTPUT FILE VDIV .EXE: NO SUCH FILE OR DIRECTORY” ERROR MESSAGE ......................................... 203
7.95 RECEIVING A “THIS ACTION CANNOT BE COMPLETED BECAUSE THE OTHER PROGRAM IS BUSY ” ERROR MESSAGE ............................ 206
7.96 RECEIVING A “LINK: FATAL ERROR LNK1181: CANNOT OPEN INPUT FILE 'WSOCK32.LIB'” ERROR MESSAGE ................................ 207
7.97 RECEIVING A “MAKE: *** INTERNAL: READDIR: INVALID ARGUMENT” ERROR MESSAGE......................................................... 208
7.98 RECEIVING IFORT ERRORS #10037 AND #10408, AND FATAL ERROR U1077........................................................................ 209
7.99 RECEIVING THE ERROR “THIS PROGRAM IS BLOCKED BY GROUP POLICY ” ................................................................................. 210
7.100 RECEIVING THE ERRORS “WAITING FOR UNFINISHED JOBS” AND “UNEXPECTED END OF FILE IN ‘*.F” ............................................ 211
7.101 RECEIVING ERROR LNK2038 “…MISMATCH DETECTED FOR COMPILING THE PROJECT …” .......................................................... 212
7.102 RECEIVING A “'MAKE' IS NOT RECOGNIZED AS AN INTERNAL OR EXTERNAL COMMAND ” BUILD MESSAGE ...................................... 213
7.103 RECEIVING ERROR “VDIV.MAK(4) : FATAL ERROR U1033: SYNTAX ERROR : ‘FORTRAN’ UNEXPECTED” ......................................... 214
7.104 RECEIVING ERRORS “NO SUCH FILE OR DIRECTORY”, “ERROR 1”, AND “WAITING FOR UNFINISHED JOBS”..................................... 216
7.105 RECEIVING ERRORS “SYNTAXERROR: INVALID SYNTAX”, “NMAKE : FATAL ERROR U1077…” AND “RETURN CODE '0X1'” .............. 218
7.106 RECEIVING BUILD MESSAGES INDICATING THAT MICROSOFT VISUAL STUDIO IS NOT DETECTED .................................................. 220
7.107 RECEIVING A “LIBUCRT.LIB(EXIT.OBJ) : FATAL ERROR LNK1112” ERROR MESSAGE .................................................................. 221
7.108 RECEIVING A “LIBUCRT.LIB(EXIT.OBJ) : FATAL ERROR LNK1112” ERROR MESSAGE .................................................................. 222
7.109 RECEIVING A “PROJECT OUTPUT STORAGE REQUIREMENTS ARE XX MB…” ERROR MESSAGE ..................................................... 223
7.110 RECEIVING A “THE STORAGE TABLE IS AT CAPACITY OF 256” ERROR MESSAGE ........................................................................ 224
7.111 RECEIVING A “NMAKE: FATAL ERROR U1077…RETURN CODE ‘0X460’” ERROR MESSAGE ...................................................... 225
7.112 GFORTRAN SUDDENLY QUITS WORKING FOLLOWING AN OPERATING SYSTEM UPDATE ............................................................ 226
7.113 WHEN MULTIPLE VERSIONS OF INTEL ONEAPI ARE INSTALLED, THE NEWEST INSTALLED VERSION WILL ALWAYS BE CALLED ............ 227
7.114 RECEIVING ERROR #1455............................................................................................................................................. 228
7.115 RECEIVING ERROR “ERROR: A LICENSE FOR FCOMPW IS NOT AVAILABLE NOW” ...................................................................... 229
7.116 RECEIVING ERROR “'GFORTRAN.EXE' IS NOT RECOGNIZED AS AN INTERNAL OR EXTERNAL COMMAND …” ....................................... 230
7.117 RECEIVING ERROR “WAITING FOR UNFINISHED JOBS” ......................................................................................................... 231
7.118 RECEIVING ERRORS “IFORT.EXE IS NOT RECOGNIZED …” AND “NMAKE : FATAL ERROR U1077: 'IFORT.EXE'…” ............................. 232
7.119 AN INTEL FORTRAN COMPILER CANNOT BE SELECTED IN THE PSCAD APPLICATION OPTIONS | DEPENDENCIES DIALOG .................. 233
7.120 RECEIVING ERRORS: 'IFORT.EXE' IS NOT RECOGNIZED AS AN INTERNAL OR EXTERNAL COMMAND , OPERABLE PROGRAM OR BATCH FILE”,
AND “FATAL ERROR U1077”, AND “RETURN CODE '0X1'”................................................................................................................ 234
7.121 RECEIVING ERROR MESSAGE "SYMBOL 'NODE' AT (<SOMENUMBER>) CONFLICTS WITH SYMBOL FROM MODULE
'<SOMEMODULENAME>'" ........................................................................................................................................................ 235
7.122 RECEIVING BUILD ERROR “SETVARS.BAT”’ IS NOT RECOGNIZED AS AN INTERNAL OR EXTERNAL COMMAND ”................................... 236
7.123 RECEIVING BUILD ERROR “\INTEL\ONEAPI\COMPILER\<VERSION>\WINDOWS\BIN WAS UNEXPECTED AT THIS TIME” .................... 237
7.124 RECEIVING BUILD ERROR “VISUAL STUDIO WAS NOT FOUND IN THE STANDARD INSTALLATION LOCATION ” .................................... 238
7.125 PSCAD V5.0.2 CRASHES WITH ERROR 0XC0000409 DURING BUILD .................................................................................. 239
7.126 RECEIVING BUILD ERROR “VISUAL STUDIO COMMAND-LINE ENVIRONMENT WAS NOT CONFIGURED” ........................................... 240
8. ISSUES WHEN RUNNING CASES IN PSCAD ....................................................................................... 241
8.1 RECEIVING A “PROJECT OUTPUT STORAGE REQUIREMENTS ARE XX MB” WARNING .................................................................. 241
8.2 RECEIVING AN “EMTDC RUNTIME ERROR” MESSAGE ....................................................................................................... 242
8.3 RECEIVING AN “ABNORMAL TERMINATION OF EMTDC BY *” ERROR MESSAGE ....................................................................... 244
8.4 RECEIVING A “SERVER BUSY” ERROR MESSAGE................................................................................................................. 245
8.5 RECEIVING A “RESULT TOO LARGE” RUNTIME ERROR MESSAGE............................................................................................ 246
8.6 RECEIVING ERROR “CONNECTION FUNCTION FAILED WITH ERROR: 10013” RUNTIME ERROR MESSAGE ....................................... 247
PSCAD - Resolving Launching, Compiling, and Running Issues
8.7 RECEIVING A “SINGULARITY (A ZERO DIAGONAL) ENCOUNTERED” RUNTIME ERROR MESSAGE .................................................... 248
8.8 UNABLE TO RUN ANY PSCAD CASES – THE REGIONAL LANGUAGE ISSUE ............................................................................... 249
8.9 RECEIVING A “PROCESS…CANNOT EXECUTE COMMAND …ERROR #5” ERROR MESSAGE............................................................ 254
8.10 RECEIVING A “THE SIMULATION PROCESS HAS STOPPED UNEXPECTEDLY . PLEASE REVIEW RUNTIME MESSAGES FOR DETAILS” ERROR
MESSAGE ............................................................................................................................................................................... 255
8.11 OTAINING DIFFERENT SIMULATION RESULTS FOR A CASE COMPILED WITH GFORTRAN AND INTEL ............................................... 257
8.12 OBTAINING ERROR 0XC0000005 WHEN ATTEMPTING TO RUN PSCAD CASES...................................................................... 257
8.13 CANNOT RUN A PROJECT WHEN CONNECTED OVER VPN ................................................................................................... 258
8.14 RUNNING PSCAD CASES PROMPTS YOU FOR WINDOWS ADMINISTRATOR RIGHTS TO EXECUTE ................................................. 259
8.15 RECEIVING RUNTIME ERROR “ACCESS IS DENIED” .............................................................................................................. 260
8.16 RECEIVING RUNTIME ERROR “THE SIMULATION IS NO LONGER RESPONDING AND MAY HAVE AN UNEXPECTED DISCONNECT ” ............ 261
8.17 SIMULATION BUILDS AND STARTS TO RUN, BUT THE RUN STALLS .......................................................................................... 262
8.18 A PROJECT THAT RUNS IN V5.0.0 FAILS TO BUILD IN V5.0.1 WITH ERROR “ABNORMAL TERMINATION OF EMTDC BY DSLINT” AND
“STATUS CODE = 2000”............................................................................................................................................................ 265
8.19 RECEIVING ERROR “UNABLE TO ASSIGN DATA PORT FOR A SIMULATION PROCESS. ABORTING LAUNCH.” ....................................... 266
8.20 THE PLOTS DO NOT BECOME POPULATED DURING A RUN .................................................................................................. 267
8.21 PSCAD HANGS AFTER A BUILD, AND DOES NOT RUN ........................................................................................................ 268
8.22 RECEIVING THE RUNTIME ERROR: “THIS PROGRAM IS BLOCKED BY GROUP POLICY ”................................................................... 269
8.23 THE RUN FAILS WHEN INITIALIZING A LARGE NUMBER OF PGBS IN PSCAD V5.0.2 ................................................................. 270
9. ISSUES WITH MYCENTRE............................................................................................................... 271
10. ISSUES WITH DIAGNOSTIC TOOLS .................................................................................................. 272
11. RESOLVING FORTRAN CODING ISSUES ........................................................................................... 273
11.1 RECEIVING AN “UNABLE TO GENERATE A SIMULATION EXECUTABLE ” ERROR MESSAGE.............................................................. 273
APPENDIX A USING THE FORTRAN MEDIC UTILITY............................................................................................. 274
A.1 OVERVIEW ................................................................................................................................................................. 274
A.2 RUNNING THE FORTRAN MEDIC UTILITY .......................................................................................................................... 274
A.3 GENERATING THE LOG FILE............................................................................................................................................ 274
A.4 FIXING ISSUES USING THE UTILITY ................................................................................................................................... 274
A.5 ERRORS LISTED IN THE UTILITY ....................................................................................................................................... 276
A.6 FUNCTIONS LISTED IN THE MEDIC ................................................................................................................................... 300
APPENDIX B USING THE GET INFO UTILITY ....................................................................................................... 305
APPENDIX C LOCK-BASED LEGACY– LICENSING - LICENSE MANAGER REQUIREMENTS ........................................... 306
APPENDIX D CERTIFICATE LICENSING – REQUIREMENTS..................................................................................... 307
APPENDIX E HOW TO DISPLAY A HIDDEN FOLDER ............................................................................................. 308
APPENDIX F FILE CREATION DURING A SIMULATION ......................................................................................... 309
APPENDIX G TESTING CONNECTIVITY FOR CERTIFICATE LICENSING ..................................................................... 312
PSCAD - Resolving Launching, Compiling, and Running Issues
1. Introduction
1.1 Overview
This manual presents known issues and solutions related to PSCAD and associated software. It is intended to be a
continuously developing diagnostic tool as the software evolves.
The information in this manual is applicable to PSCAD X4 and V5, which includes version 4.3.0 and newer, and covers the
following topics:
• Section 2: Launching PSCAD (Section 2)
• Section 3: Content (lock-based licensing issues) moved to new manual “Resolving PSCAD Lock-Based Licensing Issues” (see
Section 1.2)
• Section 4: Content (certificate licensing issues) moved to new manual “Resolving Certificate Licensing Issues” (see Section 1.2)
• Section 5: Content (Issues when Licensing PSCAD – Legacy Lockless) moved to new manual “Resolving Lock-based Licensing
Issues” (see Section 1.2)
Page 7
PSCAD - Resolving Launching, Compiling, and Running Issues
Problem
When launching PSCAD, the following error message displays:
PSCAD
Page 8
PSCAD - Resolving Launching, Compiling, and Running Issues
Solution 2
Fix this using the Fortran Medic utility:
• Run the utility as per Appendix A.
• Once the utility has retrieved the information, scroll down to the following message:
• Right-click on the “Workspace file” message and select the option to delete this file.
Page 9
PSCAD - Resolving Launching, Compiling, and Running Issues
Problem
When PSCAD is launched, it may take up to several minutes before the application is ready for user input. Specifically, when
PSCAD is launched, the PSCAD splash window remains displayed for an extended time, during which the application is
neither licensed nor usable.
System
• PSCAD v4.5.1 to v4.5.3
• All Windows platforms
• Using either the “elevated” or “non-elevated” launch links in the Windows Start menu.
Cause
The PSCAD Start Page cannot load, and is delaying the start of the application.
Solution 1
Update to PSCAD v4.5.4 or later.
Solution 2
Fix this using the Fortran Medic utility:
• Run the utility as per Appendix A.
• Once the utility has retrieved the information, scroll down to the following error message:
Conflicts
PSCAD will not display the latest Start Page.
• Right-click on the error message and select the option to repair it.
Page 10
PSCAD - Resolving Launching, Compiling, and Running Issues
Problem
When PSCAD is launched, the following error message displays:
System
• PSCAD v4.5.1 to v4.5.3
• All Windows platforms
Cause
The PSCAD Start Page cannot load, and is delaying the start of the application.
Solution 1
Update to PSCAD v4.5.4 or later.
Solution 2
Fix this using the Fortran Medic utility:
• Run the utility as per Appendix A.
• Once the utility has retrieved the information, scroll down to the following error message:
Conflicts
PSCAD will not display the latest Start Page.
• Right-click on the error message and select the option to repair it.
Page 11
PSCAD - Resolving Launching, Compiling, and Running Issues
2.4 Receiving an “Unable to locate the master library from the given file path” Error Message
Problem
When launching PSCAD, a message similar to the following is displayed:
Unable to locate the master library from the given file path:
C:PSCADProjects\Project1\Project1.o
Check the specified folder path.
Cause
An incorrect file type (for example, .o) was entered into a field reserved only for library files (.pslx). See location of this
library file field, below.
Page 12
PSCAD - Resolving Launching, Compiling, and Running Issues
Solution
(1) If PSCAD crashed: Delete your PSCAD user profile to restore the master library, as follows:
Run the Fortran Medic utility (per Appendix A1).
Once the utility has finished retrieving information, scroll down to the PSCAD version number, and note the path
for the “Workspace file”.
Open a browser to this file path, and delete the user_profile.xml file.
Proceed to Step (3) below.
(2) If PSCAD did not crash: Restore the master library in the PSCAD application as follows:
Display the Master Library field:
Delete the text text in this field, and enter the following: $(HomeDir)\master.pslx
Page 13
PSCAD - Resolving Launching, Compiling, and Running Issues
Problem
When launching PSCAD, a message similar to the following is displayed:
Script Error
Line: 6
Char: 31
Error: ___________null________“module”
Code: 0
URL: https://mycentre.hvdc.ca/static/js/angular/angular-
resource.min.js
Cause
The PSCAD “Start” page, which has links to YouTube, is being blocked. Possible causes are firewalls and anti-virus. These
blocks might be applied locally on the user’s computer, or more broadly across the network of an organization or even a
country.
Note
The Start page is a tab in PSCAD containing the same information and help videos that are available in MyCentre
(https://mycentre.hvdc.ca/).
Solution 1
Turn off the software that is blocking the start page (firewall or anti-virus).
Page 14
PSCAD - Resolving Launching, Compiling, and Running Issues
Solution 2
Disable the Start Page so that it does not attempt to load upon launching PSCAD, as follows:
• Launch PSCAD.
• Display the Application Options dialog box:
• In the Workspace page, select “No action” from the “Start Page” drop-down button:
• The Start Page will not attempt to display upon PSCAD startup.
Page 15
PSCAD - Resolving Launching, Compiling, and Running Issues
Problem
When PSCAD is launched, the following error message is displayed:
Cause
This message displays because the Visual C++ 2010 SP1 Redistributables is missing.
Solution
Use the Fortran Medic utility to install the Visual C++ 2010 SP1 Redistributables, as follows:
• Run the latest Fortran Medic utility as per Appendix A.2.
• For a Windows 32-bit machine: Select Help | Install Visual C++ 2010 SP1 Redistributables (x86).
• For a Windows 64-bit machine: Select Help | Install Visual C++ 2010 SP1 Redistributables (x64).
Page 16
PSCAD - Resolving Launching, Compiling, and Running Issues
2.7 Receiving a “The procedure entry point … dynamic link library SHELL32.dl” Error Message
Problem
When PSCAD v4.6.0 or later is launched, an error message similar to the following is displayed:
OK
Cause
This message displays because the user is trying to run PSCAD on an unsupported operating system (Windows XP).
Solution
PSCAD v4.6.0 and later is not supported on Windows XP. This software is officially supported on Windows Vista and
Windows 7, but has also been shown to work on Windows 8.
Page 17
PSCAD - Resolving Launching, Compiling, and Running Issues
2.8 Receiving a “The procedure entry point …could not be located in the dynamic link library ZSlib2.dll”
Error Message
Problem
When PSCAD is launched, an error message similar to the following is displayed:
Cause
A problem has developed with the PSCAD program files.
Solution
a. Delete the PSCAD program folder and all subfolders and files. These will typically be located in a path similar to the
following:
• C:\Program Files (x86)\PSCADx
b. Re-install or repair PSCAD. This will re-install all the required files.
For Additional Information
See Appendix A.5, Item 10.
Page 18
PSCAD - Resolving Launching, Compiling, and Running Issues
Problem
PSCAD crashes upon startup.
System
Most or all versions of PSCAD.
Cause
The corporate security package, Digital Guardian, can cause PSCAD to crash after .NET 4.6.1 is installed.
Solution
Whitelist PSCAD in the corporate security package.
Page 19
PSCAD - Resolving Launching, Compiling, and Running Issues
Problem
When launching the PSCAD Beta Edition from MyUpdater, the launch fails:
• The following error message displays in the MyUpdater messages tab:
Process terminated with error: E0434352
Runtime Error!
Program: C…
R6030
- CRT not initialized
System
PSCAD Beta Edition on Windows 7 SP1.
Cause 1
This problem is most often caused by certain security software programs. It is also possible to get this error when another
program uses code injection techniques to trap certain DLL library calls. Some intrusive security programs use this
technique. In versions of Visual C++ before Visual Studio 2015, it is possible to use a statically-linked CRT library to address
the issue, but this is not recommended for reasons of security and application updates.
Solutions 1
1. Disable your security software and see if this resolves the issue.
2. Your security software may have specific instructions for mitigating this issue. Check your security software
vendor's website for details.
3. Check for updated versions of your security software, or try different security software.
4. Re-install the Visual C++ Redistributables, then reboot your machine, and then try launching the Beta Edition again.
5. Try this installation on a different machine.
Cause 2
You may have corrupt Windows files.
Solutions 2
Have your IT staff perform an sfc scan according to the following instructions:
http://support.microsoft.com/kb/929833
If the above step fails to fix the problem, try repairing Windows.
Page 20
PSCAD - Resolving Launching, Compiling, and Running Issues
Problem
When launching PSCAD v4.6.0 or v4.6.1, the launch fails.
Note
PSCAD 32-bit does run.
Cause
There is a problem with the .NET framework installation.
Solution
Resolve the .NET Framework software installation (e.g. re-install fresh, or install the patches/updates).
Page 21
PSCAD - Resolving Launching, Compiling, and Running Issues
2.12 Receiving a “The computer must be trusted for delegation…” error message
Problem
When launching PSCAD, the launch fails, and the following messages are displayed:
Application.ThreadException
Type: System.Exception
Invoking Type: CredentialManagerLib.UCredentialManager
Method: Void initialize
Cause: The requested operation cannot be completed. The computer must be trusted for delegation and the current user account must be
configured to allow delegation.
Cause
“Mandatory profiles” are being used on the machine.
Mandatory profiles, which are essentially read-only profiles, do not support certain cryptographic functions which are
essential to the applications.
Specifically, these functions are essential for the following tasks:
• To create a new user key;
• To use functions which are used by PSCAD and the MyUpdater to cache and store the user’s MyCentre credentials,
which are required by the following:
By the MyUpdater utility to log into the user’s MyCentre account to retrieve the list of available products, and
By PSCAD to log into the user’s MyCentre account to obtain a new PSCAD license certificate.
• Currently, this also appears to be an issue even if the client would like to use the legacy lock-based License
Manager with PSCAD.
The latest Fortran Medic tool indicates if a user’s profile is mandatory. Refer to Appendix A.5,
Item 29.
Applicable to
Will likely impact the following:
• PSCAD v4.5.4 to v4.6.2 and the Free Edition
• Enerplot, FACE, Initializer, Update Client (MyUpdater)
Solutions
• Allow users to use non-mandatory profiles.
Note
PSCAD 4.6.2 is designed to not require local admin rights, and should be useable by normal users, however, this
is also dependent on certain Windows 10 Group Policy settings.
• Allow the PSCAD users to log in to local (non-domain) accounts on the machines hosting PSCAD.
2.13 How to launch PSCAD without Windows Administrator Privileges
Objective
For some facilities, users are not provided Windows Administrator privileges, and therefore must be able to launch PSCAD
with Windows Users privileges.
Page 22
PSCAD - Resolving Launching, Compiling, and Running Issues
Solution (1)
Pin a non-elevated PSCAD shortcut to the Windows Taskbar, Windows Start menu, or to the desktop. For example, for
PSCAD v4.6.2:
• Open a Windows file browser to here:
C:\Program Files (x86)\PSCAD46\bin\win64
• Right-click on “pscad.exe” file and select “Properties”. Review the properties, and ensure that this link is non-
elevated.
• Right-click on “pscad.exe”, and select the required action:
“Pin to Taskbar”, or
“Pin to Start Menu”, or
“Send to”…”Desktop”
Solution (3)
If the Solutions (1) and (2) do not work, one suggestion is to use the Microsoft Application Compatibility Toolkit to create a
custom security DB, and apply the DB to the PSCAD application. This should then force the application to run in the context
of the person trying to launch the application.
Page 23
PSCAD - Resolving Launching, Compiling, and Running Issues
2.14 Receiving a “The application was unable to start correctly (0xc00007b)” Error Message
Problem
When launching PSCAD, the following error displays:
System
This was detected when trying to launch PSCAD v4.6.2 with the following setup:
• Windows 7 SP1 Operating System
• Visual C++ 2015 Redistributable (x64) (14.0.24215.1)
Cause
Microsoft Visual C++ 2015 Redistributables is corrupted.
Solution
Re-install Microsoft Visual C++ 2015 Redistributables.
Page 24
PSCAD - Resolving Launching, Compiling, and Running Issues
Problem
When launching PSCAD, the following error displays:
Cause
The PSCAD installation became corrupted because PSCAD v4.6.1 was installed first, and then PSCAD v4.6.0 was allowed to
be installed.
Solution
Uninstall all versions of PSCAD v4.6.*, then install v4.6.1.
Note
It is possible to install multiple versions within the same branch on a machine, for example, v4.6.0 and v4.6.1.
Normally, a version within a branch (i.e. “patch”) will overwrite the previous version.
To circumvent this, install the latest version first (e.g. v4.6.1), ensuring to save the files to a new program folder
when prompted (e.g. “PSCAD v461”) rather than to the default folder (“PSCAD46”). Next, install the earlier
version (e.g. v4.6.0), ensuring to save the files to another new program folder when prompted (e.g. “PSCAD
v460”).
Page 25
PSCAD - Resolving Launching, Compiling, and Running Issues
2.16 Receiving Missing Critical File Messages when trying to launch PSCAD
Problem
When launching PSCAD, errors similar to the following display:
PSCAD
Startup Error
Startup Error
Input file
‘C:\...\links.prop
does not exist.
PSCAD
Cause (1)
The PSCAD software was installed on a machine protected by Beyond Trust, which causes the software to not fully install,
especially the required PSCAD registry keys.
Solution (1)
Contact the Beyond Trust support staff for assistance.
Continued…
Page 26
PSCAD - Resolving Launching, Compiling, and Running Issues
Cause (2)
Possibly, the PSCAD installation could have been corrupt, or anti-virus software may have corrupted the program files.
Solution (2)
• If it is suspected that anti-virus software corrupted your program files, turn this off temporarily.
• If it suspected that the PSCAD installation files were corrupted, obtain a new, trusted download of PSCAD
([email protected]).
• Uninstall PSCAD.
• Delete any desktop shortcuts to PSCAD.
• Re-install PSCAD.
• PSCAD should be able to run.
Cause (3)
Certain permissions are not available to a Windows “user”.
Solution (3)
Run PSCAD once with elevated privileges (right-click on your PSCAD launch link, and select “Run as administrator”), and
PSCAD should launch with no errors. It may be possible to launch PSCAD as normal user thereafter.
Page 27
PSCAD - Resolving Launching, Compiling, and Running Issues
2.17 Receiving an “Unable to find or create the required version registry keys” Error Message
Problem
When trying to launch PSCAD, the launch fails, and the following error displays:
Cause
The Windows Registry keys for PSCAD are not permitted to be modified. Therefore, when PSCAD is launched, and attempts
to modify the Registry Keys, this fails, and PSCAD cannot be run.
These Windows Registry keys for PSCAD are listed in the requirements document, which may be viewed from this webpage.
Solution (1) Preferred – Provide Permissions for Modifying the Windows Registry Keys for PSCAD
Configure full permissions for the Windows Registry Keys for PSCAD. The corresponding keys are listed in the above
webpage. PSCAD may then be launched, PSCAD will modify the Registry Keys, and then can be run.
Solution (2) Run PSCAD Initially with Windows Administrator Privileges
Launch PSCAD with Windows Administrator privileges (from the Windows Start menu, browse to the PSCAD XX link, right-
click and select Run as administrator, then select Yes when prompted by the Windows UAC). PSCAD should be able to
create the those registry points, and thereafter, PSCAD should be able to be launched as a“normal” Windows User.
If the Matter Persists
Please download and run our latest Fortran Medic utility, and send in the generated log file to the MHI support desk as per
these instructions.
Page 28
PSCAD - Resolving Launching, Compiling, and Running Issues
2.18 Receiving an “The code execution cannot proceed because mfc140u.dll was not found” Error Message
Problem
PSCAD 5.0.1 is installed on a new Widows 10 machine by a person who has admin rights.
The following redistributables are installed on the machine:
• Visual C++ 2017 Redistributable (x64) 14.16.27033.0. which is the latest release for this version,
• Visual C++ 2019 Redistributable (x86) 14.24.28127.4, which is the 6th of 17 releases for this version
When trying to launch PSCAD, the launch fails, and the following error displays:
Cause
Possibly corrupted install of the existing Visual C++ Redistributables.
Background
PSCAD requires the Visual C++ 2015 Redistributables, (x86) and (x64) or better, meaning that it can use any of the following
versions:
• Visual C++ 2015 - 2022 Redistributables, (x86) and (x64)
Solution
Download and install the latest available Microsoft Visual C++ 2022 Redistributables, (x86) and (x64).
Information on downloading and installing these products is listed in this article.
Page 29
PSCAD - Resolving Launching, Compiling, and Running Issues
2.19 When Launching PSCAD From a PSCAD Project, Cannot Launch the Intended Version
Problem
Some users prefer to launch PSCAD from a project file, rather than by launching the PSCAD application itself.
This is supported of course.
However, if there are multiple versions of PSCAD installed on that machine, then by default, the most recently-installed
version will be the one that is launched from a project file.
For example, if PSCAD v5.0.1 is installed first, and then PSCAD v4.6.3 is installed second, then clicking on any PSCAD
project file will always launch PSCAD v4.6.3.
The first problem with this is that the most recently-installed might not be the desired version.
The second problem with this is that the recently-installed version might not even support the project.
For example, some new features added into V5 are not backwards compatible to X4.
Note
It may seem intuitive to try to set the version from within the file properties. However, this method does not
seem possible.
Page 30
PSCAD - Resolving Launching, Compiling, and Running Issues
2.20 The Error “The End User License Agreement could not be found…” Displays upon Launching
Problem
Upon launching PSCAD, the error as shown below displays, and no further actions are allowed.
Cause
There has been some sort of issue with the installation of PSCAD.
Possibly anti-virus software is causing this issue.
Solution
Uninstall PSCAD, tune down or turn off your anti-virus software, then re-download and re-install the software.
The License Agreement page should display, and allow you to accept the Agreement, and then use the program.
Page 31
PSCAD - Resolving Launching, Compiling, and Running Issues
Page 32
PSCAD - Resolving Launching, Compiling, and Running Issues
Page 33
PSCAD - Resolving Launching, Compiling, and Running Issues
Page 34
PSCAD - Resolving Launching, Compiling, and Running Issues
Problem
PSCAD crashes when the “Associations” tab is selected in the “System Settings” dialog.
System
PSCAD v4.5.1
Cause
PSCAD v4.5.1 crashes due to a missing file: “external_tools.xml”.
Solution 1
Use the Fortran Medic utility to repair the problem:
a. Download the latest “FortranMedic” from our website:
http://updater.pscad.com/utilities/FortranMedic.zip
b. In the downloaded .zip file, run the FortranMedic.exe file.
c. Click on the “Actions” menu and select “Start”.
d. After the utility is done retrieving information:
• Scroll down to: Installed PSCAD versions | PSCAD 4.5.1.
• Locate the error in red text related to the missing file: “external_tools.xml”.
• Right-click on the error, and from the displayed menu, select the option to restore this file.
Solution 2
Obtain the file “external_tools.xml” from our Support Desk, and save it to the following location on Windows Vista and
Windows 7:
C:\Users\YourUserID\AppData\Local\Manitoba HVDC Research Centre\PSCAD
(if the appdata folder is hidden, it may be displayed as per Appendix E)
Solution 3
Update your software to v4.5.2 or later (contact our Sales Desk at [email protected]).
Page 35
PSCAD - Resolving Launching, Compiling, and Running Issues
Problem
The Component Wizard pane is not displaying correctly:
Page 36
PSCAD - Resolving Launching, Compiling, and Running Issues
Note
The correct layout for the Component Wizard is as follows:
System
PSCAD v4.5.2 and v4.5.3 x64, on platform with Windows 7
(6.01.7601 Service Pack 1)
Note
This problem does not exist using Windows 7 x86 platform.
Cause
An issue with Internet Explorer 11.
Solution
Update to PSCAD v4.5.5 or later, or
Uninstall all Internet Explorer versions, then install and use Internet Explorer 9.
Page 37
PSCAD - Resolving Launching, Compiling, and Running Issues
6.3 Receiving a “The PSCAD automated email system has failed to send your request” Message
Problem
When the “Submit” button is selected in the PSCAD application…
…the following four messages are displayed, and the support request is not e-mailed:
There was no default logon, and the user failed The PSCAD automated email system has failed to send your
to log on successfully when the logon dialog box
request.
was displayed. No message was sent.
Page 38
PSCAD - Resolving Launching, Compiling, and Running Issues
System
This occurs in PSCAD v4.5 on a select number of machines.
Cause
The cause is unknown, but is likely related to machine settings.
Solution
Consult with your IT personnel to determine what machine settings are preventing PSCAD from sending an e-mail using
your e-mail tool and your e-mail server.
Page 39
PSCAD - Resolving Launching, Compiling, and Running Issues
Problem
When PSCAD is launched, the following error message is displayed in the canvas:
Unable to display start page. Detected version of Internet Explorer is not supported.
Required: Version 7 or better
Detected: Version 7
Applicable Software
This is applicable to licensed versions of PSCAD (Educational, Professional, Trial), for versions 4.5.1, 4.5.2, and 4.5.3.
Solution 1
This has been fixed in later versions; update your software to v4.5.4 or later.
Solution 2
Use the Fortran Medic utility to resolve this issue as follows:
• Download the latest “FortranMedic” from our website:
http://updater.pscad.com/utilities/FortranMedic.zip
• The PSCAD Start Page in the PSCAD application should be available for display.
Page 40
PSCAD - Resolving Launching, Compiling, and Running Issues
6.5 Copying a Control will Switch its Value to the Default Setting
Problem
When a control is copied, the value of the original control switches to the default setting.
For example, if the following controls are copied…
Solution
Update to v4.5.4 or later; this is a bug that was present in earlier versions of PSCAD.
Page 41
PSCAD - Resolving Launching, Compiling, and Running Issues
Problem
When PSCAD is opened, the following error message displays in the canvas:
Unable to connect to
MyCentre server.
Applicable Software
This is applicable for versions 4.5.1, 4.5.2, and 4.5.3. It is less likely to appear when running v4.5.4
Note
This error is not related to using PSCAD, and is not critical. This error is displaying because PSCAD is unable to access the
MyCentre server, and cannot download and display information on the “Start Page”, as shown below.
The information that is not displaying is basically the same as that shown in the Home tab of this website.
Cause
Some of the common reasons that this message displays are if:
• You do not have internet connection, or
• Your internet is too slow, or
• MyCentre is down
Solution (1)
If using PSCAD v4.5.3 or older, update your software.
Version 4.5.4 and better have a slightly better handling for this. Version 4.5.3 tries to connect once, whereas version newer
versions will try to connect several times before giving up and displaying this message.
Solution (2)
Press the F5 key to prompt the Start Page to try loading again.
Solution (3)
Disable the Start Page, so that PSCAD will not attempt to contact the server, and the error will not display:
• In PSCAD V5:
Page 42
PSCAD - Resolving Launching, Compiling, and Running Issues
• In PSCAD X4:
Page 43
PSCAD - Resolving Launching, Compiling, and Running Issues
6.7 Receiving a “Revocation information for the security certificate for this site is not available” Error
Message
Problem
When PSCAD is run, the following error message is displayed:
When View Certificate in the above menu is selected, a message similar to the following displays:
Cause 1
This message displays when PSCAD attempts to obtain information from our server to display in the Start Page, and may be
caused by an incorrect setting in your Windows Internet Properties.
Solution 1
Change your LAN settings to “Automatically detect settings”, as shown below:
Page 44
PSCAD - Resolving Launching, Compiling, and Running Issues
Cause 2
This error is coming from one of the MHI servers, related to displaying the PSCAD Start Page.
This error is unrelated to the performance of PSCAD, and can be disregarded and even disabled.
Solution 2
Report this issue to MHI at [email protected].
So that you don’t have to wait until we fix this issue, you could simply disable the Start Page as shown below.
The error will no longer display.
Page 45
PSCAD - Resolving Launching, Compiling, and Running Issues
6.8 Receiving a “A program is trying to send an e-mail message on your behalf” Error Message
Problem
When a support question is submitted through the PSCAD application…
Cause
The Microsoft Outlook application displays the above message when it is invoked by another application, and if it detects
that a virus scanner is not installed or is not current.
Solution 1
Select the “Allow” button once the progress bar is 100% complete. The e-mail will be sent.
Solution 2
Install or update your virus scanner in order to prevent this message from displaying again.
Page 46
PSCAD - Resolving Launching, Compiling, and Running Issues
Problem
A PSCAD pane is not displayed in the PSCAD application, even though it is selected for display in the View | Panes drop-
down menu:
Solution 1
The pane is docked, or minimized as a tab along the outer edge of the application.
Display the pane by left-clicking on the tab. Left-clicking outside the application will cause this pane to revert back to the
docked position.
Page 47
PSCAD - Resolving Launching, Compiling, and Running Issues
Solution 2
The pane is hidden behind another pane. Resize the top pane to reveal the pane underneath:
Solution 3
If the above solutions do not resolve the issue, the PSCAD docked windows may be reset to the default state. This is
performed by deleting the Workspace key in the registry for the particular version of PSCAD that you are using.
Note
Windows Administrator Privileges are required for this.
• Launch the Windows Registry Editor with Windows Administrator privileges (from the Windows Start menu,
browse to and right-click on regedit.exe, and select the option to “Run as administrator” when prompted).
• In the Registry Editor, browse as follows:
HKEY_CURRENT_USER | Software | Manitoba HVDC Research Centre | PSCAD | [your
version of PSCAD] | Workspace
• Delete the entire “Workspace” folder as listed in the above path. For example, this is the path for deleting the
Workspace folder in v4.6.0:
Page 48
PSCAD - Resolving Launching, Compiling, and Running Issues
Problem
When PSCAD is run, no results are displayed in the graphs.
Cause 1
There is an issue with one or more of the components in your network.
Solution 1
Review your components.
Cause 2
Aggressive anti virus software is blocking the graphs.
This can be tested, by disabling your anti virus software, and checking whether the graphs populate.
Solution 2
If the graphs populate when the anti-virus software is disabled, then configure your anti virus software to allow the
simulation to run and populate the graphs. Information to help with this is listed in the system requirements document
posted to this article.
Page 49
PSCAD - Resolving Launching, Compiling, and Running Issues
Problem
User is unsure how to debug his Fortran files.
Solution
Refer to the PSCAD Help system, available as follows:
• Launch PSCAD.
• Display the Help system from the PSCAD Start menu:
Page 50
PSCAD - Resolving Launching, Compiling, and Running Issues
Page 51
PSCAD - Resolving Launching, Compiling, and Running Issues
Problem
The “Open Examples” menu option does not automatically display the folder containing the examples. Instead, it opens a
previously-used folder.
Cause
To be determined.
Solution
Display the available examples as shown below. The examples may then be selected and loaded into PSCAD.
Page 52
PSCAD - Resolving Launching, Compiling, and Running Issues
Problem
When trying to load a project file from a file path containing Unicode, for example Kanji characters, the project cannot
open, and returns an error similar to the following:
C:\...<SOME KANJI CHARS>\…
Cause
Unicode is not supported in PSCAD.
Solution
Ensure that PSCAD project folders and case names do not contain Unicode.
Page 53
PSCAD - Resolving Launching, Compiling, and Running Issues
Problem
In the PSCAD Appliation Options dialog, Matlab is not detected, and may therefore not be selected:
Cause (1)
Matlab is not fully installed.
The Fortran Medic utility may be used to check whether this software is fully installed. Launch the Medic utility as per
Appendix A.2, and view the Matlab installation section. If Matlab is installed, the installed version will be listed, along with
the required files, similar to the following:
Solution (1)
Fully install Matlab.
Cause (2)
An incompatible version of Matlab is installed. For example, PSCAD v4.5 and earlier are 32-bit applications, and will not
detect a 64-bit version of Matlab.
Solution (2)
Install a compatible version of Matlab.
Page 54
PSCAD - Resolving Launching, Compiling, and Running Issues
Cause (3)
The default configuration file has been changed, and is pointing to a wrong path.
Solution (3)
Reset the configuration file to the default setting:
$(HomeDir)\matlab_versions.xml)
Page 55
PSCAD - Resolving Launching, Compiling, and Running Issues
Problem
When PSCAD is launched, the following messages display:
Cause
This message is related to viewing the Start Page in the PSCAD application. It may not be viewed because the user’s
network is somehow not allowing the certificate issuer, GeoTrust SSL CA – G3.
Page 56
PSCAD - Resolving Launching, Compiling, and Running Issues
Solution (1)
Select “Yes” in the above view, and proceed to use PSCAD.
Solution (2)
If you do not want to display the Start Page at all, this may be de-activated as follows:
• Disable the Start Page from displaying:
• Modify the name of the “index.html” file, which will help to disable the Start Page. The following is an example,
showing that I added “_disable” to this filename:
C:\Program Files (x86)\PSCAD[version]\Forms\startpage\index_disable.html
Page 57
PSCAD - Resolving Launching, Compiling, and Running Issues
Problem
The tools in the menus are not active (they are greyed out). See the following image for an example:
Cause 1
PSCAD is not licensed.
Solution 1
License PSCAD.
Cause 2
PSCAD is licensed, however, a project is not loaded and selected (highlighted) in the Workspace.
For example, in the following view, the software is licensed and a project is loaded into the Workspace (“vdiv (Single Phase
Voltage Divider”). However, this project has not been selected, instead the Master Library is selected, and therefore, the
menu tools are not active.
Solution 2
Left-click on the project to select it.
Page 58
PSCAD - Resolving Launching, Compiling, and Running Issues
6.18 Receiving a “Power Systems Simulator has stopped working” Error Message
Problem 1
When using the tools in PSCAD v4.6.2 in a normal manner, the following error displays, and PSCAD crashes:
If MyUpdater is being used to launch PSCAD, a MyUpdater error displays when trying to launch PSCAD v4.6.2:
Process terminated with error: C000041D
Solution 1
• Open a web browser to here:
https://www.visualstudio.com/vs/older-downloads/
• Scroll down to the very bottom of the above webpage, select the Visual Studio C++ 2015 Redistributable
Update 3 (x64), and download and install it:
• Select the Visual Studio C++ 2015 Redistributable Update 3 (x86) edition, and download and install it:
Page 59
PSCAD - Resolving Launching, Compiling, and Running Issues
Problem 2
When trying to view the panes PSCAD v4.6.3 Update 3 in a normal manner, the following error displays, and PSCAD crashes:
Solution 2
Update PSCAD to v4.6.3 Update 5 or newer.
Page 60
PSCAD - Resolving Launching, Compiling, and Running Issues
6.19 Receiving a “Power Systems Simulator has stopped working” Error Message
Problem
When attempting to launch PSCAD, the following error displays, and PSCAD crashes:
Problem signature:
Problem Event Name: APPCRASH
Application Name: Pscad.exe
Application Version: 4.6.0.0
Application Timestamp: 557af343
Fault Module Name: KERNELBASE.dll
Fault Module Version: 6.1.7601.23714
Fault Module Timestamp: 58bf8a2e
Cause
The executable file for launching PSCAD is being blocked by protection software. For example, it may be blocked by a Data
Loss Prevention (DLP) tool.
Solution
Ensure that the protection software is configured to allow PSCAD to be launched. For example, for PSCAD v4.6, the
following files must not be blocked by the protection software:
• For PSCAD 64-bit:
C:\Program Files (x86)\PSCAD46\bin\win64\pscad.exe
• For PSCAD 32-bit:
C:\Program Files (x86)\PSCAD46\bin\win\pscad.exe
Page 61
PSCAD - Resolving Launching, Compiling, and Running Issues
Problem
When attempting to select a compiler in the PSCAD application, only the 32-bit editions of the Intel Fortran compiler are
detected. The 64-bit editions of Intel Fortran are not detected. Furthermore, the Gfortran 4.6 compiler is not detected.
System
PSCAD v4.6
Intel Fortran 15+
Cause
The fortran compiler file is outdated in the PSCAD program files. For example, the file may be from PSCAD v4.5, which did
not support 64-bit editions of the Intel Fortran compiler nor Gfortran 4.6.
Solution
Replace the existing fortran compiler file with the most recent one as follows:
• Locate the existing fortran_compilers.xml file. It will be located in a path similar to the following:
From: fortran_compilers.xml
To: fortran_compilers_1.xml
• Download and launch the latest Medic tool as per Appendix A.2.
• When the tool has finished detecting your information, scroll down to the fortran_compilers.xml file error,
which will be similar to the following, right-click on the error, and select the option to update the file:
Page 62
PSCAD - Resolving Launching, Compiling, and Running Issues
Problem
If a user does not have permissions for the LocalAppData folder, then PSCAD is unable to store its user profile settings,
other configuration files, and log files.
This folder is normally set to: C:\Users\USERID\AppData\Local
Solution (1) – For any version
Obtain permission to create and write to the above location.
Solution (2) – For PSCAD v4.6.3+
If using PSCAD v4.6.3 and later, configure PSCAD to use a folder for which you have permissions, as follows:
In a location in which you have permissions, create a new folder. For example: C:\PSCAD\LocalAppData
• Add the following path to the folder: Manitoba HVDC Research Centre\PSCAD. In the above example, the path will
become: C:\PSCAD\LocalAppData\Manitoba HVDC Research Centre\PSCAD
• Determine your preferred PSCAD launch shortcut (e.g. desktop, Windows Start menu, Task Bar).
• In the PSCAD launch shortcut, modify the command line argument to point PSCAD to the new folder. Using the
above example, this would be added as shown:
• Whenever PSCAD is launched using this shortcut, the user files will be stored in the new folder.
Note about Solution (2):
The alternative folder must be specific to a machine. You cannot use a public folder accessible to all machines as then all
users would share a common user profile settings, which could be modified by any user.
Example of a valid path:
\\SomePublicFolder\Machine1\LocalAppData [used only by Machine1]
\\SomePublicFolder\Machine2\LocalAppData [used only by Machine2]
Example of an invalid path:
\\SomePublicFolder\LocalAppData [used by all machines]
Page 63
PSCAD - Resolving Launching, Compiling, and Running Issues
6.22 Information in the PSCAD Upper and Lower Frames is not Visible
Problem
Sometimes, the information in the PSCAD upper and lower frames is not visible.
This issue can occur when a PC is disconnected and reconnected to a dock, when remote desktop is enabled then disabled,
or when a different monitor is plugged in to a PC.
Cause
The information in the upper and lower frames is actually being displayed, but it is not visible because the colour of the text
and the fill in the frames are the same.
System
This issue is known to occur with Windows 10.
This issue is known to occur when using 4k monitors.
This issue is present with PSCAD 4.6.x, as this version is built on a Microsoft framework that has known problems with
rendering application frames.
Solutions
1. Adjust your PC settings, for example, the Theme, and the Contrast.
2. Restart PSCAD
3. Update your software to PSCAD v5+ (unreleased at the time of publishing).
Page 64
PSCAD - Resolving Launching, Compiling, and Running Issues
Problem
When running a PSCAD project, the status bar is misaligned, and some data is not visible. For example:
Cause
This is a suspected bug in the Microsoft Windows 10 framework.
System
Occurs in PSCAD v4.6 on Windows 10.
Solution (1) – Update your Software
Update your software. This issue has been was fixed in PSCAD v5.0 (not released at time of publishing).
Solution (2) - Reset Docking State
The Fortran Medic can be used to reset factory default pane settings as follows:
• Run the Fortran Medic utility as per Appendix A.2.
• Scroll down to the PSCAD installation, and right click on “Dockable pane settings”.
• Select “Yes” restore factory defaults.
Page 65
PSCAD - Resolving Launching, Compiling, and Running Issues
6.24 Receiving a “The selected folder ‘*’ does not exist…” Error Message
Problem
When trying to save a project in the PSCAD application, the following error displays:
Cause
The file path contains non-English characters, which are not recognized by PSCAD.
System
Occurs in PSCAD v4.6.
Solution (1)
Update your software. This issue has been was fixed in PSCAD v5.0 (not released at the time of publish).
Solution (2)
Rename the file path to include only English characters.
Page 66
PSCAD - Resolving Launching, Compiling, and Running Issues
6.25 Receiving Error “This file appears to be from a Later Version of the Software…” When Attempting to
Open a File in PSCAD X4
Problem
When attempting to open a file in PSCAD X4, the following error message displays:
Cause
The case was created in PSCAD V5, and must first be saved into X4 format before it can be opened in X4.
Solution
Open the case in PSCAD V5, save the file in X4 version as per this article. The file should then open alright in X4.
Page 67
PSCAD - Resolving Launching, Compiling, and Running Issues
6.26 Receiving a “The workspace already contains the name space…” Error Message When Trying to
Change the Project Settings for a Multiple Simulation Set
Problem
When trying to change the Project Settings for a multiple simulation set, the following error displays:
Cause
This error displays when the EMTDC output file in the Project Settings is named the same as any loaded project
in the workspace.
Solution
Change the name of either the EMTDC output file or the project name.
Page 68
PSCAD - Resolving Launching, Compiling, and Running Issues
6.27 The Installed Version of MATLAB is Not Detected in the PSCAD Application Options, and Therefore
Cannot Be Selected
Problem
When attempting to select the installed version of MATLAB in the PSCAD Application Options, the MATLAB software is not
detected, and cannot be selected. Reinstalling the software does not resolve the matter.
This issue occurs even if:
• The MATLAB version is compatible with the PSCAD version (see Compatibility Charts here), and
Cause 1
The matlab_versions.xml configuration file that is installed alongside PSCAD contains invalid information.
Solution 1
Update your Matlab configuration file to the most recent and valid version, as per this article.
Cause 2
The Registry Key for Matlab is invalid. The Registry Key is normally found here:
\HKEY_LOCAL_MACHINE\SOFTWARE\MathWorks\MATLAB\<VERSION>\...
Solution 2
Modify the registry key if the correct value is known.
If the correct value is not known, delete the registry key and re-install the MATLAB software. The correct value should
become added during the installation.
Page 69
PSCAD - Resolving Launching, Compiling, and Running Issues
Problem
The user creates a graph in PSCAD and saves and closes the project. When the projet is reopened, the graph has shrunk.
Cause
This can occur in older projects that are missing an attribute in the graph frame code.
Solution
Delete the graph frame and create a new graph frame.
The graph should no longer shrink.
Page 70
PSCAD - Resolving Launching, Compiling, and Running Issues
Problem
Some PSCAD parameter windows are blank. For example:
Cause
Anti-virus software is blocking the display of the windows.
Solution
Ask your IT Department to add an exception for PSCAD and its installation folders.
Page 71
PSCAD - Resolving Launching, Compiling, and Running Issues
6.30 Receiving the Error "System.Xml: Managed Code Exception" When Trying to Load a PSCAD Project
Problem
When attempting to load a project into PSCAD, the load fails, and errors similar to the following are displayed:
Cause
The errors are most likely the result of corruption in the file that is being loaded.
The corruption appears to be an invalid character in the file.
This has been known to occur on some older version of PSCAD if certain Unicode characters are entered into a text
parameter.
Solution
Manually remove the offending character from the file in either of the following manners:
• Send your file in to our support desk, along with screenshots of your errors, and we will fix the file for you, or
• Send in the following information, and we will attempt to advise what you should delete, as follows:
Load the file into a text editor.
Navigate to the Line | Column as specified in the error.
For example, in the above image, the error occurred at Line 22 | Column 257.
Note
The invalid character is likely a 'Whitespace" character, meaning that you will not be able to actually see it.
Page 72
PSCAD - Resolving Launching, Compiling, and Running Issues
6.31 “Request Support” Feature – The Auto-Attach Build Messages Incorrectly Forwards a Text File with
Strings of Numbers
Problem
When using the PSCAD Request Support feature to auto-attach project Build Messages, the file that is sent in to the MHI
Support Desk is a text file with strings of numbers, instead of the actual build messages from running a project.
The following image shows an example of the generated file with strings of numbers:
Cause
The wrong string function is being called in the PSCAD Support Request feature.
System
This occurs in PSCAD v5.0.0 and v5.0.1 Initial to Update 3.
This issue has been fixed in PSCAD v5.0.2.
Solution 1 – Update your Software
Update your software to v5.0.2+.
Solution 2 – Do not use the PSCAD Support Request Feature
Send in your support request to the MHI Support Desk using a different method.
For example, you can manually attach your build messages to an email to [email protected].
Page 73
PSCAD - Resolving Launching, Compiling, and Running Issues
Problem
When compiling a case, an error message similar to the following displays:
Cause 1
Simulations are crashed in the background.
Solution 1
Reboot your computer.
Cause 2
A program called “Cygwin” is installed on this machine, and the Gfortran compiler is being used to run the case. Cygwin has
a GNU compiler that interferes with Gfortran, which comes bundled with PSCAD.
Solution 2
• Uninstall Cygwin from this machine, or.
• Retain Cygwin, but remove Cygwin variables from your Windows PATH or LIB default environment variables:
Cygwin will still be able to be used with other applications, however, the user must always first invoke Cygwin,
before use, or
• Run PSCAD on a machine on which Cygwin is not installed, or
• Switch from using Gfortran to the Intel Fortran compiler, to compile PSCAD projects. Further information is
available here.
Page 74
PSCAD - Resolving Launching, Compiling, and Running Issues
Cause 3
If the Build messages are similar to the following…
Fatal Error: Can’t open module file ‘xxx.xx’ for
reading at (1): No such file or directory
gfortran.exe: Internal error: Aborted (program f951)
make: ***[user_source_1.o] Error 1
Unable to generate a simulation executable for
namespace ‘xxx”
Make failed to generate a simulation executable for
namespace ‘xxx. Binary file was not found.
…then the error is due to a to a conflict in Fortran compilers. This project is compiling with Gfortran, but it is calling an
object or library that was precompiled with Intel Fortran.
Solution 3
The following are possible solutions:
• Install Intel Fortran Compiler, and compile the project using Intel instead of Gfortran.
• Obtain the original source code for this item, then recompile the item using Gfortran.
• Request that the developer of this item re-compile it using Gfortran.
Cause 4
If the Build messages are similar to the following...
Creating EMTDC executable...
‘\\<some network drive path>.gf42
CMD.EXE was started with the above path as the current directory.
UNC paths are not supported. Defaulting to Windows directory.
C:\Windows>call
C:\Program~2\Gfortran…gf42vars.bat
Make: <name.mak>: No such file or directory
Page 75
PSCAD - Resolving Launching, Compiling, and Running Issues
Problem
When compiling a case, an error similar to the following is displayed in the Build Messages:
*.obj : error LNK20## : unresolved external symbol _xxx
Cause 1
The case is dependent on an object or library that has not yet been declared in PSCAD.
Solution 1
• Ensure that the object or library is saved to your machine.
• Link the object or library in PSCAD. Instructions may be found on the following webpage:
https://hvdc.ca/knowledge-base/read,article/478/linking-objects-and-libraries-into-pscad/v:
Cause 2
PSCAD is not configured to run with the selected compiler; there is an issue with the environment variables.
Solution 2
Run the Fortran Medic utility and send in the generated log file (see Appendix A.3).
Page 76
PSCAD - Resolving Launching, Compiling, and Running Issues
Problem
When compiling a case, the Fortran compiler that was specified within PSCAD is not detected, and the following error
message is displayed:
Cause 1
User updated PSCAD from v4.6 (or older) to V5.0.0, opened V5.0.0, and tried to run a project without first selecting one of
the supported compilers (Gfortran v4.6 or v8.1).
Solution 1
• Ensure one of the Gfortran compilers supported by PSCAD V5 is installed (v4.6 or v8.1). To check this, if
installed, the compiler/s will be listed in Windows Control Panel | Apps & Features. If not installed, please
refer to this article for downloads and setup instructions.
• Select the compiler in PSCAD V5 and test your setup as per this article.
Cause 2
The Fortran compiler has not yet been installed.
Solution 2
Install the Fortran compiler, then log out and log back in on your computer to apply the changes.
Cause 3
There is an issue with the Windows operating system.
Solution 3
Defragment the Windows operating system.
Cause 4
PSCAD and Gfortran compiler v4.2.1 were initially both installed, and were used to run a project. However, the Gfortran
compiler v4.2.1 was later uninstalled.
Page 77
PSCAD - Resolving Launching, Compiling, and Running Issues
Solution 4
There are two options:
• Re-install the Gfortran compiler v4.2.1, then log out and log back in on your computer to apply the changes. See
the instructions posted here:
https://hvdc.ca/knowledge-base/read,article/357/installing-the-gfortran-compiler/v:
• Or, install Gfortran compiler v4.6.2, log out and log back in on your computer, and select Gfortran 4.6 in the
PSCAD application per this article.
Note
This option is applicable if you are running PSCAD v4.6.0 or later, or the Free Edition.
Page 78
PSCAD - Resolving Launching, Compiling, and Running Issues
Problem 1
When compiling a PSCAD case, the following error message displays:
Note
See Section 8.9 for similar issue.
Cause 1-1
This error can occur if a previous simulation is still active in the background, either because it crashed or stopped
responding for some reason.
Solution 1-1
To resolve this, reboot your computer to force it to shut down the previous simulation. Then retry the simulation.
In the future, you can check the processes running on your computer and manually end the process if required. For
example if your project is called HVSIM and it crashes, you may be left with an orphaned simulation called HVSIM.exe that
is still running.
Cause 1-2
PSCAD was able to compile the emtdc executable, but was not able to launch/start it due to: Error #5
Solution 1-2
Error 5 = Access denied. The following are possible reasons for why access is denied:
• Firewall or anti-virus is preventing the launching of the EMTDC executable. Refer to Appendix F for information
on the creation of executable files during a simulation.
• Machine is otherwise locked down by IT. Refer to Appendix F for information on the creation of executable files
during a simulation.
• User did not launch PSCAD with elevated privileges on a Windows Vista/7 operating system.
Page 79
PSCAD - Resolving Launching, Compiling, and Running Issues
Problem 2
When running a PSCAD case, the following messages display:
Cause 2-1
This issue may be due to the anti-virus or a firewall being set too aggressively. Refer to Appendix F for information on the
creation of executable files during a simulation.
Solution 2-1
Turn off all anti-virus programs and firewalls and see if that fixes your problem. If it fixes the problem, then PSCAD should
be put on the anti-virus and/or firewall exceptions list.
Other Causes / Solutions
Refer to the following sections for further possible causes / solutions: Sections 7.79, 7.114, 8.16.
Page 80
PSCAD - Resolving Launching, Compiling, and Running Issues
Problem
When compiling a case, compiling messages similar to the following display:
Will execute: call C:\Program Files\Gfortran\4.2.1\bin\gf42vars.bat
Will execute: make -f vdiv.mak
Will execute: C:\Users\Public\DOCUME~1\PSCAD44\examples\tutorial\VDIV~1.GF4\vdiv.bat
Creating EMTDC executable...
C:\Users\Public\Documents\PSCAD44\examples\tutorial\vdiv.gf42>call
C:\PROGRA~1\Gfortran\428484~1.1\bin\gf42vars.bat
cygwin warning:
MS-DOS style path detected: C:\Users\ YOUR-USER-ID\AppData\Local\Temp\make44882.sh
Preferred POSIX equivalent is: /cygdrive/c/Users/ YOUR-USER-ID/AppData/Local/Temp/make44882.sh
CYGWIN environment variable option "nodosfilewarning" turns off this warning.
Consult the user's guide for more details about POSIX paths:
http://cygwin.com/cygwin-ug-net/using.html#using-pathnames
Compiling Main.f
cygwin warning:
MS-DOS style path detected: C:\Users\ YOUR-USER-ID\AppData\Local\Temp\make44883.sh
Preferred POSIX equivalent is: /cygdrive/c/Users/ YOUR-USER-ID/AppData/Local/Temp/make44883.sh
CYGWIN environment variable option "nodosfilewarning" turns off this warning.
Consult the user's guide for more details about POSIX paths:
http://cygwin.com/cygwin-ug-net/using.html#using-pathnames
gfortran.exe: Main.f: Invalid argument
gfortran.exe: no input files
make: *** [Main.o] Error 1
Cause
Another product (compiler) is interfering with the operation of Gfortran. The following compilers have been seen to
interfere with Gfortran:
• Cygwin
• QNX
Solution (1)
See solutions in Section 7.1, or see below…
Page 81
PSCAD - Resolving Launching, Compiling, and Running Issues
Solution (3)
If the offending compiler is required on this computer, locate and change the compiler program folder names temporarily,
for using PSCAD, then switch the names back to use the offending compiler.
For example, the following changes were made on one customer’s machine to retain the QNX software on the machine, but
prevent it from interfering when running a PSCAD case:
• Open a Windows browser to here: C:\Program Files (x86)\QNX Software Systems\
• Change the name of this folder:
From: QNX Software Systems
To: QNX Software Systems_1
• Open a Windows browser to here: C:\QNX650
• Change the name of this folder:
From: QNX650
To: QNX650_1
• PSCAD cases may be run using Gfortran. When done, change the folder names back to their original name in
order to use the QNX software.
To automate the changes in the above example, the script below may be used. Save the text below in a Microsoft text file,
change the file name extension from .txt to .bat, and run the .bat file. This script toggles both filenames, to include or to
remove the _1:
IF EXIST "C:\Program Files (x86)\QNX Software Systems" (
ren "C:\Program Files (x86)\QNX Software Systems" "C:\Program Files (x86)\QNX Software Systems_1"
) ELSE (
IF EXIST "C:\Program Files (x86)\QNX Software Systems_1" (
ren "C:\Program Files (x86)\QNX Software Systems_1" "C:\Program Files (x86)\QNX Software Systems"))
IF EXIST "C:\QNX650" (
ren "C:\QNX650" "C:\QNX650_1"
) ELSE (
IF EXIST "C:\QNX650_1" (
ren "C:\QNX65_1" "C:\QNX650"))
Note
The above script must always be run with Windows administrator privileges, since this is required for modifying
the name of one of the folders (C:\Program Files (x86)). To do so, ensure you have privileges on your machine,
right-click on the .bat file, select “Run as administrator”, and select “Yes” when prompted by Windows whether
to allow it.
Page 82
PSCAD - Resolving Launching, Compiling, and Running Issues
7.6 Receiving an “Error 1 – Multiple definition of ‘…’ - first defined here” Error
Problem
When compiling a case, the following compiling error is displayed:
Linking object code and libraries into binary ‘*.exe’
Main.o:Main.f:(.text+0x0): multiple definition of ‘dsdyn_’
DS.o:DS.f:(.text+0x0): first defined here
Main.o:Main.f:(.text+4ec): multiple definition of ‘dsout_’
DS.o:DS.f:(.text+0x803): first defined here
Main.o:Main.f:(.text+0x743): multiple definition of ‘dsdyn_begin_’
DS.o:DS.f:(.text+0xac5): first defined here
Main.o:Main.f:(.text+9bc): multiple definition of ‘dsout_begin_’
DS.o:DS.f:(.text+0xc02): first defined here
collect2: Id returned exit status
make: ***[config1.exe] Error 1
Cause
The case had a definition titled ‘DS’, which is a reserved name that cannot be used in PSCAD.
Solution
Rename the definition DS and then recompile the case.
Page 83
PSCAD - Resolving Launching, Compiling, and Running Issues
Problem
When compiling a case, the following compiling error is displayed:
Error: Syntax error in argument list at (1)
Make: ***[Main.o] Error 1
Unable to execute make.
Cause
The PSCAD 4.3.0 master library had a bug in the synchronous machine model.
Solution
Upgrade to the latest release of PSCAD X4.
Page 84
PSCAD - Resolving Launching, Compiling, and Running Issues
7.8 Receiving a “Make failed to generate a simulation executable for namespace ‘***’. Binary file was not
found” Error
Problem 1
When compiling a case, a compiling error similar to the following is displayed:
Creating EMTDC executable…
C:\Users\Public\Documents\Pscad4.5\Examples\tutorial\vdiv.gf42> call
C:\PROGRA~2\GFortran\420484~1.1\bin\gf42vars.bat
‘make’ is not recognized as an internal or external command,
operable program or batch file
Unable to generate a simulation executable for namespace ‘***’
Make failed to generate a simulation executable for namespace ‘***’. Binary file was not found.
Cause 1
This error occurs if GFortran 4.2.1 (and possibly GFortran 4.6) is installed, but the user did not log out of Windows then log
back in after installing GFortran.
Solution 1
After installing GFortran 4.2.1 or GFortran 4.6, the user should log out of Windows, then log back in.
Cause 2
Some problems with your settings.
Solution 2
• Ensure that the “Compiler…Environment Variables” in the “Workspace Settings” are set to “Private to process only”:
PSCAD X4:
PSCAD V5:
Continued…
Page 85
PSCAD - Resolving Launching, Compiling, and Running Issues
• Turn off your anti-virus or firewall, as it may be preventing the compiler from being launched.
• Test your setup by trying to run a simple PSCAD example:
C:\Users\Public\Documents\PSCAD\...\Examples\tutorial\vdiv.pscx
• If the example does not run, send your Build Messages and your Fortran Medic log file (Appendix A.3) to our Support
Desk ([email protected]).
Cause 3
GFortran is missing an environment variable, so PSCAD cannot use this compiler.
This can be verified using the Fortran Medic utility:
• Download and run the most recent version of the Fortran Medic utility as per Appendix A.2.
• When the utility has finished displaying the results, review the messages in Item 54 of Appendix A.5.
• If the variable is “not found”, then proceed to Solution 3, below.
Solution 3
If the variable is missing, please uninstall then re-install GFortran as follows:
• Uninstall GFortran from the Windows Apps & features tool.
• Download and install GFortran as per this article.
Note
When re-installing GFortran, ensuring to launch the installation by first right-clicking on the file, then selecting
“Run as administrator”.
Problem 2
The following compiling error is displayed:
make: [<file.exe>] Error 1
Unable to generate a simulation executable for namespace ‘<file>’
Make failed to generate a simulation executable for namespace ‘<file>’. Binary file was not found.
Cause 1
The case calls an object and library; these are not properly linked.
Solution 1
Link the object and library properly in PSCAD as follows:
• Ensure the case files are all saved in the same folder (not required at v4.6.0 and later).
• Ensure the library is loaded ahead of the case. For example, “Library_1” is loaded ahead of (or, above)
“Project_1”:
Continued…
• Open the PROJECT’S Project Settings, delete any links in the field, then select “OK”:
Page 86
PSCAD - Resolving Launching, Compiling, and Running Issues
• Open the LIBRARY’S Project Settings, and delete any links in the field:
• Still in the library project settings, select the “Browse” button, browse to and select the object, then select
“Open”.
Continued…
Page 87
PSCAD - Resolving Launching, Compiling, and Running Issues
Problem 3
When compiling a case, the following compiling error is displayed:
Will execute: call C:\Program Files (x86)\Gfortran\4.2.1\bin\gf42vars.bat
Will execute: make –*.mak
Will execute: C:\...*.BAT
Creating EMTDC executable…
C:\Users….*BAT’ is not recognized as an internal or external command
Operable program or batch file.
Unable to generate a simulation executable for namespace ‘*’
Make failed to generate a simulation executable for namespace ‘*’. Binary file was not found.
Cause 3
This is caused by a bug in the PSCAD software in v4.5.1.
Solution 3
Update your software to v4.5.5 or later.
Page 88
PSCAD - Resolving Launching, Compiling, and Running Issues
Problem 1
When compiling a case, the following compiling error is displayed:
C:\Users\user\DOCUME~1\tmp\…\gf46\multiply.o:multiply.f:(.text+0x4): undefined reference to
`__e_s1_MOD_timezero'
collect2: ld returned 1 exit status
make: *** [fortran_test.exe] Error 1
(Where test case e_s1 is the module name and timezero is one of the variables in that module.)
Cause
This error occurs if you are using GFortran 4.6, and linking in a GFortran 4.2 library when using any of EMTDC include files.
Solution 1
Use GFortran 4.2 to compile this project.
Solution 2
Recompile the library using GFortran 4.6, and run the case with these files using GFortran 4.6.
Problem 2
The following compiling error is displayed:
C:\Users\user\DOCUME~1\tmp\hello\gf42\multiply.o:multiply.f:(.text+0x4): undefined reference to
`__e_s1__timezero'
collect2: ld returned 1 exit status
make: *** [fortran_test.exe] Error 1
(Where test case e_s1 is the module name and timezero is one of the variables in that module.)
Cause
This error occurs if you are using GFortran 4.2, and linking in a GFortran 4.6 library when using any of EMTDC include files.
Solution (a)
Use GFortran 4.6 to compile this project.
Note
GFortran 4.6 is compatible with PSCAD v4.6 and later.
Solution (b)
Recompile the library using GFortran 4.2, and run the case with these files using GFortran 4.2.
Continued…
Page 89
PSCAD - Resolving Launching, Compiling, and Running Issues
Problem 3
The following compiling error is displayed:
C:\Users\<user>\DOCUME~1\ tmp\hello\gf46\*.o:*.f:(.text+0x59): undefined reference to
`_gfortran_transfer_character_write'
collect2: ld returned 1 exit status
make: *** [fortran_test.exe] Error 1
Cause
This error occurs if you are using GFortran 4.2, trying to run a simple program with just a PRINT statement in it, and trying
to link in a GFortran 4.6 library.
Solution (a)
Use GFortran 4.6 to compile this project.
Note
GFortran 4.6 is compatible with PSCAD v4.6 and later.
Solution (b)
Recompile the library using GFortran 4.2, and run the case with these files using GFortran 4.2.
Problem 4
When trying to compile a PSCAD project using GFortran 4.2, the following messages display in the Build Pane
Generating 'C:\...\*.gf42\*.map'.
Will execute: "C:\...\*.gf42\*.mak.bat"
Creating EMTDC executable...
Linking objects and libraries into binary '*.exe'
C:\...\gf46\...undefined reference to '*'
collect2: ld returned 1 exit status
make: *** [*.exe] Error 1
Unable to generate a simulation executable for namespace '*'
Cause
Incompatible compilers: Using GFortran 4.2 to run a model that was developed using GFortran 4.6:
• GFortran 4.2 cannot run models created using GFortran 4.6.
• Likewise, GFortran 4.6 cannot run models created using GFortran 4.2.
Solution 1 – Change your Compiler
Compile the project using GFortran 4.6.
Setup and test GFortran 4.6.
Solution 2 – Change your Lib File
Obtain the lib file built with GFortran 4.2, and use GFortran 4.2 to run the project.
7.10 Receiving an "Error U1052 – file‘data.mak’ not found " Error
Problem
When compiling a case, the following compiling error is displayed:
Page 90
PSCAD - Resolving Launching, Compiling, and Running Issues
Software
PSCAD X4, Intel 11.0
Cause 1
The error occurred when using PSCAD X4 to compile a case located on a network drive (no error occurs if PSCAD 4.2.1 is
used). PSCAD X4 does not allow you to compile cases stored on a network drive.
Solution 1
• Copy the case to your local machine and then open it with PSCAD.
or
• Mapping your network drive to a local drive might resolve the issue:
Cause 2
PSCAD case names should NOT contain spaces.
Page 91
PSCAD - Resolving Launching, Compiling, and Running Issues
Solution 2
Rename the PSCAD case so that it does not contain spaces.
Cause 3
User was trying to re-compile a previously compiled case, but was not running with elevated privileges.
Solution 3
Launch PSCAD with elevated privileges, then re-compile the case.
Page 92
PSCAD - Resolving Launching, Compiling, and Running Issues
Problem
When compiling a case, a Build Message error similar to the following is displayed:
Make File Error: The local project requires a link library:
‘C:\….o’
that does not exist with the given path.
Cause
An object is not properly linked to the project in the PSCAD settings.
Solution
Link the object in the PSCAD settings as per the section “Link” in the On-Line Help System:
Page 93
PSCAD - Resolving Launching, Compiling, and Running Issues
Problem
When compiling a case, the following message is displayed in the Build Messages:
Unable to solve line constants, check the log file for details.
Cause
There is a flaw with the design of the circuit. For example, improper grounding, or a component open at one side.
Solution
Review the case and build messages and all T-Line messages.
For more information on reviewing the messages, refer to the PSCAD On-Line Help System:
For further analysis of a particular error message, please forward a request for assistance to our Support Desk
([email protected]), along with all Build messages, and a snampshot of any error dialog boxes.
Note
Certain restrictions apply related to PSCAD license and version.
Page 94
PSCAD - Resolving Launching, Compiling, and Running Issues
7.13 Receiving a “‘cl.exe’ is not recognized as an internal or external command” build message
Problem
When compiling a case, the following build messages display:
Compiling “<file path and object name>.c”.
cl.exe … “<object name>.c”
‘cl.exe’ is not recognized as an internal or external command.
Cause
The case is trying to build an object containing c-code, but a c-compiler is not installed. “<object>.c” is the object
prorgrammed in c-code; “cl.exe” is the c-compiler.
Solution
Install a c-compiler (the full version of Microsoft Visual Studio) along with Intel Fortran composer. The Visual Studio that
comes bundled for free with Intel Fortran (i.e. premier partner) is not a c-compiler; the full version is commercially
available.
Once Intel Fortran and the c-compiler have been installed, test your setup by trying to run one of the c-code examples
located at the following path:
C:\Users\Public\Documents\Pscad<version>\examples\CInterface
Page 95
PSCAD - Resolving Launching, Compiling, and Running Issues
7.14 Receiving a “The number of projects included in the simulation list must not exceed %d” Error
Message
Problem
When compiling a case, the following error message displays:
Version
This is a bug present in PSCAD v4.5.1 and earlier.
Solution
Update software to v4.5.2 or later.
Page 96
PSCAD - Resolving Launching, Compiling, and Running Issues
Problem 1
When compiling a case, the first compile passes and the case runs fine, however, some subsequent compiles fail with the
following error message:
Cause 1
Aggressive anti-virus scanner
Solution 1
Attempt to disable the virus scanner or reduce it aggressiveness, or add PSCAD to the scanner’s “exceptions” list as a
trusted program.
Cause 2
System process with PID 4 is holding onto this previously created file.
Solution 2
In the Windows Services Control Panel, re-enable Application Experience Service, which is used to process application
compatibility cache requests for applications as they are launched.
Problem 2
When compiling a case, an error message similar to the following displays:
File Write Error Unable to open file [SOME DRIVE]:\[SOME PATH]\[SOME PROJECT]
For example:
File Write Error: Unable to open file F:\........\\Project1
Cause
The user does not have Windows administrator permisssions for the specified drive.
Solution
Copy the project files to a different folder on which you have permissions (e.g. desktop, My Documents…).
Page 97
PSCAD - Resolving Launching, Compiling, and Running Issues
Problem
When compiling a case, the compile fails with the following error message:
Cause
Possible corruption of PSCAD installation due to reasons unknown.
Solution
Uninstall, then re-install, PSCAD.
Page 98
PSCAD - Resolving Launching, Compiling, and Running Issues
Problem
When compiling a case, PSCAD stops unexpectely, with no runtime error.
Running the PSCAD case manually outside of the PSCAD application results in the following error message:
Note
Running a project manually outside of PSCAD is performed by first building the case in PSCAD, closing PSCAD,
then running the .exe file from the build folder.
Cause
A required DLL file is missing.
Solution 1
Locate this DLL and place it inside the .if12 folder
Page 99
PSCAD - Resolving Launching, Compiling, and Running Issues
Solution 2
Locate this DLL and add the location to your PATH environment variable. That way the simulation will know where to look
when it tries to link in the DLL.
Page 100
PSCAD - Resolving Launching, Compiling, and Running Issues
Problem
When building a case, the following error message displays:
Coding aborted due to Error: Unable to create output directory. Error code = 112
Cause 1
User is not an administrator on the computer and has limited access.
Solution 1
Obtain administrator rights on the computer.
Cause 2
The user is an administrator, but the Network is imposing limits on folder permissions.
Solution 2
• Move case files to a folder for which user has permissions, or
• Remove limits on folder permissions (this might require the assistance of your organization’s IT staff).
Cause 3
Overly aggressive virus scanner.
Solution 3
Turn your virus scanner down or off, then retry the build.
Page 101
PSCAD - Resolving Launching, Compiling, and Running Issues
Problem
When building a case, the following message displays in the Build Messages:
Transmitter ‘*' cannot be added. The storage table is at capacity of 256.
Cause
The number of allowed radio links is exceeded.
Solution
Increase the number of allowed wireless radio links as shown:
Page 102
PSCAD - Resolving Launching, Compiling, and Running Issues
7.20 Receiving a “EMTDC: Error while opening file for read” Build Message
Problem
When compiling a case that is declaring an external file, the following build error is displayed:
EMTDC: Error while opening file for read: [some file name]
ERROR: Abnormal termination of EMTDC by OPENFILE
Non-standard Messages:
Abnormal progress termination
EMTDC Runtime Error: abnormally terminated
Simulation stopped
Cause 1
The error is related to the project being unable to open the declared file, because the declared file is residing in a folder
that is different from where the PSCAD project file resides.
Solution 1
Copy the declared file to the same folder as the project resides in.
Cause 2
The error is related to the project being unable to open the declared file, because there is an error in the declared path.
Solution 2
Correct any errors in your path.
Page 103
PSCAD - Resolving Launching, Compiling, and Running Issues
Problem
When compiling a case in the Free Edition using the GFortran compiler, the following error is displayed:
make: *** [Station.o] Error 126
Unable to generate a simulation executable for namespace ‘***’
Cause
WinAVR is installed, and is somehow interfering with compiling the PSCAD case.
Solution
• Locate the WinAVR folder (e.g. C:\WinAVR-20081205).
• Slightly change the folder name so that it won’t be able to be executed. For example:
Note
You must be an administrator on your machine to perform this step.
Change this:
C:\WinAVR-20081205
To this:
C:\WinAVR-20081205_1
• Load your project in PSCAD.
• Clean the directory (right-click on the project in the Workspace, and select “Clean Temporary Directory”):
Page 104
PSCAD - Resolving Launching, Compiling, and Running Issues
Problem
When compiling a case in PSCAD v4.5.3, the following error is displayed:
Exchange Error
Page 105
PSCAD - Resolving Launching, Compiling, and Running Issues
b. When the Workspace Settings dialog box displays, perform the following:
Select the “Runtime” page from the drop-down menu.
Select “31000” from the “Communication Port Base Value” drop-down menu:
Page 106
PSCAD - Resolving Launching, Compiling, and Running Issues
7.23 Receiving a “connect function failed with error: 10061” Error Message
Problem
When compiling a case in PSCAD v4.6.0 or later, the following error is displayed:
Cause
This is an error thrown by the EMTDC Client side of Electric Network Interface (ENI) configuration. The
description is as follows:
This error can occur if the Windows firewall or anti-virus program is too aggressive. This may be tested by
shutting off the anti-virus and firewall, then running PSCAD again.
Solution
Add PSCAD to the firewall or anti-virus whitelist of trusted programs.
Page 107
PSCAD - Resolving Launching, Compiling, and Running Issues
7.24 Receiving an “unable to execute a simulation executable for namespace ‘*’” Error Message
Problem 1
When compiling a PSCAD case using an Intel Fortran compiler, the following errors are displayed:
Cause 1
These errors are due to a compatibility issue: You are using an Intel Fortran compiler to compile a case that contains a
library that was precompiled using the Compaq Fortran compiler. It is not supported to run a case using a compiler that is
different from the precompiled library.
Solution 1
The preferred solution is to obtain an Intel Fortran-precompiled copy of this library, and set it up as follows:
• Unload the Compaq Fortran-precompiled library and the project from your PSCAD workspace.
• Load the Intel Fortran-precompiled library and the project (in that order) into your PSCAD workspace.
• Clean the project directory.
• Run your case.
The least preferred solution is to run your case using Compaq Fortran 6. There are two things to note with this solution:
• Compaq is not supported as of PSCAD v4.6 and later.
• Compaq is old technology and may be difficult to install.
Page 108
PSCAD - Resolving Launching, Compiling, and Running Issues
Problem 2
When compiling a PSCAD case using an Intel Fortran compiler, the following build message is displayed:
\S was unexpected at this time.
Unable to generate a simulation executable for namespace '*'
The Fortan Medic utility will detect this issue: refer to Appendix A.5, Item #14.
Cause 2
The PATH environment variable key for a software contains the “&” symbol. This symbol can affect PSCAD, Intel Fortran,
Microsoft Visual Studio, and other software.
Solution 2
• Uninstall the software that contains the “&” symbol in the PATH environment variable key.
• Re-install this software, but during the installation, select a folder path name that does not include the “&”
symbol. For example, replace “&” with “and”.
Page 109
PSCAD - Resolving Launching, Compiling, and Running Issues
7.25 Receiving a “Simulation set ‘*’ has xx project tasks included, exceeding the workspace selected limit of
* tasks” Error Message
Problem
When running a simulation set in PSCAD, the following error is displayed:
Cause
This error is due to exceeding the maximum number of projects that may be included in a simulation set.
Solution 1
Decrease the number of projects in your simulation set as required.
Solution 2
Update your PSCAD version and/or licensing configuration in order to accommodate the desired number of projects in your
simulation set. The following specifications may be used to determine the best setup:
https://hvdc.ca/knowledge-base/read,article/41/network-splitting-using-electric-network-interface-eni/v:
https://hvdc.ca/knowledge-base/read,article/365/computer-cpu-cores-and-instances-of-emtdcs/v:
“What’s New in PSCAD v4.6.0”: https://hvdc.ca/knowledge-base/read,article/229/what-s-new-in-pscad-at-each-
version/v:
Page 110
PSCAD - Resolving Launching, Compiling, and Running Issues
7.26 Running a PSCAD v4.6 Simulation on Windows 10 is slower than running earlier versions of PSCAD
Problem
When running simulations on Windows 10, using v4.6 is sometimes slower than using an earlier version of PSCAD. Earlier
versions of PSCAD may also be affected.
Cause
Windows 10 defender, which is the anti-virus that is released with Windows 10, can slow the simulation for v4.6.
Solution 1
Turn off the Windows 10 Defender, as follows:
• From the Start menu, open Windows Defender.
Page 111
PSCAD - Resolving Launching, Compiling, and Running Issues
Solution 2
Add the PSCAD v4.6 program folder to the Windows Defender exclusion list, as follows:
• From the Start menu, open Windows Defender.
Page 112
PSCAD - Resolving Launching, Compiling, and Running Issues
• When prompted, select a folder to exclude, and then select “Exclude this folder”.
Page 113
PSCAD - Resolving Launching, Compiling, and Running Issues
Solution 3
Add the PSCAD executable to the Windows Defender exclusion list, as follows:
• From the Windows Start menu, browse to and select the “Windows Defender Settings” link.
• Select “Add an exclusion”:
Page 114
PSCAD - Resolving Launching, Compiling, and Running Issues
7.27 Receiving an “‘nmake’ is not recognized as an internal or external command…” Error Message
Problem (1)
When compiling a PSCAD case using the Intel Fortran compiler, the following build errors display:
Cause (1)
The INTEL_LICENSE_FILE environment variable has duplicate path segments, such as:
C:\Program Files (x86)\Common Files\\Intel\Licenses;C:\Program Files (x86)\Common Files\Intel\Licenses
Which is effectively:
C:\Program Files (x86)\Common Files\\Intel\Licenses;
C:\Program Files (x86)\Common Files\Intel\Licenses
Solution (1)
The Fortran Medic may be used to replace the duplicated path segments with a single path segment, which in the above
example, would be
C:\Program Files (x86)\Common Files\Intel\Licenses
Note
Double slashes, \\, are converted to single slashes prior to any comparison for duplicates.
For more information on using the Fortran Medic for this repair, see Appendix A.5, Item 11.
Cause (2)
Previous additions to, or modifications to, the Windows PATH by installing a FORTRAN compiler have not been recognized
by Windows.
Solution (2)
Log out then log back in on your machine (preferred), or restart your machine. Then retry compiling PSCAD.
Page 115
PSCAD - Resolving Launching, Compiling, and Running Issues
Problem (2)
The following build messages display in the PSCAD application when trying to run a case using PSCAD v4.6 and Intel Fortran
v17:
Creating EMTDC executable…
C:\…
Intel® MPI Library 2017 Update 1 for Windows* Target Build Environment for Intel® 64…
Copyright (C) 2007-2016 Intel Corporation. All rights reserved.
ERROR: Visual Studio 2012, 2013 or 2015 is not found in the system.
‘nmake’ is not recognized as an internal or external command,
operable program or batch file.
Unable to generate a simulation executable for namespace ‘*’
Cause (2)
Prerequisite software was missing. Specifically, Microsoft Windows SDK for Windows 8.1 should have been installed prior to
installing the compiler.
Solution (2)
Install missing prerequisite software, then re-install Intel Fortran. Ensure to log out and log back in following these
installations, to apply the changes.
Page 116
PSCAD - Resolving Launching, Compiling, and Running Issues
7.28 Receiving a “ ‘C:Program’ is not a valid internal or external command” Error Message
Problem
When building a PSCAD case, the following build error is displayed:
When the latest Fortran Medic tool is run, an error message is displayed as indicated in Appendix A.5, Item 15.
Cause
The computer is not correctly generating short filenames.
Version
This issue affects PSCAD v4.5.3.
Solution
This issue has been resolved in PSCAD v4.5.4. Update your software to v4.5.4 or later.
Page 117
PSCAD - Resolving Launching, Compiling, and Running Issues
Problem
When building one of the Matlab examples that comes bundled with PSCAD, a build error similar to the following is
displayed:
Cause
When Matlab R2015a (64-bit) or better is installed, but not the 32-bit version, PSCAD incorrectly thinks that the 32-bit
version of Matlab was also installed. PSCAD would then incorrectly list them both as the following in the application:
R2015b, 8.5, (64-bit): This points to the 32-bit path, in which a 32-bit version SHOULD be installed, but is not
installed.
R2015b, 8.5, (64-bit): This correctly points to the 64-bit path to where the 64-bit version IS installed.
The issue arises because regardless which installation is selected in the PSCAD application, the code will always select the
first match from the list, which points to the non-existent 32-bit installation, and the error will display when the case is run.
Version
PSCAD v4.6.0 (64-bit), together with Matlab R2015a or better
Solution
• Use PSCAD v4.6.1 or later (this issue is corrected in these versions), or
• Use PSCAD v4.6.0 or earlier, and install both the 32-bit and the 64-bit versions of Matlab R2015a or better
(unverified)
Page 118
PSCAD - Resolving Launching, Compiling, and Running Issues
7.30 Receiving a “The simulation process has stopped unexpectedly” error message when running any
simulation
Problem
The following build error message is displayed when running any simulation:
Page 119
PSCAD - Resolving Launching, Compiling, and Running Issues
Furthermore, the build messages indicate that Visual Studio 2015 version 221 is being used in the simulation:
Intel(R) Parallel Studio XE 2015 Update 4 Composer Edition (package 221)
Microsoft (R) Program maintenance utility version 14.00.23506.0
Copyright (C) Microsoft Corporation. All rights reserved.
Also refer to Appendix A, Item 16 to see the message that is displayed in the Fortran Medic utility.
Cause
This error is due to incompatible software:
If Intel Fortran Composer 15.0.221 and earlier detects that Microsoft Visual Studio 2015 (v14) is installed, it will try to use
that software for the compilation. However, these software programs do not appear to be compatible, and as a result, the
simulation terminates.
Note
Intel Fortran Composer 15.0.208 and later is compatible with Microsoft Visual Studio 2015, according to Intel
Fortran Release Notes.
Solution
Two options are as follows:
• To retain Microsoft Visual Studio 2015 (v14), switch out your compiler:
Install a compatible version of Intel Fortran (e.g. v15.0.208 or later, or v16). During this installation ensure to
integrate Intel Fortran with the installed VS2015.
Log out then log back in on your computer to apply the changes.
Launch PSCAD, and set PSCAD to use the newly installed compiler (in the Application Options dialog box, display
the Dependencies page, and select the compiler from the “Fortran Compiler…Version” field).
• Or, to retain Intel Fortran 15.0.221 or earlier, switch out your Visual Studio version:
Uninstall Visual Studio 2015 (or at least rename the VS installation folder in C:\Programs Files (x86) so that Intel
Fortran does not try to use it).
Install a compatible version of Visual Studio software (VS2010, VS2012 or VS2013).
Re-install Intel Fortran 15.0.221 or earlier, ensuring to integrate it with the new
VS software.
Log out then log back in on your computer to apply the changes.
Launch PSCAD, and ensure PSCAD is still set to use this compiler (in the Application Options dialog box, display the
Dependencies page, and select the compiler from the “Fortran Compiler…Version” field).
Page 120
PSCAD - Resolving Launching, Compiling, and Running Issues
7.31 Receiving a “No rule to make target ‘*.mak’. Stop” error message
Problem
The following build error messages are displayed when running any PSCAD case:
C:\Windows>call “*.bat”
CMD does not support UNC paths as current directories.
make: *.mak: No such file or directory
make: *** No rule to make target ‘*.mak’. Stop.
Unable to generate a simulation executable for namespace '*'
Or, the following, slightly different build error messages are displayed when running any PSCAD case:
Cause
The project files are being run either from “D” drive, or from a network folder:
Solution
Move your project files into a local folder on your C drive (e.g. on your desktop), load the project from your local folder, and
run the simulation.
Page 121
PSCAD - Resolving Launching, Compiling, and Running Issues
Problem
A build error message similar to the following is displayed when running any PSCAD case:
CALL E_VARIMPBRANCH( (IBRCH(3)+10),SS(3),1,0,0,R_B,,0.0)
1
Error: Syntax error in argument at (1)
Cause
PSCAD is incorrectly generating Fortran code with missing parameters when using GFortran 4.2.1 or 4.6.2, which generates
compile errors.
Solution
• Update to PSCAD 4.6.1 or better (if available), or
Page 122
PSCAD - Resolving Launching, Compiling, and Running Issues
Problem
When running a PSCAD case, the following error displays in the build messages:
forrtl: severe (41): insufficient virtual memory
Cause 1
Your project is exceeding the 2 GB memory limit imposed on any 32-bit application. Your simulation is demanding more
memory than is physically possible to adress. For more information on this, please refer to the following link:
https://en.wikipedia.org/wiki/2_GB_limit
Solution 1
• Decrease the size of your network, or
• Switch to a PSCAD 64-bit application, available as of PSCAD v4.5. To pdate your software, please forward your
license number to our Sales Desk ([email protected]).
Note
See Cause 2 to ensure that you have enough RAM for your 64-bit application.
Cause 2
Your computer does not have enough RAM installed.
Solution 2
• Use PSCAD v4.5.3 or later, as this is optimised to handle large cases, and/or
• Ensure that you have adequate RAM for your case size. As a general rule, the Windows operating system requires
2 GB, the PSCAD GUI requires 2 GB to load the case and set up the memory for the simulation run, and the
simulation and frequency scan require 2 to 4 GB of RAM, depending on network size.
Page 123
PSCAD - Resolving Launching, Compiling, and Running Issues
7.34 Compiling any Case Containing Many Transmission Lines is Extremely Slow
Problem
Compiling cases containing many transmission lines is extremely slow to solve.
Version
Applicable to PSCAD v4.5.0 and later.
Cause
The anti-virus, Webroot, is installed on your machine, and is causing extremely slow simulations. This software forces
PSCAD to solve all lines in series rather than in parallel, and it starts solving them quite slowly.
This issue is applicable to PSCAD v4.5.0 and later, as the feature of solving transmission segments in parallel was first
introduced at v4.5.0.
To determine whether you have Webroot installed on your machine, look for the icon in the notifications taskbar, similar to
the following:
Solution
Webroot should be removed from your computer if you are running PSCAD cases with many transmission lines.
Page 124
PSCAD - Resolving Launching, Compiling, and Running Issues
Problem
Compiling a PSCAD case results in the following error in the Build Messages:
gcc.exe: C:PROGRA…lib: No such file or directory
Possible Cause
An installed compiler, OpenModelica, might be conflicting with the GFortran compiler that is used with PSCAD.
Solutions (unverified)
Try one of the following:
Note
You will require admin privileges on your machine to perform the first two options.
From: C:\OpenModelica1.9.1
To: C:\OpenModelica1.9.1_
• Uninstall openModelica.
Page 125
PSCAD - Resolving Launching, Compiling, and Running Issues
7.36 Receiving “unresolved external symbol _sprintf” and “return code '0x460'” Error Messages
Problem
Error messages similar to the following display when compiling PSCAD cases with Intel Fortran:
Note
Refer to Section 7.43 for another issue related to Visual Studio 2015.
Cause
Due to some changes made to the libraries as of Microsoft® Visual Studio 2015, PSCAD must be configured to use one of
two program files:
• The default configuration upon PSCAD installation is for using earlier versions of Visual Studio (VS2010, VS2012,
VS2013).
• The new configuration is for using newer versions of Visual Studio (VS2015+).
If PSCAD is configured to use the wrong VS version, the build will fail, and errors similar to the above will display. The
solution, listed below, contains instructions for configuring PSCAD to use the appropriate program file.
Note
The program file for VS2015+ is only supported in PSCAD v4.6.1 and newer; older versions of PSCAD do not
support using Visual Studio 2015+.
Note
Objects or libraries containing c-code precompiled with the earlier versions of Visual Studio (VS2010, VS2012,
VS2013) are not compatible with VS2015. The reverse is true as well (objects or libraries containing c-code
precompiled with VS2015 are not compatible with VS2010, VS2012, VS2013).
For any objects of libraries not containing c-code, they may be precompiled and then run using any of the above
versions. There should not be any compatibility issue.
Page 126
PSCAD - Resolving Launching, Compiling, and Running Issues
Applicability
The solution to this issue is applicable to the following:
• PSCAD v4.6.1 and newer;
• A supported version of Intel Fortran compiler (as specified in the Compatibility Charts, Chart 4);
• Visual Studio 2015 and newer
Continued…
Page 127
PSCAD - Resolving Launching, Compiling, and Running Issues
Solution (1.a) – Configuring for Visual Studio 2015+ on a Windows 64-bit Machine (Automatically using the Fortran
Medic)
Use the Fortran Medic tool to automatically configure PSCAD to use Visual Studio 2015+ on a Windows 64-bit machine as
follows:
Warning
If you proceed with this action, PSCAD will no longer be configured for using Visual Studio 2010, 2012, or 2013.
Notes
1. Windows Administrator privileges are required.
2. Applicable for v4.6.1 and newer (not supported with earlier versions).
• Run the Fortran Medic tool as per Appendix A.2, and scroll down to the following heading:
• Under the PSCAD 64-bit installation (v4.6.1 or newer), right-click on the message as shown below, and select the
option to update PSCAD to use VS 2015+:
• Your environment will be configured to use Visual Studio 2015 and newer.
Notes
1. This function does not install Visual Studio software, it merely configures PSCAD to be able to use
Visual Studio 2015 and newer.
2. Configuration with Visual Studio 2010, 2012, and 2013 may be restored as per Appendix A.6,
Function 2.
Continued…
Page 128
PSCAD - Resolving Launching, Compiling, and Running Issues
Solution (1.b) – Configuring for Visual Studio 2015+ on a Windows 32-bit Machine (Automatically using the Fortran
Medic)
Use the Fortran Medic tool to automatically configure PSCAD to use Visual Studio 2015+ on a Windows 32-bit machine as
follows:
Warning
If you proceed with this action, PSCAD will no longer be configured for using Visual Studio 2010, 2012 or 2013.
Notes
1. Windows Administrator privileges are required.
2. Applicable for v4.6.1 and newer (not supported with earlier versions).
• Run the Fortran Medic tool as per Appendix A.2, and scroll down to the following heading:
• Under the PSCAD 32-bit installation (v4.6.1 or newer), right-click on the message shown below:
• The following dialog will display. Select “Visual Studio 2015 and better”, then “OK” to proceed:
• Your environment will be configured to use Visual Studio 2015 and newer.
Notes
1. This function does not install Visual Studio software, it merely configures PSCAD to be able to use
Visual Studio 2015 and newer.
2. Configuration with Visual Studio 2010, 2012, and 2013 may be restored as per Appendix A.6,
Function 2.
Continued…
Page 129
PSCAD - Resolving Launching, Compiling, and Running Issues
Solution (2) – Configuring for Visual Studio 2015+ on a Windows 32-bit or 64-bit Machine (Manually)
Manually change your environment in order to use Visual Studio 2015 and newer.
Warning
This is not the recommended solution, as errors could be introduced to your environment when performed
manually. We recommend following the automatic instructions in Solution (1.a or 1.b) above, instead.
Notes
1. Windows Administrator privileges are required
2. Applicable for v4.6.1 and newer (not supported with earlier versions)
3. Ensure to perform all changes, for both the main.obj and emtdc.lib files, and for both the 32-bit and 64-bit
applications.
• If using v4.6.1, locate the following two files, and rename the “main.obj” file as “main_vs2010.obj”:
• If using v4.6.2 or newer, locate the following three files, and delete the “main.obj” file:
• Then, for v4.6.1 and newer, make a copy of the “main_vs2015.obj” file, and rename it as “main.obj”:
• Make the same changes to the main object files as performed in Step 1 above.
Continued…
Page 130
PSCAD - Resolving Launching, Compiling, and Running Issues
• If using v4.6.1, locate the following two files, and rename the “emtdc.lib” file as “emtdc_vs2010.lib”:
• If using v4.6.2 and newer, locate the following three files, and delete the “emtdc.lib” file:
• Then, for v4.6.1 and newer, make a copy of the “emtdc_vs2015.lib” file, and rename it as “emtdc.lib”:
• Make the same changes to the emtdc library files as performed in Step 3 above.
Notes
1. This function does not install Visual Studio software, it merely configures PSCAD to be able to use
Visual Studio 2015 and newer.
2. Configuration with Visual Studio 2010, 2012, and 2013 may be restored as per Appendix A.6,
Function 2.
Solution (3) – Configuring for Visual Studio 2013 and older (Automatically using the Fortran Medic)
Refer to Appendix A.6 Item 2.
Page 131
PSCAD - Resolving Launching, Compiling, and Running Issues
Problem
The following dialog displays:
Runtime Error!
Program: C:\:...exe
Cause
A “Single Sign On Engine” (SSO) application is installed on this machine, and apears to halt PSCAD from building a
simulation.
It is suspected that SSO restricts which executable files are allowed to be run on a machine. If so, SSO prevents PSCAD from
running the executable files that PSCAD creates when building a case. There is no practical way to whitelist these
executables ahead of time as the executable names depend on the existing PSCAD case names and new case names as the
user creates more PSCAD cases.
To see the messages that are displayed when the Fortran Medic tool is run, refer to Appendix A.5, Item 20.
Solution (1)
A possible solution would be to confine all PSCAD cases to a particular folder, (e.g. C:\PSCAD), and to see if the SSO may be
configured to allow all executable files to run from that folder without restriction. This would be analogous to giving a
‘sandbox folder’ to PSCAD which would not be monitored by the SSO.
Solution (2)
Remove SSO from your computer.
Page 132
PSCAD - Resolving Launching, Compiling, and Running Issues
Problem
Build messages similar to the following display when trying to run a PSCAD case using GFortran or the Intel Fortran:
• If using Intel:
Creating EMTDC executable
…
Compiling Main.c
cl: Command line warning D9035 : option ‘GX’ has been deprecated and will be removed in a future release
cl : Command line warning D9036 : use ‘EHsc’ instead of ‘GX’
cl : Command line warning D9002 : ignoring option ‘/ML’
Main.f(6) : fatal error C1034: f2c.h: no include path set
NMAKE : fatal error U1077: “‘c:\program files (x86)\microsoft visual studio 9.0\vc\bin\cl.exe”’ : return code ‘0x2’
Stop.
C:\Program Files (x86)\PSCAD50\emtdc\if18\inc\nd.h(2): error #7881: This module file was generated for a different platform or by an incompatible
compiler or compiler release. It cannot be read. [NDDE]
0 normal (Icon) vdiv USE NDDE
0 normal (Icon) vdiv ----^
200008 warning (Icon) vdiv warning #6717: This name has not been given an explicit typ e. [NCALL_NO]
0 normal (Icon) vdiv ICALL_NO = NCALL_NO
0 normal (Icon) vdiv ------------------^
200008 warning (Icon) vdiv warning #6717: This name has not been given an explicit typ e. [NNODE]
0 normal (Icon) vdiv NNODE = NNODE + 2
0 normal (Icon) vdiv ------^
200008 warning (Icon) vdiv warning #6717: This name has not been given an explicit typ e. [NCSCS]
0 normal (Icon) vdiv NCSCS = NCSCS + 0
0 normal (Icon) vdiv ------^
200008 warning (Icon) vdiv warning #6717: This name has not been given an explicit typ e. [NCSCR]
0 normal (Icon) vdiv NCSCR = NCSCR + 0
0 normal (Icon) vdiv ------^
0 normal (Icon) vdiv compilation aborted for Station.f (code 1)
300006 error (Icon) vdiv NMAKE : fatal error U1077: '"C:\Program Files
(x86)\Intel\oneAPI\compiler\2021.2.0\windows\Bin\intel64_ia32\ifort.exe"' : return code '0x1'
0 normal (Icon) vdiv Stop.
324001 error (Icon) vdiv Unable to generate a simulation executable for namespace 'vdiv'
Page 133
PSCAD - Resolving Launching, Compiling, and Running Issues
400003 normal (Icon) vdiv Finished compiling with PSCAD Version 5.0.0
400001 normal (Icon) vdiv ======== Build: 18 errors, 32 warnings ========
See Appendix A.5, Item 21 to see the corresponding Fortran Medic error.
Cause
GIT and MingW are installed and are causing this error, and are causing PSCAD to fail when compiling cases with either
GFortran or the Intel Fortran compiler.
Solution (1)
Rename the GIT folder.
Solution (2)
Remove GIT and MingW.
Solution (3)
Install PSCAD and GFortran/Intel Fortran on a different machine, one that does not also have GIT and MingW.
Page 134
PSCAD - Resolving Launching, Compiling, and Running Issues
Problem
The following build messages display in the PSCAD application when compiling a PSCAD case with an Intel Fortran compiler:
Unable to generate a simulation executable for namespace ‘*’
See Appendix A.5, Item 24 to see the corresponding Fortran Medic error.
Cause
A file path contains a ‘+’, which causes the Intel Fortran compiler to halt.
Note
The & symbol is also not allowed in a file path.
Solution (1)
Run the Fortran Medic utility as per Appendix A.2, then use the utility to remove the path containing the ‘+’ as per Appendix
A.5, Item 24.
Solution (2)
Identify the software which requires the ‘+’ character, then remove the software, and re-install the software into a folder
which does not contain the ‘+’ character.
Page 135
PSCAD - Resolving Launching, Compiling, and Running Issues
Problem
When building a PSCAD case, the error below displays. Restarting PSCAD sometimes resolves this.
Background
When PSCAD (GUI) starts a simulation (project), a socket connection is created so that PSCAD and the simulation can
exchange data. PSCAD will randomly pick a communications socket on the computer to do this. When this socket is not
available, the above error will display. Three typical causes for a socket not being available are listed below.
Cause (1): A Crashed Simulation is still running in the Background
Sometimes when a simulation has crashed, the simulation will be stuck running in the background, and will still hold a
communications socket open. If PSCAD is run again, it may try to use that same socket, but it will not be available.
Solutions (1): A Crashed Simulation is still running in the Background
• Display the Task Manager and end any simulations running in the background to free up the socket; or
• Simply restart the computer to end any simulations running in the background to free up the socket.
Cause (2): Running a Second Instance of PSCAD
Two instances of PSCAD are being run, and both instances are trying to use the same communications socket.
Solution (2): Running a Second Instance of PSCAD
Change the port range for one of the PSCAD instances as shown:
Page 136
PSCAD - Resolving Launching, Compiling, and Running Issues
7.41 Receiving an “Unable to generate a simulation executable for namespace” Error Message
Problem
When building a PSCAD case, the following error displays:
Unable to generate a simulation executable for namespace ‘*’
Cause
Anti-virus software is preventing PSCAD from compiling the case.
Specifically, Avast has been known to do this.
Solution
White-list PSCAD activities in Avast.
Page 137
PSCAD - Resolving Launching, Compiling, and Running Issues
Problem
When building a PSCAD case, the following error displays in the Build messages:
LIBCMTD.lib (*.obj) : error LNK2005: * already defined in LIBCMT.lib (*.obj)
Unable to generate a simulation executable for namespace ‘*’
Make failed to generate a simulation executable for ‘*’. Binary file was not found.
Cause
The EMTDC configuration file is outdated.
Applicability
PSCAD v4.5.1 to v4.5.3
Intel Fortran Composer XE 2013
Solution
• Update your software ([email protected]), or
• Retain your software version, but update your EMTDC configuration file:
Request the file from PSCAD Support ([email protected]). Ensure to include your PSCAD version and license number.
Delete the existing “emtdc.cfg” file from this path:
C:\Program Files (x86)\PSCAD453\emtdc\if12\windows\
Save the file you receive from Support to the above path.
Page 138
PSCAD - Resolving Launching, Compiling, and Running Issues
Problem
When using VS2015 or newer to compile a model that (1) contains non-Fortran code (e.g. c-code) and (2) was precompiled
using VS2013 or older, then some errors similar to the following may display in the Build messages:
error LNK2001: unresolved external symbol ___iob_func
error LNK2001: unresolved external symbol _printf
Or
When using VS2013 or older to compile a model that (1) contains non-Fortran code (e.g. c-code) and (2) was precompiled
using VS2015 or newer, then some errors similar to the following may display in the Build messages:
Notes
1. “Pre-VS2015” refers Microsoft® Visual Studio 2013 and earlier, for example VS2010, VS2012 and VS2013.
4. Refer to Section 7.36 for another issue related to Visual Studio 2015.
Cause
Due to changes made to the Microsoft® Visual Studio library at VS2015, any PSCAD libraries created with an earlier version
of Visual Studio (VS2013 and earlier) containing c-code are not compatible with VS2015 (and later).
Conversely, any PSCAD libraries created with VS2015 (and later) containing c-code are not compatible with VS2013 and
earlier.
Note
For PSCAD libraries that do not contain c-code, this compatibility issue is not present.
Solution
Switch to a compatible version of Visual Studio. See Section 7.36 for PSCAD configuration.
Page 139
PSCAD - Resolving Launching, Compiling, and Running Issues
7.44 Receiving a “The simulation process has stopped unexpectedly” Message when Windows goes to
Sleep while a case is Running
Problem
The following error is displayed when Windows goes to sleep (or hibernating) while a case is running:
Cause
Windows is likely set to go to sleep after a specified period of time. This halts all processes, including the PSCAD simulation.
Solution
Whenever a computer will be unattended during a simulation (e.g. over lunch break, overnight, etc…), the Windows
settings could be switched to not go to sleep. The settings for this are in the Windows Control Panel | Power Options. It is
recommended to set Windows to high or full performance, and to never go to sleep, as shown in the following example:
Note
If the above solution does not prevent the computer from going to sleep during a simulation, then a network
policy is likely being imposed which overrides the Windows settings. In this case, your IT staff should be
consulted.
Page 140
PSCAD - Resolving Launching, Compiling, and Running Issues
Problem
The following error is displayed when a simulation is run:
Cause
Required prerequisite software for using PSCAD v4.6.2 and later is missing. Specifically, Visual Studio C++ 2015
Redistributables are missing.
Solution
Install this software as follows:
• Download and launch the Fortran Medic tool as per Appendix A.2 (Steps (a) and (b)).
• From the “Install” menu, install both the x86 and the x64 editions of Visual Studio C++ 2015 Redistributables.
Page 141
PSCAD - Resolving Launching, Compiling, and Running Issues
7.46 Receiving a “Visual Studio 2010, 2012 or 11 is not found in the system” Error Message
Problem 1
When compiling a PSCAD case, the following build errors display:
The above errors are present when using the following software:
• An Intel Fortran compiler, and
• One of the standalone editions of Visual Studio 2015+, for example the Professional Edition.
Note
The above errors are not present when using the Premier Partner (Shell) Edition of Visual Studio, which is free
and comes bundled with a licensed Intel Fortran compiler.
And when the Fortran Medic tool is run on this machine (as per Appendix A.2), this tool displays errors similar to either of
the following:
Visual Studio VS2015 C compiler is not fully installed. VC\lib folder does not exist.
Cause 1
The Visual Studio software installation is incomplete, and required software is missing. This can occur when a default
installation is performed, rather than a custom installation.
Page 142
PSCAD - Resolving Launching, Compiling, and Running Issues
Solution 1
Reinstall the Visual Studio software, ensuring to select a “Custom” installation, and ensuring to install all of the required
software. The following screenshot shows the required software, and should be similar to a user’s setup:
Following the installation, log out then log back in on the machine, then load and try compiling one of the simple PSCAD
cases (C:\Users\Public\Documents\PSCAD\4.x\Examples\tutorial\vdiv.pscx).
If there are no errors, the setup is ready. Or, if there are any errors, Intel Fortran might require re-installation. During this
installation:
• Select the “custom” rather than “default” installation on the “Options” page.
• If this is a licensed edition of the Intel Fortran compiler, de-select the Visual Studio software.
• On the integration page, select the appropriate Visual Studio edition and version.
• Log out then log back in on the machine.
• Test the setup by trying to run the simple PSCAD case again.
Problem 2
When compiling a PSCAD case, the following build errors display:
ERROR: Visual Studio 2013, 2015 or 2017 or 2019 is not found in the system.
‘ifort.exe’ is not recognized as an internal or external command,
NMAKE : fatal error U1077: ‘ifort.exe’ : return code ‘0x1’
Unable to generate a simulation executable for namespace ‘*’
The above errors are present when using the Intel Fortran compiler and Microsoft Visual Studio compiling combination.
And when the Fortran Medic tool is run on this machine (as per Appendix A.2), this tool displays errors as shown in
Appendix A.5 Item 62.
Cause 2
The wrong variable for Visual Studio is set in the environment.
For example, Visual Studio <version> Professional Edition is installed, but the variable is for the Community Edition.
This may occur if multiple editions of Visual Studio are or previously were installed.
Solution 2
Fix the variable path.
Page 143
PSCAD - Resolving Launching, Compiling, and Running Issues
This may be done using the Fortran Medic utility, see Appendix A.5 Item 62.
Or this may be done manually. Manual instructions are in this article.
Warning – Be very careful during the manual update!
Page 144
PSCAD - Resolving Launching, Compiling, and Running Issues
7.47 Receiving a “‘make’ is not recognized…” error message when compiling with GFortran
Problem
When compiling a PSCAD case using a GFortran compiler, build errors similar to the following display:
C:\Users\Public\...\*.gf42>call…
‘make’ is not recognized as an internal or external command, operable program or batch file.
Unable to generate a simulation executable for namespace ‘*’
And when the Fortran Medic tool is run, the following error displays:
The PATHEXT environment variable is missing the .EXE extention
(see Appendix A.5 Item 27 for further information on this error in the Fortran Medic tool)
Cause
The “.exe” is missing from the PATHEXT environment variable.
Solution
Use the Fortran Medic tool to easily fix the PATHEXT environment variable as per Appendix A.5 Item 27.
Page 145
PSCAD - Resolving Launching, Compiling, and Running Issues
Problem
When compiling a PSCAD case when user has no Windows Administrator privileges, build errors similar to the following
display:
Creaing EMTDC executable
C:\...\*.gf46>call “C:\Program Files (x86)\GFortran\4.6\bin\gf46vars.bat”
process_begin: CreateProcess(C:\...)…failed.
make (e=5): Access is denied.
make: *** [Station.o] Error 5
Unable to generate a simulation executable for namespace ‘*’
Cause
When a simulation is launched, executable files are created and launched as indicated in Appendix F. However, network
policies can detect and block these actions if the user has no Windows Administrator privileges.
Solution (1)
Always launch PSCAD with Windows Administrator privileges (from the Windows Start menu, browse to and right-click on
the PSCAD link, and select “Run as administrator”).
Solution (2)
Ask your IT staff to modify your network policy to allow a user to create and run batch files with normal Windows “User”
privileges.
Page 146
PSCAD - Resolving Launching, Compiling, and Running Issues
7.49 Security Software is preventing Simulations residing outside the Program Directories from running
Problem
Simulations will not run because security software, for example AppLocker, is configured to prevent creating or running
executable files that reside outside the Program directories.
See Appendix F for a list of executable files that are created and launched during a simulation.
Solution (1)
For future development (i.e. after v4.6.2)
The long term solution for future patches and releases is for PSCAD to create a modified <case name>mak.bat file which
redirects the TMP and TEMP variables. When the compile is complete, the TMP and TEMP user variables revert
automatically to the user’s environment settings.
Solution (2)
For PSCAD v4.6.2 and earlier
Redirect where these batch files are created by changing where the TMP and TEMP user environment variables point to:
Page 147
PSCAD - Resolving Launching, Compiling, and Running Issues
Solution (3)
For PSCAD v4.6.2 any version
Run your PSCAD project files from within Program Files. This setup may be tested to see if this resolves the issue as follows:
Note
Windows Administrator privileges are required for this.
• Make a copy of the “simpleac” file, which resides in the following location:
C:\Users\Public\Documents\PSCAD\4.6\Examples\tutorial\
• Save the copied file to a new folder in the Program Files.
• Launch PSCAD with elevated privileges (from the Windows Start menu, browse to and right-click on the PSCAD
link, and select “Run as administrator”).
• Load the simpleac example from Program Files, and run it.
• If the anti-virus does not prevent the case from running, you could move all of your cases to Program Files, and run
them from there.
Note
To avoid having to have administrator rights, your IT staff could make the Program Files accessible to all users,
rather than to just administrators.
Page 148
PSCAD - Resolving Launching, Compiling, and Running Issues
7.50 Receiving an “Unable to generate a simulation executable for namespace ‘*’” Error Message
Problem
When trying to compile a PSCAD project, the build fails, with the following messages:
Creating EMTDC executable...
…
Intel(R) MPI Library 2018 Update 1 for Windows* Target Build Environment for Intel(R) 64 applications
Copyright (C) 2007-2017 Intel Corporation. All rights reserved.
Copyright (C) 1985-2017 Intel Corporation. All rights reserved.
Page 149
PSCAD - Resolving Launching, Compiling, and Running Issues
7.51 Receiving an “Unable to generate a simulation executable for namespace ‘*’” Error Message
Problem
When trying to compile a PSCAD project with an Intel Fortran compiler, the editor is opened with a file called “echo.vbe”, at
which point the compiling pauses, and only resumes once the window is manually closed. This sequence repeats itself
several times, depending on the project size.
Ultimately, the build fails, with the following message:
Unable to generate a simulation executable for namespace ‘*’
When the latest Fortran Medic tool is run as per Appendix A.2, the messages as shown in Appendix A.5 Item 30 are
displayed.
This issue is not present when using the GFortran compiler to compile a PSCAD case.
Cause
A file called echo.vbe, installed by some unknown and unrelated third party software, is interfering when compiling a
PSCAD project with Intel Fortran composer.
Solution
Perform a search on your machine for the file called echo.vbe. This file may be in C:\Windows. Then, either:
• Remove this file if you do not require it for another application, or
• Temporarily rename this file when using PSCAD. For example:
Rename this file from echo.vbe to echo_1.vbe
Note
Windows administrator privileges are required to perform this.
Page 150
PSCAD - Resolving Launching, Compiling, and Running Issues
Problem
When compiling a PSCAD project, the simulation halts, and a message similar to the following displays:
Cause
Some object files that the PSCAD project requires are missing. For example, the above screenshot indicates that the library
associated with the Battery Model is missing.
Solution
Locate and link in the associated library. See Linking Objects and Libraries into PSCAD for more details.
Page 151
PSCAD - Resolving Launching, Compiling, and Running Issues
Problem
In PSCAD v4.5.4 or v4.6.0, when compiling a project that contains a blackbox module using the GFortran compiler, the
simulation halts, and a message similar to the following displays:
"filename.mak:line numer *** target pattern contains no '%'. Stop.
Unable to execute make.
Note
This model runs in PSCAD v4.5.4 or v4.6.0 when using the Intel Fortran compiler.
Applicability
PSCAD v4.5.4 v4.6.0
GFortran compiler
Cause
The associated object (*.o) file fails to generate when using the GFortran compiler due to blackbox incorrectly using
quotations in some of the file paths in the *.mak file.
Solution
Update your software to PSCAD v4.5.5, v4.6.1 or later.
Page 152
PSCAD - Resolving Launching, Compiling, and Running Issues
7.54 Receiving a “…machine type ‘X86’ conflicts with target machine type ‘X64’” Error Message
Problem
When compiling a case using the Intel Fortran compiler, build errors similar to the following display:
C:\...\*.obj : fatal error LNK1112: module machine type ‘86’ conflicts with target machine type ‘x64’
NMAKE : fatal error U1077: ‘“C:\...\Intel Fortran\...\link.exe”’ : return code ‘0x458’
Unable to generate a simulation executable for namespace ‘*’
Or
Cause
A 64-bit edition of Intel Fortran is being used to run a project, however, the project has precompiled libraries that were
created using a 32-bit edition of Intel Fortran. The same edition of Intel Fortran compiler used to create the libraries must
also be used to run the project.
Solution (1)
Re-run your project using the 32-bit edition of Intel Fortran.
Solution (2)
If you have access to the source code for the libraries, re-compile the libraries using Intel Fortran 64-bit edition, then re-run
the project using the 64-bit edition of Intel Fortran.
Page 153
PSCAD - Resolving Launching, Compiling, and Running Issues
7.55 Receiving a “This version of *\gcc.exe is not compatible with the version of Windows you’re running”
Error Message
Problem
When compiling a case using the GFortran 4.2 compiler, the build fails, and the following messages display in the Build
pane:
Creating EmTDC executable…
C:\...\*.gf42><call “C:\Program Files (x86)\GFortran\4.2.1\bin\gf42vars.bat”
Linking objects and libraries into binary ‘*.exe’
This version of C:\Program Files (x86)\GFortran\4.2.1\bin\gcc.exe is not compatible with the version of Windows you’re running. Check your
computer’s system information and then contact the software publisher.
make: *** [*exe] Error 1
Unable to generate a simulation executable for namespace ‘*’
Cause
Unknown at this time
Applicability
PsCAD v4.5.4 and v4.6.1 (may be applicable to other versions)
GFortran v4.2
Solution
Uninstall GFortran 4.2, re-install GFortran 4.2, log out and log back in your machine.
Test whether you can run one of the simple PSCAD examples with GFortran 4.2:
C:\Users\Public\Documents\PSCAD*\examples\tutorial\vdiv.pscx
Page 154
PSCAD - Resolving Launching, Compiling, and Running Issues
7.56 Receiving a “Visual Studio <versions> is not found in the system” Error Message
Problem
When compiling a case using the Intel Fortran compiler, the build fails, and the following error displays in the Build pane:
ERROR: Visual Studio <versions> is not found in the system.
When the Fortran Medic tool is run (Appendix A.2), an error similar to the one shown in Appendix A.5, Item 17 is displayed.
Cause
The required common tools environment variable, for example, VS90COMNTOOLS, is not defined on the machine.
Solution
Preferred Solution: Use the Fortran Medic tool to create the environment variable as per Appendix A.5, Item 17.
If not possible to use the Medic tool to create the environment variable, this may be done manually as follows:
a. Display your Environment Variables:
c. Using the table listed in this article, enter the variable name and value as applicable.
Page 155
PSCAD - Resolving Launching, Compiling, and Running Issues
7.57 Receiving a “Cannot determine the location of the VS Common Tools folder” Error Message
Problem 1
When compiling a case using the Intel Fortran compiler and Visual Studio, the build fails, and the following Build Messages
display:
ERROR: Cannot determine the location of the VS Common Tools folder.
LINK : fatal error LNK1181: cannot open input file '*.lib'
NMAKE : fatal error U1077: '"C:\Program Files (x86)\Microsoft Visual Studio 12.0\VC\bin\amd64\link.exe"' : return code '0x49d'
Unable to generate a simulation executable for namespace ‘*’
When the Fortran Medic tool is run (Appendix A.2), the error as indicated in Appendix A.5, Item 32 is displayed.
Note
By contrast, refer to Section 7.57 Problem 2, below, if the Medic does not detect an error with Visual Studio
common tools folder.
Cause 1
The PATH environment is missing the following standard system paths:
• C:\Windows
• C:\Windows\System32
System 1
This error was detected for the following software combination, but is certainly not limited to this combination:
PSCAD v4.5.4, Intel Fortran compiler 17, Microsoft Visual Studio 2012
Solution 1
• Run the Fortran Medic tool as per Appendix A.2, then create / add the values as per Appendix A.5, Item 32.
• Log out, then log back in to Windows. No need to reboot.
• Try compiling your PSCAD cases again.
Continued…
Page 156
PSCAD - Resolving Launching, Compiling, and Running Issues
Problem 2
When attempting to compile a project using Intel Fortran and Visual Studio, the compile fails, and the following message
displays:
ERROR: Cannot determine the location for the VS Common Tools folder.
‘nmake’ is not recognized as an internal or external commend,
Unable to generate a simulation executable for namsepace ‘*’
And when the Medic log file is run as per Appendix A.2, the VS Common Tools folder is properly detected. For example, the
following shows that Visual Studio 2010 is properly detected:
Required Environment and Registry Values
VS100COMNTOOLS (environment)
C:\Program Files (x86)\Microsoft Visual Studio 10.0\Common7\Tools\
Note
By contrast, refer to Section 7.57 Problem 1, above, if the Medic does detect an error with Visual Studio common
tools folder.
Cause 1
It is suspected that different editions of the Visual Studio version are somehow causing some confusion with regard to the
required Common Tools folder. For example, for the Visual Studio 2010 version, the Professional, Premier Partner, and
Express Editions are all installed.
Solution 1
• Uninstall all editions of Visual Studio
• Log out and log back in on your machine.
• Install VS2010 Pro (please do not install the other editions)
• Re-install Intel Fortran
• Test your setup as per this article:
https://www.pscad.com/knowledge-base/article/566
Continued…
Cause 2
The user does not have the required privileges to set the variables during a PSCAD build / run.
Page 157
PSCAD - Resolving Launching, Compiling, and Running Issues
Therefore, the Intel compiler is not able to detect the Visual Studio software.
Solution 2.1
1. Hard code the PATH and LIBRARY for the Visual Studio software.
2. Disable Private-to-Process in PSCAD:
PSCAD X4
PSCAD V5
Solution 2.2
Try always launching the PSCAD application with Windows Administrator privileges. Instructions are listed in this article.
This should give you the required privileges.
Page 158
PSCAD - Resolving Launching, Compiling, and Running Issues
7.58 Receiving an “Unable to generate a simulation executable for namespace ‘*’” Error Message
Problem
When compiling a case using the Intel Fortran compiler, the build fails, and the following Build Messages display:
Creating EMTDC executable
…
Intel Parallel Studio XE 2011
…Setting environment for using Microsoft Visual Studio 2008 x86 tools.
Unable to generate a simulation executable for namespace ‘*’
Cause
The cause is unknown at this time. It is only known that if the customer did not have Windows Administrator Privileges,
then a setting in PSCAD, that of hiding the Program Windows, was generating an error.
System
Using PSCAD v4.6.2, the PSCAD case could be compiled using GFortran, but not using Intel Fortran 12 / Microsoft Visual
Studio 2008.
Solution
1. Obtain Windows Administrator Privileges, then run the case using Intel Fortran 12 / VS 2008, or
2. Continue with simple Windows User Privileges, but set PSCAD to “Show Program Windows” during a run:
Page 159
PSCAD - Resolving Launching, Compiling, and Running Issues
Problem
When attempting to co-simulate a case with Matlab, the build fails, and the following Build warnings display:
Error Status returned from MATLAB engine when…
engOpen invalidCheck Matlab installation!
Cause
This error may be related to trying to use one version of Matlab (for example R2016a), but a different version of Matlab (for
example R2017b), is registered and integrated with external software.
Solution (1)
Switch the registered version of Matlab to the desired version. For example, if trying to register R2016a, perform as follows:
Note
Windows Administrator privileges are required.
a. Open the Windows Command prompt (from the Windows Start menu, right-click on “Command Prompt” and
select “Run As Administrator”):
b. Change the directory to the “bin” folder of the desired Matlab version. For example:
Page 160
PSCAD - Resolving Launching, Compiling, and Running Issues
d. This should register Matlab R2016a, and allow you to run one of the simple Matlab cases.
Solution (2)
Switch the registered version of Matlab to the desired version. For example, if trying to register R2016a, perform as follows:
Notes
1. Windows Administrator privileges are required.
2. This is applicable for Windows 10
a. Start MATLAB.
b. and typed regmatlabserver into the Command Window.
c. Register from MATLAB Command Prompt
d. To register MATLAB as an Automation server from within MATLAB, first start MATLAB, with the Run as
administrator option, if necessary.
e. Because you open MATLAB to run this command, you know what MATLAB version you will register.
Page 161
PSCAD - Resolving Launching, Compiling, and Running Issues
7.60 Receiving a “make: *** No rule to make target ‘*.mak’” Error Message
Problem
When attempting to run a PSCAD project with GFortran 4.2, the build fails, and the following Build messages display:
Creating EMTDC executable…
‘[SOME NETWORK PATH AND FILE NAME].gf42’
CMD.EXE was started with the above path as the current directory.
UNC paths are not supported. Defaulting to Windows directory.
C:\Windows>call “C:\Program Files (x86)\GFortran\4.2.1\bin\gf42vars.bat”
CMD does not support UNC paths as current directories.
Make: *.mak: No such file or directory
Make: *** No rule to make target ‘*.mak’. Stop
Unable to generate a simulation executable for namespace ‘*’
Background
The PSCAD project was originally loaded and being run from a local drive. The local drive was backed up while PSCAD was
being used, which caused the project to appear to be loaded and run from the backup location (on a network drive).
Compiling failed because a project may not be run from a network drive, it must be run from a local drive.
Cause
A file backup process running on the user’s machine, and is somehow changing the Working Directory, which affects PSCAD.
Solution
Modify the backup process so that it does not operate while PSCAD is running.
Perhaps the backup process could be configured to operate outside company hours.
Page 162
PSCAD - Resolving Launching, Compiling, and Running Issues
Problem
When attempting to run a PSCAD project with GFortran 4.6, the build fails.
Cause
Software that is incompatible with GFortran is installed. For example:
Cygwin, NutC, Winavr or Octave
Solution
Uninstall the offending software, or the issue might be resolved by simply changing the installation directory name of the
offending software.
See Section 7.1 for further options for Cygwin.
Page 163
PSCAD - Resolving Launching, Compiling, and Running Issues
Problem
When attempting to run a PSCAD project, the build fails.
The build message is similar to the following:
Unable to find C:/Program
When the Fortran Medic utility is run as per Appendix A.2, the message as shown in Appendix A.5 Item 44 will display.
Cause
The 8.3 filenames are not enabled.
Solution (1)
• Ask your IT staff to enable 8.3 filenames, or
Solution (2)
• Ensure that all folder and file names in the path to the PSCAD project file:
Are 8 characters or less, and
Contain no spaces.
Resources
See Sections 7.78 and 7.104 for further information.
Page 164
PSCAD - Resolving Launching, Compiling, and Running Issues
7.63 Receiving an “Unable to find Microsoft Visual C++ *** or higher” Build Error
Problem
When attempting to run one of the simple PSCAD projects using Intel Fortran, the build fails, with build messages similar to
the following:
Will execute (1): call "C:\Program Files (x86)\Intel\Compiler\Fortran\10.1.034\IA32\bin\ifortvars.bat" ia32
Will execute (2): nmake -f *.mak
Will execute (2): "C:\Users\Public\Documents\PSCAD\4.6\Examples\*\*.if9\*.mak.bat"
Creating EMTDC executable...
C:\Users\Public\Documents\PSCAD\4.6\Examples\*\*.if9>call "C:\Program Files (x86)\Intel\Compiler\Fortran\10.1.034\IA32\bin\ifortvars.bat" ia32
Intel(R) Visual Fortran Compiler for applications running on IA-32, Version 10.1.034
Copyright (C) 1985-2010 Intel Corporation. All rights reserved.
Unable to find Microsoft Visual C++ 7.1 or higher.
Note
The above messages occurred with Intel Fortran 10.1.
Cause
This error message can occur for many reasons. One reason is that the following software was installed in the wrong order:
Intel Fortran compiler (IVF) and Visual Studio (VS).
The VS software should be installed before IVF. Then when IVF is installed, IVF detects the installation of VS.
Note
This issue is likely to happen with older versions of Intel Fortran, e.g. IVF 10.1.
Solution
Re-install IFV so that it detects VS.
Page 165
PSCAD - Resolving Launching, Compiling, and Running Issues
7.64 Unable to Build a PSCAD Project using GFortran 4.2.1 or GFortran 4.6.2
Problem
when building one of the simple PSCAD projects with GFortran 4.2.1 or GFortran 4.6.2, the build fails, with a message
similar to the following:
Unable to generate a simulation executable for namespace '*'
When the Medic utility is run (Section A.2), an error is displayed, similar to the following:
Detected GFortran Versions
…
GFortran 4.2.1 compiles might fail - msys appears to be installed
Right-clicking on the error displays the following warning:
msys appears to be installed and can cause compiles using GFortran 4.2.1 to fail.
Cause
MSYS/MinGW is installed, and is causing an issue with the GFortran compiler.
Solution
There are three options for resolving this issue:
• Use the Medic utility to remove msys from the PATH: (or)
Run the Medic utility (Appendix A.2), and scroll to the heading “PATH (Machine) Environment Variable”.
Locate and right-click on the msys path, then select to remove it from the PATH, similar to the following:
Scroll to the heading “PATH (User) Environment Variable”, locate and right-click on the msys path, then
select to remove it from the PATH (similar to above screenshot).
• Rename the msys installation folder when using GFortran, (or)
• Uninstall the msys application.
Page 166
PSCAD - Resolving Launching, Compiling, and Running Issues
7.65 Unable to Build a Project in the PSCAD Free Edition using GFortran 4.2.1 or GFortran 4.6.2
Problem
When building one of the examples with GFortran 4.2.1 or GFortran 4.6.2, the build fails, with a message similar to the
following:
<SomePath>\<SomeCase>.gf46>call "D:\Program Files (x86)\GFortran\4.6\bin\gf46vars.bat"
Compiling 'Station.f' into object Code.
gfortran.exe: error: CreateProcess: No such file or directory
make: *** [Station.o] Error 1
Unable to generate a simulation executable for namespace '*'
Cause
The GFortran compiler is installed on the D drive, instead of on the default drive, drive C.
On some machines, if the GFortran compiler is installed on the D drive, compiling can fail.
Solution (1)
Move GFortran to C Drive as follows:
• Completely uninstall GFortran.
• Re-install GFortran, taking care to install it to C drive.
• Log out then log back in on your machine (or restart your machine) to apply the changes.
• Test your setup by trying to run one of the simple example cases using the newly installed compiler:
C:\Users\Public\Documents\PSCAD\<version>\Examples\tutorial\vdiv.pscx
• If there are no build errors, the issue should be resolved.
Solution (2)
Obtain Windows Administrator privileges, and always launch PSCAD with Windows Administrator privileges (right-click on
the PSCAD launching link, and select the option to “Run as administrator”).
Page 167
PSCAD - Resolving Launching, Compiling, and Running Issues
7.66 Receiving an “ERROR: Visual Studio 2013, 2015 or 2017 is not found in the system” Message
Problem
When trying to compile a PSCAD project using the following software combination:
• PSCAD v4.6.3 / Intel Fortran 18 / Visual Studio 2017 Professional Edition
The following Build Messages display:
ERROR: Visual Studio 2013, 2015 or 2017 is not found in the system.
'nmake' is not recognized as an internal or external command, operable program or batch file.
Unable to generate a simulation executable for namespace '*'
According to the Fortran Medic utility (Appendix A.3), the setup all looks good:
• The software is properly installed (PSCAD v4.6.3 / Intel Fortran 18 / Visual Studio 2017).
• PSCAD is configured to use Visual Studio 2017 (see Section 7.36).
• Intel Fortran 18 and Visual Studio 2017 are detected as being integrated.
Also, the user has logged in and out of the machine following the above installations.
Cause
There is likely an issue with integration between Intel 18 and Visual Studio 2017, even though no issue integration was
detected by the Medic utility.
Solution
Re-install Intel Fortran 18. During this installation, select the option to customize the installation when prompted, to be able
to:
• Ensure that the Visual Studio 2015 Shell Edition that comes bundled in the Intel 18 installation is not selected for
installation.
• Ensure that Visual Studio 2017 is selected to be integrated with Intel Fortran 18.
• Launch PSCAD and test your setup by trying to compile the vdiv example case:
C:\Users\Public\Documents\PSCAD\<version>\Examples\tutorial\vdiv.pscx
• If the above case build with no errors, your setup is ready for your own cases.
Page 168
PSCAD - Resolving Launching, Compiling, and Running Issues
7.67 Receiving a “'C' is not recognized as an internal or external command operable program or batch file”
Message
Problem
When trying to compile a PSCAD project using a GFortran compiler, the build fails, with the following messages:
'C’ is not recognized as an internal or external command, operable program or batch file.
…
Linking objects and libraries into binary ‘*.exe’
Unable to generate a simulation executable for namespace '*'
Cause
The PATH environment has the ampersand symbol, “&”, which is causing the build to fail. For example:
"C:\Program Files (x86)\<Some folder containing the & symbol>\<some installed program>\"
Continued…
Page 169
PSCAD - Resolving Launching, Compiling, and Running Issues
• Right-click on the green arrow corresponding to that path, and a dialog similar to the following should display:
• Select the option to “Delete it from the System environment variable space”, then select OK.
• Log out of Windows (to ensure that the PATH environment is completely updated), then log back in.
• Launch PSCAD and test your setup by trying to compile the vdiv example case:
C:\Users\Public\Documents\PSCAD\<version>\Examples\tutorial\vdiv.pscx
• If the above case build with no errors, your setup is ready for your own cases.
Solution (2): Remove any programs that are saved to the folder with the “&” symbol
• Open the Windows Programs and Features.
• Uninstall the program(s) residing within the folder with the “&” symbol, and any other programs that were
installed by it.
• Re-install the uninstalled software, but do not install it to the folder with the “&” symbol. Instead, install it to a
folder which does not contain the “&” symbol in the folder name.
• Log out of Windows (to ensure that your PATH environment is completely updated), then log back in.
• Launch PSCAD and test your setup by trying to compile the vdiv example case:
C:\Users\Public\Documents\PSCAD\<version>\Examples\tutorial\vdiv.pscx
• If the above case build with no errors, your setup is ready for your own cases.
Page 170
PSCAD - Resolving Launching, Compiling, and Running Issues
7.68 Receiving an “Export argument '*' cannot be declared since it is already declared as a local signal”
Error Message
Problem
When trying to compile a PSCAD project, the build fails, with the following messages:
Generating network and source code '*\*.f'
Export argument '*' cannot be declared since it is already declared as a local signal.
Cause
The same name has been used to define two items, which is not permitted. Each item must be uniquely identified. For
further information, please refer to the following article:
https://hvdc.ca/webhelp/ol-help.htm#Master_Library_Models/Miscellaneous/Export.htm
Solution 1 – Manually Rename one of the Items
Simply rename one of the items.
Solution 2 –Automatically Rename any Common Items
• Unload the PSCAD project.
• Select the setting “Make exported and local signals distinct”:
• Reload the PSCAD project. Any items with a common name will be renamed.
• Save the project.
• Further information on this setting may be found in this article:
https://hvdc.ca/webhelp/PSCAD/Application_Project_and_Workspace_Options/Application/Workspace.htm
Page 171
PSCAD - Resolving Launching, Compiling, and Running Issues
7.69 Receiving a “connect function failed with error: 10060” Error Message
Problem
When a machine is connected over VPN, a PSCAD build fails, with the following Build messages:
Linking objects and libraries into binary ‘*’
…
connect function failed with error: 10060
Then, if trying to run the project using Python scripting, the following message is displayed:
Exchange::Connect:Connection Timed Out: 10060
Failed to connect to [some IP address]:[some port]
Simulation has ended. Status code = 1
Also, an attempt to run the simulation from the DOS windows is successful (*.exe -startup:alone).
Background
Connection over VPN is somehow causing communication to be blocked between the PSCAD GUI (graphical user interface)
and the EMTDC (solution engine). This communication is required during a PSCAD simulation, and is performed over the
following port range: 30,000 to 40,000.
Even if VPN is later disabled, this communication can remain blocked (restarting the machine may restore communication).
Software
This issue has been seen with VPN software called Check Point Endpoint Connect.
Cause (1)
Protection software is causing this block, for example through the group policy or through the firewall.
Solution (1)
Configure PSCAD to use a specific port, and whitelist that port in the protection software.
Cause (2)
Other applications may be using all available ports.
Solution (2)
Determine which software is using the required ports, and turn off the software. To help determine this, you could try using
a software that oversees port usage (e.g. “CurrPorts”, https://www.nirsoft.net/utils/cports.html).
Cause (3)
In older versions, if PSCAD crashed, the ports being used by PSCAD could still be held up even after restarting the machine.
Solution (3)
Either update your software (preferred), ([email protected] (provide your license number)), or clear the ports manually.
Page 172
PSCAD - Resolving Launching, Compiling, and Running Issues
7.70 Receiving Error “make: *** [<some file name>.exe] Error 1” Error Message with an ETRAN
precompiled library
Problem
When attempting to run a project that is using an ETRAN precompiled library, the following Build messages display:
Cause
The ETRAN precompiled library was not linked in PSCAD.
Solution
• Download the (free) ETRAN library from the following website
(scroll down to “Download the E-TRAN Runtime library for PSCAD” on the webpage):
http://www.electranix.com/E-TRAN/support_downloads.htm
• Load the E-TRAN library into PSCAD.
For older PSCAD versions, the library must appear first (above the PSCAD project), in the workspace pane.
• Link the ETRAN precompiled library. Refer to the following article for tips:
https://hvdc.ca/knowledge-base/read,article/478/linking-objects-and-libraries-into-pscad/v:
Page 173
PSCAD - Resolving Launching, Compiling, and Running Issues
Problem
When attempting to run a PSCAD project, Error 1 displays in the Build Messages.
Cause
An access violation has occurred. The user does not have permissions for the file path in which the project files reside,
resulting in PSCAD not being to read or write to the file or directory, and the build fails.
Solution
Obtain full permissions for the file path, or
Move the project files to a location with full permissions.
Page 174
PSCAD - Resolving Launching, Compiling, and Running Issues
7.72 Receiving “The system cannot find the path specified” Error Message on a Machine with Anaconda
and PowerShell
Problem
On a machine on which Anaconda Python (aka Conda) and PowerShell are being used, when attempting to build a PSCAD
project, the build fails, and messages similar to the following display in the Build Messages pane:
Creating EMTDC executable…
The system cannot find the path specified.
C:\...\<filename>.gf46>call "C:\Program Files (x86)\GFortran\4.6\bin\gf46vars.bat"
The system cannot find the path specified.
Compiling 'Station.f' into object code.
Unable to generate a simulation executable for namespace ‘<filename>’
Cause
PowerShell is being used to activate Anaconda environments. However, this affects software which run terminal
commands, such as PSCAD, which for example invokes the Fortran compiler and linker. As a result, PSCAD builds fail.
Solution
Disable PowerShell from activating Anaconda environments (for example, run the command “conda init --reverse
powershell”). Then, use a different method for activating Conda environments, for example:
• Launch the Anaconda Prompt from the Windows Start menu, or
• Use a different framework. For example, one customer uses Git Bash, which is enabled by running command
“conda init bash”.
Page 175
PSCAD - Resolving Launching, Compiling, and Running Issues
7.73 Receiving a “LINK : fatal error LNK1181: cannot open input file 'ws2_32.lib'” Error Message
Problem
When attempting to build a PSCAD project with the Intel Fortran compiler, the build fails, and messages similar to the
following display in the Build Messages pane:
Linking objects and libraries into binary ‘*.exe'
LINK : fatal error LNK1181: cannot open input file 'ws2_32.lib'
NMAKE : fatal error U1077: '"C:\Program Files (x86)\Microsoft Visual Studio 12.0\VC\BIN\link.exe"' : return code '0x49d'
Stop.
Unable to generate a simulation executable for namespace '*'
Cause 2 - Windows 10 SDK Is Installed, but the PSCAD Compilation Cannot Access This
If the Windows 10 SDK kit installed, it is possible that there is some sort of policy on your machine that is blocking Windows
“Users” from being able to access this. There are two options to overcome this:
Ad‘ the 'ws2_’2.lib' path to the environment variable as per Solution 2.1, below, or
Always launch PSCAD with Windows Administrator Privileges as per Solution 2.2, below.
Solution 2.1 – Windows 10 SDK Kit is Installed, Windows “User” Does Not Have Access, So Hard Code the Variable
If the Windows 10 SDK kit is installed, but PSCAD can’t access the kit with Windows “User” privileges, add (hardcode) the
'ws2_32.lib' path to the environment variable as follows:
Warning:
1. Users should be very careful when modifying environment variables, to ensure programs are not affected.
Page 176
PSCAD - Resolving Launching, Compiling, and Running Issues
2. This solution will only allow running either the 32-bit or 64-bit edition of Intel Fortran.
Notes
This solution requires Windows Administrator privileges
Steps:
1. Determine ws2_32.lib path for either the 32-bit or 64-bit edition on your machine:
For example, if you want to be able to use the 32-bit edition of Intel Fortran, this path would be similar to the
following:
C:\Program Files (x86)\Windows Kits\10\Lib\10.<your version>\um\x86
For example, if you want to be able to use the 64-bit edition of Intel Fortran, this path would be similar to the
following:
C:\Program Files (x86)\Windows Kits\10\Lib\10.<your version>\um\x64
2. Display your Windows Environment Variables:
3. Add the path that you selected in Step 1 above to “System variables” as follows:
a. If you already have an existing “LIB” variable, highlight the LIB, select “Edit”, add the “New” path, and
select “OK”. For example, if the 64-bit edition of Intel Fortran will be used, the path would be entered
Page 177
PSCAD - Resolving Launching, Compiling, and Running Issues
b. Or, if you do not already have an existing LIB variable, select “New”, enter LIB in the "Variable name"
field, enter the path into the "Variable value" field, and select “OK”. For example, if the 64-bit edition of
Intel Fortran will be used, the path would be entered similarly to the following:
5. Log out then log back in on your machine to apply the changes.
Page 178
PSCAD - Resolving Launching, Compiling, and Running Issues
Solution 2.2 - Windows 10 SDK Kit is Installed, but Windows “User” Does Not Have Access, Maybe “Administrator” Does
If the Windows 10 SDK kit installed, it is possible that there is some sort of policy on your machine that is blocking Windows
“Users” from being able to access this.
This can be tested, by launching PSCAD with Windows Administrator Privileges. If this works, then PSCAD should always be
run with Administrator privileges (or else ask your IT team to not block users from Windows 10 SDK kit).
Page 179
PSCAD - Resolving Launching, Compiling, and Running Issues
7.74 Receiving a “Fatal Error: Parse error when checking module version for file 'ndde.com' opened at (1)”
error Message
Problem
When attempting to build a PSCAD project with a GFortran compiler, the build fails, and messages similar to the following
display in the Build Messages pane:
Fatal Error: Parse error when checking module version for file 'ndde.mod' opened at (*)
make: *** [Station.o] Error 1
Unable to generate a simulation executable for namespace '<some file name>'
Cause
Two or more GFortran versions have been installed into the same program folder. This causes the compilers to interfere
with each other because they are both becoming invoked, and compiling fails.
Solution
Uninstall all versions of GFortran, then reinstall them, ensuring that each version is saved to its own folder.
Please see the following references:
• Software download and setup tips
Page 180
PSCAD - Resolving Launching, Compiling, and Running Issues
7.75 Receiving a “Reached Buffer Limit:” Error Message When Calling in a Blackboxed Module
Problem
When attempting to call a blackboxed module from an external file, the following error displays in the Build Messages pane:
FORTRAN Script Error: Reached Buffer Limit: The buffer limit for a single line has been reached.
Cause
Perhaps the length of some of the signal names are too long.
Solution
Use shorter signal names.
Page 181
PSCAD - Resolving Launching, Compiling, and Running Issues
Problem
When trying to compile a PSCAD project using Intel Fortran, the simulation fails, and the following build errors display:
Cause
The Libucrt.lib path, which is part of Windows 10 SDK, is missing from the environment variables, causing the simulation to
fail.
Solution
Add (hardcode) the Libucrt.lib path into the environment variables.
Warning:
1. Users should be very careful when modifying environment variables, to ensure programs are not affected.
2. This solution will only allow running either the 32-bit or 64-bit edition of Intel Fortran.
Notes
This solution requires Windows Administrator privileges
Steps:
1. Determine the Libucrt.lib path on your machine:
For example, if you want to be able to use the 32-bit edition of Intel Fortran, this path would be similar to the
following:
C:\Program Files (x86)\Windows Kits\10\Lib\10.<your version>\ucrt\x86
For example, if you want to be able to use the 64-bit edition of Intel Fortran, this path would be similar to the
following:
C:\Program Files (x86)\Windows Kits\10\Lib\10.<your version>\ucrt\x64
2. Display your Windows Environment Variables:
Page 182
PSCAD - Resolving Launching, Compiling, and Running Issues
3. Add the path that you selected in Step 1 above to “System variables” as follows:
c. If you already have an existing “LIB” variable, highlight the LIB, select “Edit”, add the “New” path, and
select “OK”. For example, if the 64-bit edition of Intel Fortran will be used, the path would be entered
similarly to the following:
d. Or, if you do not already have an existing LIB variable, select “New”, enter LIB in the "Variable name"
field, enter the path into the "Variable value" field, and select “OK”. For example, if the 64-bit edition of
Intel Fortran will be used, the path would be entered similarly to the following:
Page 183
PSCAD - Resolving Launching, Compiling, and Running Issues
5. Log out then log back in on your machine to apply the changes.
6. Test your setup as per this article:
https://www.pscad.com/knowledge-base/article/566
7. If you then receive an error “LINK : fatal error LNK1181: cannot open input file 'ws2_32.lib'”, please refer to
Section 7.73 Cause 2.
8. If the issue persists, please feel free to obtain assistance from the MHI support desk ([email protected]). Include the
following in your request:
• A description of the issue.
• The items as listed in steps 2.c and 2.f of the following article:
https://www.pscad.com/knowledge-base/article/164
• Your PSCAD license number
Page 184
PSCAD - Resolving Launching, Compiling, and Running Issues
Problem
When trying to compile a PSCAD project using Intel Fortran, the simulation fails, and the following build errors display:
Cause
The settings for using Visual Studio within the PSCAD program files contain conflicting information. This is determined by
running the Medic utility as per Appendix A.2, and viewing the output for the PSCAD installation. As shown below, the
“state” is set to a mixture of 62 and 63; whereas the state should be set either to 62 (PSCAD is configured to use VS 2013
and older) or 63 (PSCAD is configured to VS 2015 and newer).
• Scroll down to the PSCAD v4.6.3 (x64) installation, right-click on the switch to reconfigure PSCAD (see above
screenshot). If prompted whether to reconfigure PSCAD for VS 2013 and older select the option to proceed.
• Re-run the Medic utility (select “Start” from the Actions menu).
• Scroll down to the PSCAD v4.6.3 (x64) installation, and check that the states are all set "62". If they are, then
select one of the following options:
If you will be using Visual Studio 2013 and older with your Intel Fortran compiler, leave the setting as is.
Your setup is properly configured.
If you will be using Visual Studio 2015 and newer with your Intel Fortran compiler, right-click on the
switch, and select the option to proceed when prompted whether to configure for VS 2015+.
• Launch PSCAD, and test your setup by trying to run one of the simple examples:
https://www.pscad.com/knowledge-base/article/566
• If the issue is not resolved, please feel free ton contact the MHI support desk ([email protected]). In your
request, include your license number, your Medic log file (Appendix A.3), and the build messages from trying
to run the test example in the above paragraph.
Page 185
PSCAD - Resolving Launching, Compiling, and Running Issues
• After PSCAD has been installed, select one of the following options:
If you will be using Visual Studio 2013 or older with your Intel Fortran compiler, there is no configuration
change to be made to PSCAD; PSCAD will be configured to use VS 2013 and older by default upon
installation.
Or, if you will be using Visual Studio 2015 or newer with your Intel Fortran compiler, download the most
recent version of the Medic utility and run it (Appendix A.2).right-click on the switch as shown in the
above screenshot, and select the option to proceed when prompted whether to configure for VS 2015+.
• Launch PSCAD, and test your setup by trying to run one of the simple examples:
https://www.pscad.com/knowledge-base/article/566
• If the issue is not resolved, please feel free to contact the MHI support desk ([email protected]). In your
request, include your license number, your Medic log file (Appendix A.3), and the build messages from trying
to run the test example in the above paragraph.
Page 186
PSCAD - Resolving Launching, Compiling, and Running Issues
7.78 Receiving the error message “Windows is not generating short (8.3) pathnames” Error Message in the
Fortran Medic Utility
Problem
When building a PSCAD case, the build fails, and the Fortran Medic (Appendix A) displays the following error (see Appendix
A.5, Item 15):
Folder Info
…
Conflicts
Windows is not generating short (8.3) pathnames
8.3 filename creation is disabled on all volumes
Also, your PSCAD project file and/or path do not conform to short 8.3 pathnames, which requires that filenames and
pathnames are no longer than eight characters each nor contain any spaces:
• Example of project name and path containing eight characters or less, and containing no spaces:
C:\12345678\123\123456_8.pscx
Cause
Windows is not generating short (8.3) pathnames on this machine. Consequently, compiling will fail for any PSCAD projects
when the above-listed 8.3 pathnames are not conformed to.
Solution
Ensure all PSCAD project files and path folder names are each a maximum of eight characters, and contain no spaces.
Page 187
PSCAD - Resolving Launching, Compiling, and Running Issues
7.79 Receiving the error message “” Error Message When Trying to Build a Project
Problem
When building a PSCAD case, the build fails, and the following error displays:
Cause
From the above screenshot, it is shown that the file is being run a network shared folder, and there may be an issue with
the network share.
Solution
Build and run the PSCAD project from a local drive:
• Copy the PSCAD case files to a local drive (for example, a new folder on your desktop).
Please note that it is not supported to run projects that are saved to a network drive; project files should be saved to and
run from a local drive.
Page 188
PSCAD - Resolving Launching, Compiling, and Running Issues
Problem
When trying to build a PSCAD project, a delay occurs before building commences.
Cause
Protection software might be blocking the simulation; in particular, the compiling software or PSCAD is being blocked.
Solution
Refer to the information, "Execution Permissions" listed in the Requirements documents, to help with whitelisting PSCAD
and compiler activities. The Requirements for PSCAD may be displayed from this topic.
Page 189
PSCAD - Resolving Launching, Compiling, and Running Issues
Problem
When using a newer Intel Fortran compiler that can use VS 2019 or VS 2019, a build error displays indicating that VS 2017
or VS 2019 is not detected.
And, running the Fortran Medic as per Appendix A.2 results in messages similar to the following:
Cause
The Visual Studio installation did not create the environment variable VS2017INSTALLDIR or VS2019INSTALLDIR (as
applicable) needed for the scripts to work. If this variable is not present, Intel will not detect Visual Studio, and PSCAD
compiling will fail, unless
Note
Compiling might work if another installation of Visual Studio that is compatible with the Fortran compiler is
present.
Solution
Use the Fortran Medic utility to create the environment variable as per Appendix. A.5 Item 52.
Notes
1. This solution is only applicable if using the Professional or Community Editions of Visual Studio.
If the Medic will not create the environment variable, this can be done manually as per this article.
Once the changes have been made, re-start the Fortran Medic (go Actions | Start), and the messages should now be
changed to be similar to the following:
Page 190
PSCAD - Resolving Launching, Compiling, and Running Issues
7.82 Receiving the error “Only a single namespace is included in the simulation set”
Problem
When attempting to run a simulation set, the build fails, and the following error displays:
Cause
The build fails because the user attempted to run one of the single projects, rather than the simulation set, when trying to
run a co-simulation project. For example, using the ENI co-simulation project that comes installed with PSCAD v4.6:
• One of the individual projects is selected to be run. The build fails, and the error displays.
Solution
Run the simulation set instead of the individual project, as shown:
Page 191
PSCAD - Resolving Launching, Compiling, and Running Issues
Problem
When attempting to run a simulation containing linking to a user written C library, the build fails, and the following Build
messages display:
fatal error C1900: Il mismatch between ‘P1’ version ‘[some_version]’ and ‘P2’ version ‘[some_version]’
LINK : fatal error LNK1257: code generation failed
NMAKE: fatal error U1077: ‘ “C:\Program Files (x86)\Microsoft Visual Studio\...\link.exe” ‘ : return code ‘0x4e9’
Stop
Unable to generate a simulation executable for namespace ‘[some_case]’
Solution
Patch your Microsoft Visual Studio software version to the most recent patch.
Resources
The following screenshot is from this link:
Page 192
PSCAD - Resolving Launching, Compiling, and Running Issues
7.84 Receiving a “The application was unable to start correctly (0xc0000142)” Error Message
Problem
When trying to build a case using PSCAD V5 Free Edition and GFortran 4.6, the build fails, with the following error:
Cause
winAVR may be installed. If so, this compiler is conflicting with the GFortran compiler and causing the build to fail.
Solution
Uninstall WinAVR, rename the WinAVR program folder, or remove all WinAvr segments from the PATH.
Page 193
PSCAD - Resolving Launching, Compiling, and Running Issues
7.85 Receiving an “ifortvars.bat...is not recognized as an internal or external command” Error Message
Problem
When trying to compile a project with Intel compiler 19.2+, following build messages similar to the following display:
When the Fortran Medic utility is run as per Appendix A.2, the error as shown in Appendix A.5 Item 61 is displayed.
Cause
The PSCAD installed file (fortran_compilers.xml file), which is used to call the Intel 19.2 batch file, is outdated and using the
wrong batch name:
• For Intel 19.2+: The batch name has been changed to setvars.bat
Solution
Update the fortran_compilers.xml file as per this article.
Page 194
PSCAD - Resolving Launching, Compiling, and Running Issues
Problem
When trying to compile a PSCAD project, the build fails, and the following build messages display:
‘“C:\Program Files (x86)\...\VCVarsQueryRegistry.bat”’ is not recognized as an internal or external command, operable program or batch file”
Error: Cannot determine the location of the VS installation.
…
Compiling…into object code.
…NMAKE : fatal error U1073: don’t know how to make ‘<SomePath>’
Stop
Unable to generate a simulation executable for namespace ‘*’
Cause
Visual Studio 2015 is being called with Intel compiler for the build. However, the Visual Studio 2015 installation is
incomplete or corrupt.
Solution
• Launch the Visual Studio installer and try to repair this installation, then test your setup as per this article.
Please note that the components as check-marked in the following image are required for building PSCAD projects, and
must be installed:
Page 195
PSCAD - Resolving Launching, Compiling, and Running Issues
Problem
When trying to compile a PSCAD project, the build fails, and the following build messages display:
Creating EMTDC executable
…
\Windows was unexpected at this time.
Unable to generate a simulation executable for namespace ‘*’
Cause
This error has been known to occur when there is an issue with an environment path.
Solution
Use the Fortran Medic Utility to detect and fix this issue as follows:
Note
Windows Administrator privileges on your machine are required to perform this.
• Download and run the most recent version of the Medic utility as per Appendix A.2.
Note
Ensure to select Yes when prompted by the Windows User Account Control so that the Utility can make the
change below.
• When the Utility has finished displaying your machine information, scroll down to see if there are any environment
errors. The following screenshot is an example showing an issue with the quotations in the User PATH:
• In the Utility, right-click on the error, and when prompted, select the option to let the Utility fix the path.
Page 196
PSCAD - Resolving Launching, Compiling, and Running Issues
Problem
When trying to compile a PSCAD project, the build fails, and the following build messages display:
Linking objects and libraries into binary '*.exe'
LINK : fatal error LNK1171: unable to load c2.dll
NMAKE : fatal error U1077: '"C:\Program Files (x86)\Microsoft Visual Studio 10.0\\Intel Fortran\Microsoft Files\VC\bin\link.exe"' : return code '0x493'
Stop.
Unable to generate a simulation executable for namespace '*'
Cause
The project that is being built contains c-code, but cannot be compiled because the installed edition of Visual Studio does
not have a c-compiler.
Solution
Install an edition of Visual Studio that contains a c-compiler. Further information on selecting your edition is available in this
article.
Resources
See Sections 7.62 and 7.104 for further information.
Page 197
PSCAD - Resolving Launching, Compiling, and Running Issues
7.89 Receiving a “Input file '\windows\emtdc.cfg' does not exist” Error Message
Problem
When trying to compile a PSCAD project, the build fails, and a build message similar to the following displays:
Background
The emtdc.cfg is a critical file that is installed during the PSCAD installation, and is installed to a number of locations:
• For GFortran v8.1, the file is saved to: C:\Program Files (x86)\<PSCADVersion>\emtdc\GF81\windows
• For GFortran v4.6, the file is saved to: C:\Program Files (x86)\ <PSCADVersion>\emtdc\GF46\windows
• For GFortran v4.2, the file is saved to: C:\Program Files (x86)\ <PSCADVersion>\emtdc\GF42\windows
• For Intel 12-14, the file is saved to: C:\Program Files (x86)\ <PSCADVersion>\emtdc\IF12\windows
• For Intel 15+ (x64), the file is saved to: C:\Program Files (x86)\ <PSCADVersion>\emtdc\IF15\windows
• For Intel 15+ (x86), the file is saved to: C:\Program Files (x86)\ <PSCADVersion>\emtdc\IF15_x86\windows
When the Fortran Medic Utility is run as per Appendix A.2, an error is displayed as shown in Item 66 of Appendix A.5.
Cause 1 – The File Was Missing from the Installer
The file “emtdc.cfg”, which is normally installed during the PSCAD installation, is somehow absent.
Perhaps the file was missing in the installer.
Solution 1
Obtain a new download of the PSCAD installer, and reinstall this software.
Cause 2 – The File was Not Installed During the PSCAD Installation
PSCAD may not be fully installed. Perhaps something went wrong during the installation of PSCAD.
Solution 2
Try uninstalling PSCAD and then installing everything again.
Cause 3 – Protection Software is Removing the File
Protection software may have stripped the file either during or after the installation.
Solution 3
Configure your protection software to not strip the file from the location/s listed in “Background”, above.
Cause 4 – The File Is Available, but the User Does Not Have Permissions
The user does not have file/folder permissions to access the locations listed in “Background”, above.
Solution 4
Obtain full file/folder permissions on the locations listed in “Background”, above.
Page 198
PSCAD - Resolving Launching, Compiling, and Running Issues
7.90 Receiving a “The tools for the configuration might not be installed” Error Message
Problem
When trying to compile a PSCAD project, the build fails, and the following build message displays:
Creating EMTDC executable…
C:\...\*.if15>call "C:\Program Files (x86)\Intel\Composer XE 2015\bin\ifortvars.bat" intel64
Intel(R) Parallel Studio XE 2015
Copyright (C) 1985-2015 Intel Corporation. All rights reserved.
Intel(R) Parallel Studio XE 2015 Update 6 Composer Edition (package 285)
The specified configuration type is missing. The tools for the configuration might not be installed.
Microsoft (R) Program Maintenance Utility Version 12.00.21005.1
Copyright (C) Microsoft Corporation. All rights reserved.
Linking objects and libraries into binary 'vdiv.exe'
LIBCMT.lib(typinfo.obj) : fatal error LNK1112: module machine type 'X86' conflicts with target machine type 'x64'
NMAKE : fatal error U1077: '"C:\Program Files (x86)\Microsoft Visual Studio 12.0\VC\bin\link.exe"' : return code '0x458'
Stop.
Unable to generate a simulation executable for namespace 'vdiv'
Cause
The 64-bit libraries in Visual Studio are not installed.
Solution
Uninstall then re-install the Visual Studio software.
During the installation, customize the installation, and ensure that all required items are installed.
Information on the required items is listed in Step c. of this article.
Reference
Refer to Section 7.107 for similar issue (missing VS 32-bit libraries)
Page 199
PSCAD - Resolving Launching, Compiling, and Running Issues
7.91 Receiving a “NMAKE : fatal error U1073: don’t’ know how to make…” Error Message
This is a very common error, and can be caused by several different reasons.
The following causes are only a few of many possible ones.
Problem 1
When trying to compile a PSCAD project, the build fails, and the following build messages display:
NMAKE : fatal error U1073: don't know how to make 'C:\<SomeFilepathAndFile>.lib'
Stop.
Unable to generate a simulation executable for namespace '<SomeProjectName>'
Cause 1
The project was being run from a network drive.
Solution 1
Move the project to a local drive (e.g. new folder on your desktop).
Problem 2
When trying to compile a PSCAD project, the build fails, and the following build messages display:
NMAKE : fatal error U1073: don't know how to make 'C:\<SomePartialFilepath><FollowedByASpace>'
Stop.
Unable to generate a simulation executable for namespace '<SomeProjectName>'
Note
In the above U1073 error line, the full path to the project file is supposed to be detected and listed.
However, the path is only detected up to the first space character, and the rest is not listed.
Cause 2
The presence of spaces in the filepath cause this error.
The cause is unknown, and this only occurs on some customers’ machines.
Solution 2
Ensure your project filename has no spaces in it.
And move your project files to a new filepath, one without any spaces in it.
Page 200
PSCAD - Resolving Launching, Compiling, and Running Issues
7.92 Receiving a “NMAKE : Visual Studio version Error: Unknown error environment configured” Error
Message
Problem
When trying to compile a PSCAD project, the build fails, and the following build messages display:
Visual Studio version Error: Unknown error environment configured.
Unable to generate a simulation executable for namespace '*'
Cause
It is suspected that one cause is due to the user missing some sort of permissions on his machine. Consequently, when
PSCAD is launched with normal Windows “user” privileges, PSCAD does not have the permissions to execute the action.
Solution
One possible work-around is to launch your PSCAD instance with Windows “Administrator” privileges. It is possible that the
Administrator level will give PSCAD the permissions to execute the action.
Information on launching PSCAD with Windows Administrator privileges is listed in this article.
Page 201
PSCAD - Resolving Launching, Compiling, and Running Issues
7.93 Receiving a “Link : fatal error LNK1104: cannot open file 'ifconsol.lib'” Error Message
Problem
When trying to compile a PSCAD project with Intel oneAPI Compiler and Microsoft Visual Studio, the build fails, and the
following build messages display:
Link : fatal error LNK1104: cannot open file 'ifconsol.lib'
NMAKE : fatal error U1077: '"C:\Program Files...\link.exe"' : return code '0x450'
Stop.
Unable to generate a simulation executable for namespace '*'
Software
Specifically, we have seen this issue once, with the following software:
PSCAD v5.0.0 Update 1, Intel Compiler 19.2.3663 and Microsoft Visual Studio 2022
Cause
During the build, the file ifconsol.lib, which is one of the Intel compiler files, cannot be found.
Solution
• Locate the path to the ifconsol.lib file.
For example, for the x64 edition of Intel 2022.0.0, this would be:
C:\Program Files (x86)\Intel\oneAPI\compiler\2022.0.0\windows\compiler\lib\intel64_win
For example, for the x86 edition of Intel 2022.0.0, this would be:
C:\Program Files (x86)\Intel\oneAPI\compiler\2022.0.0\windows\compiler\lib\ia32_win
For example, for Intel oneAPI v2022.0.0 (x64), the library path would be added as shown:
Page 202
PSCAD - Resolving Launching, Compiling, and Running Issues
7.94 Receiving a “cannot open output file vdiv.exe: No such file or directory” Error Message
Problem
When trying to compile a PSCAD project, the build fails, and the following build messages display:
Creating EMTDC executable...
C:\Users\Public\Documents\PSCAD\4.6\Examples\tutorial\vdiv.gf46>call "C:\Program Files (x86)\GFortran\4.6\bin\gf46vars.bat"
Linking objects and libraries into binary 'vdiv.exe'
vdiv c:/program files (x86)/gfortran/4.6/bin/../lib/gcc/mingw32/4.6.2/../../../../mingw32/bin/ld.exe: cannot open output file vdiv.exe: No such file or
directory
collect2: ld returned 1 exit status
make: *** [vdiv.exe] Error 1
Unable to generate a simulation executable for namespace 'vdiv''*'
Software
PSCAD v4.6.3 Update 3 and GFortran v4.6
Cause
To be determined.
Next Steps
To obtain an analysis from our support desk ([email protected]), please run the *.mak.bat file manually in a Windows
Command prompt in the temp folder, and send in the generated information:
• Launch PSCAD, and ensure Intel GFortran v4.6 is the selected compiler.
• Load the simple vdiv example into PSCAD:
C:\Users\Public\Documents\PSCAD\<version>\Examples\tutorial\vdiv.pscx
• Launch the run.
• When the build fails, display the temporary build folder:
(right-click on the project in the Workspace, select “Show in Folder”, and when the file explorer displays, open the
temporary “vdiv.gf46” folder). See image below.
Continued…
Page 203
PSCAD - Resolving Launching, Compiling, and Running Issues
Page 204
PSCAD - Resolving Launching, Compiling, and Running Issues
• At the prompt, enter the following text, then press the Return key:
vdiv.mak.bat
• Please send in a screenshot showing all the text that gets generated from that action in the Command Prompt for
our analysis.
Page 205
PSCAD - Resolving Launching, Compiling, and Running Issues
7.95 Receiving a “This action cannot be completed because the other program is busy” Error Message
Problem
When trying to compile a PSCAD project, the build fails, and the following error displays:
Server Busy
Software
Windows Server 2012 R2
PSCAD v4.6.0 or v4.6.1
Cause
The above error was displayed by Windows, and not by PSCAD. A PSCAD query to Windows may have just taken too long to
process, or just been rejected, resulting in the error.
Background
A query was added at PSCAD v4.6.0, requesting the number of physical and logical cores from Windows. This query was
related to a new high performance feature added at v4.6, PNI. The goal of the query was to be able to warn the user if the
number of machine cores was not suitable for high performance computing.
These queries were done via COM-based WMI (Windows Management Instrumentation), which generally, works well. But
in some instances, queries can time out, take a while, or be rejected, at which time, the above error message would display.
Specifically, this error was seen when using Windows Server 2012 R2.
Therefore, this query was removed as of v4.6.1 Hot Fix 1, and from any newer versions.
Solution 1
Try running PSCAD v4.6.0 or v4.6.1 on a different machine (i.e. not Window Server 2012 R2).
Solution 2
Try pressing “Retry” in the above dialog, this may work.
Page 206
PSCAD - Resolving Launching, Compiling, and Running Issues
7.96 Receiving a “LINK: fatal error LNK1181: cannot open input file 'wsock32.lib'” Error Message
Problem
When trying to compile a PSCAD project, the build fails, and the following Build error displays:
LINK: fatal error LNK1181: cannot open input file 'wsock32.lib'
Solution
Set PSCAD Build to “Private to process only”.
Page 207
PSCAD - Resolving Launching, Compiling, and Running Issues
7.97 Receiving a “make: *** INTERNAL: readdir: Invalid argument” Error Message
Problem
When trying to compile a PSCAD project, the build fails, and the following Build error displays:
make: *** INTERNAL: readdir: Invalid argument
System
This has been seen with GFortran v4.2 and v4.6.
Cause 1
Protection software may have quarantined one or more GFortran program files, causing the build to fail.
For example, for GFortran v4.6, the following file may have been quarantined:
C:\Program Files (x86)\GFortran\4.6\bin\make.exe
Solution 1
Uninstall then re-install the GFortran software, and whitelist the GFortran compiler in your protection software.
Test your setup by trying to run a simple example using GFortran:
C:\Users\Public\Documents\PSCAD\<version>\Examples\tutorial\vdiv.pscx
If the matter persists, please send the following to our support desk:
Your full build messages/errors/warnings, and your Fortran Medic log file as per Appendix A.3
Cause 2
Protection software may have quarantined one or more PSCAD program files.
The Fortran Medic Utility will detect the missing files, as shown in Item 56 of Appendix A.5.
Solution 2
Uninstall PSCAD. Then somehow whitelist the PSCAD installation in your protection software, then reinstall PSCAD.
Page 208
PSCAD - Resolving Launching, Compiling, and Running Issues
7.98 Receiving ifort Errors #10037 and #10408, and Fatal Error U1077
Problem
When trying to compile a PSCAD project, the build fails, and the following Build errors display:
ifort: error #10408: The Intel(R) 64 target compiler cannot be found in the expected location. Please check
your installation...
ifort: error #10037: could not find 'C:\Program...\ifort.exe'
MMAKE : fatal error U1077: '"C:\Program...\ifort.exe"' : return code '0xffffffff'
Stop
Unable to generate a simulation executable for namespace '*'
System
When using compiling combination: Intel Fortran Compiler and Microsoft Visual Studio
Cause 1
The PSCAD fortran_compilers.xml file is outdated, and is not detecting the newest compiler.
Solution 1
Follow instructions in this article to update your fortran_compilers.xml file.
Cause 2
Intel oneAPI is trying call the ifort.exe file, but is looking in the wrong location:
• According to the Build messages, Intel is looking in this location:
C:\Program Files (x86)\Intel\oneAPI\compiler\latest\windows\bin\
Cause 3
Your Intel oneAPI installation may not be complete.
Possibly, one of the two required “kits” is missing, or a required component of one of the kits was not selected during the
installation.
Solution 3
If your installed version of Intel oneAPI is not the most recent version, then download both the Base and HPC “kits” of the
latest version, ensuring to take notes of the setup tips as listed in Step 3 of this article.
Or if your installed version of Intel oneAPI is already the most recent version, then re-install both Intel oneAPI Base and HPC
“kits”, ensuring to take notes of the setup tips as listed in Step 3 of this article.
Page 209
PSCAD - Resolving Launching, Compiling, and Running Issues
Problem
When trying to compile a PSCAD project, the build fails, and the following Build errors display:
Creating EMTDC executable…
This program is blocked by group policy. For more information, contact your system administrator.
Unable to generate a simulation executable for namespace ‘*’.
Cause
Some permissions may be missing on your machine, or some activities may be blocked by your Group Policy.
Solution
Open a web browser to this page, locate and display the corresponding System Requirements document.
In the document, you will find Usage Requirements that must be enabled to build PSCAD projects.
Page 210
PSCAD - Resolving Launching, Compiling, and Running Issues
7.100 Receiving the Errors “waiting for unfinished jobs” and “unexpected end of file in ‘*.f”
Problem
When trying to compile a project, the build fails, and the following error messages display:
Cause
To be determined
Solution
To be determined
Page 211
PSCAD - Resolving Launching, Compiling, and Running Issues
7.101 Receiving Error LNK2038 “…mismatch detected for compiling the project…”
Problem
When trying to compile a PSCAD project, the build fails, and the following Build errors display:
*.lib(*.obj) : error LNK2038: mismatch detected for '*': value '1600' doesn't match value '1900' in main.obj
Cause
It seems that Visual Studio 2010 is installed and being called for compiling the project (i.e. “value '1600'”). However, the
project seems to require the use of Visual Studio 2015 (i.e. “value ‘1900’”).
Solution
Switch to the newer version of Visual Studio.
Some information on Visual Studio software is available in this article.
Page 212
PSCAD - Resolving Launching, Compiling, and Running Issues
7.102 Receiving a “'make' is not recognized as an internal or external command ” Build Message
Problem
When building a PSCAD case with a GFortran compiler, the simulation fails, with build messages.
For example, for GFortran v8.1:
Will execute (1): call C:\Program Files (x86)\GFortran\8.1\x64\bin\gf81vars.bat
Will execute (1): call "C:\Program Files (x86)\GFortran\8.1\x64\bin\gf81vars.bat"
Will execute (2): make -j -f vdiv.mak
Will execute (2): "C:\Users\Public\Documents\PSCAD\5.0\Examples\tutorial\vdiv.gf81\vdiv_mak.bat"
Creating EMTDC executable...
C:\Users\Public\Documents\PSCAD\5.0\Examples\tutorial\vdiv.gf81>call "C:\Program Files
(x86)\GFortran\8.1\x64\bin\gf81vars.bat"
'make' is not recognized as an internal or external command,
operable program or batch file.
Unable to generate a simulation executable for namespace 'vdiv'
Cause 1
The installation was not finalized because the user did not log out / log in on their machine after the compiler installation.
Solution 1
Log out / log in on your machine, then test your compiler.
Cause 2
The GFortran “make.exe” file is missing, likely blocked by your protection software.
This can be checked, by verifying whether the “make.exe” file is present.
For example, for the 64-bit edition of GFortran v8.1, the make.exe file would be located in a path similar to the following:
C:\Program Files (x86)\GFortran\8.1\x64\bin
Solution 2
If the make.exe file is missing, as determined in the above Cause:
• Turn off your protection software.
• Uninstall GFortran
• Re-download the GFortran installer and install this software, as per Step 4 of this article.
• Determine how to whitelist the GFortran program files in your protection software so that you can turn the
protection software back on.
• Log out / log in on your machine, then test your compiler.
Page 213
PSCAD - Resolving Launching, Compiling, and Running Issues
7.103 Receiving Error “vdiv.mak(4) : fatal error U1033: syntax error : ‘Fortran’ unexpected”
Problem
When building a PSCAD case, the simulation fails, with the following build messages:
Cause
It can be caused by non-ascii characters being present in the Intel Fortran Display String in the Windows Registry.
• For example, for the 64-bit edition of Intel 19.2.3747, the symbol “®” is included as shown in this location:
Computer\HKEY_LOCAL_MACHINE\SOFTWARE\WOW6432Node\Intel\Compilers\Fortran\192.3747\EM64T_NAT
IVE
• For example, for the 32-bit edition of Intel 19.2.3747, the symbol “®” is included as shown in this location:
Computer\HKEY_LOCAL_MACHINE\SOFTWARE\WOW6432Node\Intel\Compilers\Fortran\192.3747\IA32
System
This non-ascii character was introduced by Intel as of Version 19.2 (oneAPI). Previously (Intel 19.1 and older), ascii
characters were used, i.e. “(R)”.
This non-ascii character only seems to cause this compiling issue on some machines for some unknown reason. Most
machines with these versions of Intel containing the non-ascii character do not encounter the same error.
Continued…
Page 214
PSCAD - Resolving Launching, Compiling, and Running Issues
Solution 1 – Remove the Non-ascii Character Automatically Using the Fortran Medic Utility
The Fortran Medic utility can be used to replace the non-ascii text with ascii text:
• Run the Utility as per Appendix A.2. Ensure to launch the Utility with Windows Admininstrator privileges, so that
the Utility has the permissions to make changes.
• Fix the text as per Item 58 of Appendix A.5.
Note – perform this fix to both editions of Intel (x86 and x64)
• Scroll to both 64-bit and 32-bit locations, and remove the non-ascii script (i.e. “®”), as shown in the above images
under the “Cause” heading.
• Close the Registry.
• Launch PSCAD and test your setup by trying to build the vdiv project.
(Load the vdiv example from the location listed below, “Clean” the project (right-click on the project in the
Workspace tree, and select Clean), and run the project).
C:\Users\Public\Documents\PSCAD\<version>\Examples\tutorial\vdiv.pscx
Page 215
PSCAD - Resolving Launching, Compiling, and Running Issues
7.104 Receiving Errors “No such file or directory”, “Error 1”, and “Waiting for unfinished jobs”
Problem
When building a PSCAD case using GFortran compiler, the simulation fails, with the following build messages:
Creating EMTDC executable...
<some_project_filepath>\*.gf81>call "C:\Program Files (x86)\GFortran\8.1\x64\bin\gf81vars.bat"
Compiling 'Station.f' into object code.
Compiling 'Main.f' into object code.
gfortran.exe: error: Files: No such file or directory
gfortran.exe: error: Files: No such file or directory
gfortran.exe: error: (x86)\PSCAD50\emtdc\gf81\inc: No such file or directory
gfortran.exe: error: (x86)\PSCAD50\emtdc\gf81\inc: No such file or directory
gfortran.exe: error: Files: No such file or directory
gfortran.exe: error: Files: No such file or directory
gfortran.exe: error: (x86)\PSCAD50\emtdc\gf81\windows: No such file or directory
gfortran.exe: error: (x86)\PSCAD50\emtdc\gf81\windows: No such file or directory
make: *** [*.mak:70: Station.o] Error 1
make: *** Waiting for unfinished jobs....
make: *** [vdiv.mak:70: Main.o] Error 1
Unable to generate a simulation executable for namespace 'vdiv'
Versions
This issue occurred with PSCAD v500 Update 1, and for all three versions of GFortran that are compatible with PSCAD
v5.0.0:
• GFortran v4.6
• GFortran v8.1 (32-bit)
• GFortran v8.1 (64-bit)
Cause
The issue occurs due to the presence of spaces within the filepath to the “emtdc” folder.
For example, the path may be similar to the following, which contains two spaces:
C:\Program Files (x86)\PSCAD50\emtdc
The machine is not configured to resolve spaces within the pathname, therefore the build fails.
Solution 1 – Enable Windows short 8.3 pathnames
Enable Windows short 8.3 pathnames on your machine. See Sections 7.62 and 7.78 for more information.
Solution 2 – Move the “emtdc” Folder to a Filepath That Does Not Contain Spaces
• Close all instances of PSCAD.
• Move the entire “emtdc” folder to a filepath not containing any spaces:
The emtdc folder should be in a location similar to the following: C:\Program Files (x86)\PSCAD50\emtdc
Move the emtdc folder to a folder with no spaces in the path, for example: C:\PSCADFix\emtdc
• Launch PSCAD.
Continued…
Page 216
PSCAD - Resolving Launching, Compiling, and Running Issues
• Configure the new location in Application Options | Tools | EMTDC Folder as follows:
Delete “$(HomeDir)\emtdc”.
Enter the new filepath to the emtdc folder. For example, “C:\PSCADFix\emtdc”.
Press “Ok” to save your changes.
• Restart PSCAD, and test and troubleshoot your compiling setup as per this article.
Resources
• See Sections 7.62 and 7.88 for further information.
Page 217
PSCAD - Resolving Launching, Compiling, and Running Issues
7.105 Receiving Errors “SyntaxError: invalid syntax”, “NMAKE : fatal error U1077…” and “return code '0x1'”
Problem
When trying to build a PSCAD case, the simulation fails, with similar build messages:
SyntaxError: invalid syntax
NMAKE : fatal error U1077: 'C:\Python37\Lib\copy.PY' : return code '0x1'
NMAKE : fatal error U1077: 'C:\Python37\Libs\copy.PY' : return code '0x1'
NMAKE : fatal error U1077: 'C:\Python37-32\Lib\copy.PY' : return code '0x1'
NMAKE : fatal error U1077: 'C:\Python37-32\Libs\copy.PY' : return code '0x1'
NMAKE : fatal error U1077: 'C:\Python27\Lib\copy.PY' : return code '0x1'
NMAKE : fatal error U1077: 'C:\Python27\Lib\copy.PY' : return code '0x1'
Unable to generate a simulation executable for namespace '*'
Causes
First, one or more of the following Python variables are present in the Machine PATH, which are somehow blocking the
simulation:
• C:\Python37\Libs
• C:\Python37\Lib
• C:\Python37-32\Libs
• C:\Python37-32\Lib
• C:\Python27\Lib
• C:\Python27\libs
Solution
Remove the above Python variables as applicable from the Machine PATH using the Fortran Medic utility:
• Launch and start the Medic as per Appendix A.2.
Note – At Step b, ensure to select Yes to run with Administrator privileges.
• Scroll down to the heading “PATH (Machine) Environment Variable”.
• Under the above heading, if any of the Python variables as listed above are present, right-click on each of them, and select
the option to remove them from the environment.
For example:
Continued
Page 218
PSCAD - Resolving Launching, Compiling, and Running Issues
For example:
Page 219
PSCAD - Resolving Launching, Compiling, and Running Issues
7.106 Receiving Build Messages Indicating that Microsoft Visual Studio is Not Detected
Problem
When trying to build a PSCAD case, the simulation fails, with build messages indicating that Microsoft Visual Studio is not
installed.
When the Fortran Medic utility is run as per Appendix A.2, the messages as shown in Appendix A.5 Item 59 are displayed.
Cause
A suitable, compatible version of the Microsoft Visual Studio software is installed, but is missing the C++ workload
(module).
Without the C++ workload module, PSCAD cases cannot be compiled using the Intel FORTRAN compiler/Visual Studio
compiling software.
Solution 1 - If using Visual Studio 2015
• Uninstall Visual Studio 2015.
• Launch the Visual Studio 2015 installer. When the Visual Studio installer launches, select Programming Languages
| Visual C++, and also select the required sub-elements as specified in Step c of this article.
• After the intallation, log out then log back in on your machine.
• Test your setup as per this article.
Solution 2 - If using Visual Studio 2017 and Newer
Modify your existing Microsoft Visual Studio software to include the missing C++ workload and sub-elements as follows:
• Launch the Windows Apps & Features or Programs and Features.
• Locate and select the Visual Studio product, then select the option to “Modify” or “Change” as applicable.
• When the Visual Studio installer launches, select the “Desktop development with C++” for installation, as well as
required sub-elements as specified in Step c of this article.
• After the intallation, log out then log back in on your machine.
• Test your setup as per this article.
Page 220
PSCAD - Resolving Launching, Compiling, and Running Issues
Problem
When trying to build a PSCAD case, the simulation fails, with build messages similar to the following:
Linking objects and libraries into binary ‘vdiv.exe’
libucrt.lib(exit.obj) : fatal error LNK1112: module machine type ‘x64’ conflicts with target machine type ‘x86’
NMAKE : fatal error U1077: ‘”C:\Program Files (x86)\Microsoft Visual
Studio\2017\Professional\VC\Tools\MSVC\14.16.27023\bin\HostX86\x86\link.exe”’ : return code ‘0x458’
Stop.
Unable to generate a simulation executable for namespace ‘vdiv’
Cause
There are missing Visual Studio 32-bit libraries.
System
Trying to build with Intel 32-bit
Solution
For VS 2015: Uninstall Visual Studio completely. Then relaunch the installer, customize the installation, and ensure that all
required items are selected for installation as per Step c. of this article.
For VS 2017 and newer: Relaunch the Visual Studio Installer. During the installation, customize the installation, and ensure
that all required items are selected for installation as per Step c. of this article.
Reference
Refer to Section 7.90 for similar issue (missing VS 64-bit libraries)
Page 221
PSCAD - Resolving Launching, Compiling, and Running Issues
Problem
When trying to build a PSCAD case, the simulation fails, with build messages similar to the following:
ERROR: Visual Studio 2022 or 2019 or 2017 is not found in "C:\Program Files [(x86)]\Microsoft Visual Studio\<2022 or 2019 or
2027>\<Edition>", please set VS2022INSTALLDIR or VS2019INSTALLDIR or VS2017INSTALLDIR
Cause
Visual Studio is only partially installed. During the installation, the Visual C++ was not selected for installation.
Solution
For VS 2015: Uninstall Visual Studio completely. Then relaunch the installer, customize the installation, and ensure that all
required items are selected for installation as per Step c. of this article.
For VS 2017 and newer: Relaunch the Visual Studio Installer. During the installation, customize the installation, and ensure
that all required items are selected for installation as per Step c. of this article.
Page 222
PSCAD - Resolving Launching, Compiling, and Running Issues
7.109 Receiving a “Project Output Storage Requirements are xx MB…” Error Message
Problem
When atempting to build a PSCAD project, the build stops, and a message similar to the following displays:
Cause
It is likely that the Intel 32-bit edition is selected for this simulation. This edition, having a 2 GB memory allocation limit, is
not sufficient for your similation. When the simulation is launched, the memory allocated for the transfer of data between
PSCAD and the simulation may be pushing past the 32 bit limit on the simulation side.
Solution
The following are some suggestions for overcoming this error:
• Switch to a 64-bit edition of Intel compiler.
Page 223
PSCAD - Resolving Launching, Compiling, and Running Issues
Problem
When attempting to build a project, build errors similar to the following display:
‘*’ cannot be added. The storage table is at capacity of 256.
Cause
The user is hitting the 256 limit for number of Radio Links in their project. This limit was set to protect the user from using
an excessive number of Radio Links and affect simulation and memory. The limit has a hard limit of 1024 but the setting in
PSCAD workspace has a soft limit of 256.
It may be possible to increase the limit (Workspace Options | Build | Maximum Radio Links). However, an excessive number
may cause performance problems.
Page 224
PSCAD - Resolving Launching, Compiling, and Running Issues
7.111 Receiving a “NMAKE: fatal error U1077…return code ‘0x460’” Error Message
Problem
When attempting to build a project, build errors similar to the following display:
Linking objects and libraries into binary '*.exe'
*.exe : fatal error LNK1120: XX unresolved externals
NMAKE : fatal error U1077: '"C:\Program Files (x86)\Microsoft Visual
Studio\2019\Community\VC\Tools\MSVC\14.28.29910\bin\HostX86\x86\link.exe"' : return code '0x460'
Stop.
Unable to generate a simulation executable for namespace '*'.
Cause
Visual Studio is only partially installed. During the installation, the Visual C++ was not selected for installation.
Solution
For VS 2015: Uninstall Visual Studio completely. Then relaunch the installer, customize the installation, and ensure that all
required items are selected for installation as per Step c. of this article.
For VS 2017 and newer: Relaunch the Visual Studio Installer. During the installation, customize the installation, and ensure
that all required items are selected for installation as per Step c. of this article.
Page 225
PSCAD - Resolving Launching, Compiling, and Running Issues
Problem
Compiling PSCAD projects with GFortran used to work. However, after an update to the Operating System, build errors
similar to the following display when attempting to build a project with GFortran:
Creating EMTDC executable…
<some file path>\gf46vars.bat”
'make’ is not recognized as an internal or external command, operable program or batch file.
Unable to generate a simulation executable for namespace '*'.
Cause
Some unknown issue caused by the Operating System update.
Solution
Uninstall then reinstall the GFortran compiler as follows:
• Uninstall GFortran using the Windows Control Panel:
• After the installation, log out then log back in on your machine, then retry running the simple example:
C:\Users\Public\Documents\PSCAD\<version>\Examples\tutorial\vdiv.pscx
• If the example does not run, try uninstalling then reinstalling PSCAD, then retry running the above example.
• If the matter is still not resolved, please send in Items 2.c and 2.f of this article to the support desk, along with
your PSCAD license number.
Page 226
PSCAD - Resolving Launching, Compiling, and Running Issues
7.113 When Multiple Versions of Intel oneAPI are Installed, the Newest Installed Version Will Always Be
Called
Problem
When multiple versions of Intel oneAPI are installed, the newest installed version of Intel oneAPI will always be
called and used to compile a PSCAD project, regardless of which version has been selected in the PSCAD
Application Options.
Cause
Somehow, the “newest” version of Intel becomes called into the build, instead of the absolute version number
being called into the build.
Solution
If you do not need to switch between multiple versions of Intel oneAPI, simply uninstall any versions that you do
not need, and just use the one version.
Or, if you do need to switch between multiple versions, then consider using two different machines, each one
with a different version.
Page 227
PSCAD - Resolving Launching, Compiling, and Running Issues
Problem
An error similar to the following displays when attempting to build a project:
Cause
Error #1455 can be related to the machine memory. This error can display when the PSCAD TLine program is being run,
indicating that the machine is running out of either memory or virtual paging memory.
Further information may be found here:
https://community.intel.com/t5/Intel-Fortran-Compiler/OMP-System-error-1455-The-paging-file-is-too-small-for-this/td-
p/932165
Solution
Ideally, to ensure system stability:
• The paging file size should be 1.5 times the physical memory at a minimum.
For example, for 8 GB RAM, the minimum paging file size would be: 8 GB x 1.5 = 12 GB RAM
• The paging file size should be up to 4 times the physical memory at most.
For example, for 8 GB RAM, the maximum paging file size would be: 8 GB x 4 = 32 GB RAM
Page 228
PSCAD - Resolving Launching, Compiling, and Running Issues
7.115 Receiving Error “Error: A license for FCompW is not available now”
Problem
An error similar to the following displays in the Build Messages when attempting to build a project:
Error: A license for FCompW is not available now (-96,7,11001).
NMAKE : fatal error U1077: '"C:\Program Files (x86)\Intel\Composer XE 2011 SP1\bin\ia32\ifort.exe"' : return code '0x1'
When the Fortran Medic Utility is run as per Appendix A.2, the error as shown in Appendix A.5 Item 63 displays.
Cause
The INTEL_LICENSE_FILE variable is either not set or is improperly set in the Windows Environment Variables.
The variable should be set as shown:
Solution
Fix or add the variable in the Windows Environment Variables.
Further Resources
Other reasons for being unable to activate an Intel License are listed in this article.
Page 229
PSCAD - Resolving Launching, Compiling, and Running Issues
Problem
When trying to build a project using a GFortran compiler, an error similar to the following displays:
Cause
The above errors typically display if GFortran was installed, but the installation was not properly finalized. Since the
compiler installation was not properly finalized, the compiler was not detected during the PSCAD build.
Solution
Normally, to properly finalize the GFortran installation, the user would need to log out and log back in on the machine.
If the above step does not resolve the matter, try restarting the machine.
Page 230
PSCAD - Resolving Launching, Compiling, and Running Issues
Problem
When trying to build a project using a GFortran compiler, build messages similar to the following display:
<SomePath>\<SomeFileName>.zip\...
Compiling ‘*’ into object code.
Compiling ‘*’ into object code.
Gfortran.exe …
make: *** [Main.o] Error 1
make: *** Waiting for unfinished jobs…
make: *** [Station.o] Error 1
Unable to generate a simulation executable for namespace ‘*’
Cause
There were multiple causes, but only one was identified to MHI, that the project being run had not been unzipped first
(as shown in the above build messages).
Solution
Ensure to unzip the project files, then retry building the project.
Page 231
PSCAD - Resolving Launching, Compiling, and Running Issues
7.118 Receiving Errors “ifort.exe is not recognized…” and “NMAKE : fatal error U1077: 'ifort.exe'…”
Problem
When trying to build a PSCAD project using the Intel oneAPI Fortran compiler, the build fails, and messages similar to the
following display:
Compiling 'Station.f' into object code.
'ifort.exe' is not recognized as an internal or external command,
operable program or batch file.
NMAKE : fatal error U1077: 'ifort.exe' : return code '0x1'
Stop.
Unable to generate a simulation executable for namespace ‘<SomeFileName>’
When the Intel program files are viewed, it is determined that the application launch file, Ifort.exe, is missing. Therefore,
the Intel compiler cannot be launched during the attempted build.
For example, for Intel oneAPI, the Ifort.exe file should be listed in the following two locations:
• C:\Program Files (x86)\Intel\oneAPI\compiler\latest\windows\bin\intel64_ia32, and
• C:\Program Files (x86)\Intel\oneAPI\compiler\latest\windows\bin\intel64
Cause
This issue can occur when one of the two Intel oneAPI toolkits is updated, and the other is not. Here are some possible
scenarios:
• The Base Toolkit and HPC Toolkits were previously installed on a machine.
Then, the Base Toolkit becomes updated, but the HPC Toolkit is not updated.
• Or, for a new installation of Intel oneAPI, the Base Toolkit is installed, but the HPC toolkit is not.
Solution
Update the HPC Toolkit to the same version as the Base Toolkit.
Then log out then log back in on your machine, and test your setup.
Page 232
PSCAD - Resolving Launching, Compiling, and Running Issues
7.119 An Intel Fortran Compiler Cannot be Selected in the PSCAD Application Options | Dependencies Dialog
Problem
An installed Intel Fortran compiler is not listed in the PSCAD Application Options | Dependencies dialog, and can therefore
not be selected for use.
Cause 1
There was some sort of issue during the installation of this product, and it did not properly install.
Solution 1
Reinstall this compiler. Refer to the following articles for setup information:
• Licensed Version of Intel
Then log out and log back in on your machine, and test your setup.
Cause 2
The compiler was fully installed, but for some reason, the variable was not added to the Windows Environment during the
installation.
On some machines, the absence of the variable does not affect PSCAD builds. But on other machines, if the variable is
missing, PSCAD is unable to detect and use the compiling software during a build.
To view whether the variable is present in the Windows Environment, run the Fortran Medic Utility as per Appendix A.2,
and then refer to Appendix A.5 Item 8 for details.
Solution 2
Add the variable to the Windows Environment as per Appendix A.5 Item 8.
Then log out and log back in on your machine, and test your setup.
Cause 3
If the compiler is Intel oneAPI, the installation may be incomplete; the Base Toolkit was installed, but the HPC Toolkit was
not.
The normal process is to install the Base toolkit first, and the HPC toolkit second. It is during the installation of the HPC
toolkit that the variable is set in the Environment.
Solution 3
Install the HPC Toolkit. Refer to Solution 1 above for reference information.
Then log out and log back in on your machine, and test your setup.
Page 233
PSCAD - Resolving Launching, Compiling, and Running Issues
7.120 Receiving Errors: 'ifort.exe' is not recognized as an internal or external command, operable program or
batch file”, and “fatal error U1077”, and “return code '0x1'”
Problem
When trying to build a PSCAD project using Intel Fortran v19.2 compiler, the build fails, with messages similar to
the following:
Also, when the Fortran Medic utility is run (Appendix A.2), messages similar to Item 65 in Appendix A.5 display.
Cause
There is an issue with the Intel 19.2 installation. Either it is not fully installed, or else different versions of the
two toolkits (Base and HPC) were mixed.
Solution
• Completely uninstall your Intel v19.2 software.
• Re-download the Intel v19.2 software as specified in Step (e) of this article.
• Re-install the software, taking care to follow the information in Step (e) of the above article, to:
Page 234
PSCAD - Resolving Launching, Compiling, and Running Issues
7.121 Receiving Error Message "Symbol 'node' at (<SomeNumber>) conflicts with symbol from module
'<SomeModuleName>'"
Problem
When trying to build a PSCAD project, the build fails, with the following build error:
Error: Symbol 'node' at (<SomeNumber>) conflicts with symbol from module '<SomeModuleName>', use-
associated at (2)
Cause
A bus in the system is named "Node", which seems to be causing a conflict with the compiler.
Solution
Change the name of the bus. Do not name any components "Node".
Page 235
PSCAD - Resolving Launching, Compiling, and Running Issues
7.122 Receiving Build Error “setvars.bat”’ is not recognized as an internal or external command”
Problem
When trying to build a project using Intel oneAPI compiler, the build fails, and the following errors display in the Build
Messages pane:
‘“C:\Program Files (x86)\...\setvars.bat”’ is not recognized as an internal or external command,
nmkake is not recognized as an internal or external command,
Unable to generate a simulation executable for namespace ‘*’
Cause
The Intel oneAPI Base Toolkit and the HPC Toolkit are not fully installed.
Solution
Reinstall the Base Toolkit and the HPC Toolkits.
Refer to this article for hints on minimum required components to select during the installation.
Page 236
PSCAD - Resolving Launching, Compiling, and Running Issues
Problem
When trying to build a project using Intel oneAPI compiler, the build fails, and the following errors display in the Build
Messages pane:
:: initializing oneAPI environment…
Initializing Visual Studio command-line environment…
“\Intel\oneAPI\compiler\<version>\windows\bin was unexpected at this time.
Unable to generate a simulation executable for namespace ‘*’
Also, when Fortran Medic utility is run as per Appendix A.2, an error is displayed as shown in Appendix A.5 Item 8.
Cause 1
The Intel compiler variable is not set in the Windows Environment, and therefore, PSCAD is unable to use the compiler.
Solution 1
Add the variable to the Environment as per this article.
Cause 2
There is a problem with the installation of the Intel oneAPI compiler.
Solution 2
Refer to this article for setup tips.
Cause 3
Unbalanced double quotes are present in the Windows Environment.
See Appendix A.5 Item 51 for further information.
Solution 3
Remove the double quotes:
• Download and run the newest Fortran Medic utility as per Appendix A.2
Note - Ensure to select Yes, to launch this utility with elevated privileges.
• When the results display in the Utility, scroll down to find and fix the error as specified in Appendix A.5 Item
51.
Page 237
PSCAD - Resolving Launching, Compiling, and Running Issues
7.124 Receiving Build Error “Visual Studio was not found in the standard installation location”
Problem
When trying to build a project using the Intel Fortran compiler, the build fails, and the following errors display in the Build
Messages pane:
:: WARNING: Visual Studio was not found in the standard installation location:
LINK : fatal error LNK1181: cannot open file 'ws2_32.lib'
NMAKE : fatal error U1077: 'link.exe /out:<SomeFileName>.exe / nologo /nodefaultlib:libc.lib /nodefaultlib:libcmtd.lib /subsystem:console...
Unable to generate a simulation executable for namespace '<SomeFileName>'
Also, it is detected that one or more of the required Visual Studio installation files are missing. For example, these are the
required files:
Cause
Visual Studio is not completely installed. Some required components were not selected during the installation.
Solution
Launch the Visual Studio installer, and ensure to select and install the required components as specified in this article.
Page 238
PSCAD - Resolving Launching, Compiling, and Running Issues
Problem
While using PSCAD v5.0.2 to build a PSCAD project, PSCAD crashes with error 0xC0000409 every time.
Cause
One of the definitions in the PSCAD project has a long text description, and this is causing PSCAD to crash.
Please see the following sample:
Page 239
PSCAD - Resolving Launching, Compiling, and Running Issues
7.126 Receiving Build Error “Visual Studio command-line environment was not configured”
Problem
When attempting to bId a project using the Intel Fortran compiler, the build fails, with messages similar to the following:
Creating EMTDC executable...
C:\Users\Public\Documents\PSCAD\5.0\Examples\tutorial\vdiv.if18_x86>call "C:\Program Files
(x86)\Intel\oneAPI\compiler\2023.2.0\window\..\..\setvars.bat" VS2022 ia32
:: initializing oneAPI environment...
Initializing Visual Studio command-line environment...
Visual Studio command-line environment was not configured.
Unable to generate a simulation executable for namespace 'vdiv'
When the Fortran Medic Utility is run as per Appendix A.2, an error similar to that shown in Appendix A.5 Item 67 is
displayed.
Cause
The Visual Studio installation is missing the following file:
C:\Program Files\Microsoft Visual Studio\2022\Community\Common7\Tools\vsdevcmd.bat
It appears that this program is not fully installed.
Solution 1
Launch the installer for Visual Studio. Ensure that the items as listed in Step (c) of this article are selected and installed.
Log out and log back in on your machine, then test your setup as per this article.
Solution 2
If Solution 1 does not resolve the matter, your protection software might be removing the above listed *.bat file.
If you, your protection software will need to be toned down somehow.
Page 240
PSCAD - Resolving Launching, Compiling, and Running Issues
Problem
The following warning displays during a project run:
Cause
This warning is displayed to reduce the risk that any unsaved changes in the project are lost. Unsaved changes will be lost if
the operating system becomes unstable and crashes due to the large simulation storage requirements.
Solution
Select from the following options:
• Select “Yes” to save the project and proceed with the simulation.
• Select “No” to not save the project and proceed with the simulation.
Page 241
PSCAD - Resolving Launching, Compiling, and Running Issues
Problem
When a case is run, the following dialog box is displayed:
When the Fortran Medic utility is run (Appendix A.2), the conflicts as shown in Appendix A.5 Item 39 are displayed.
Cause
PSCAD is compiling cases using GFortran, and the computer is set to use non-English decimal number format. However,
GFortran is affected by Regional and Language Settings.
Solution 1
Switch to Intel Fortran compiler, as this compiler is not affected by regional and language settings.
Solution 2
Continue to use GFortran, but perform the following configurations:
• Launch the Fortran Medic utility (see instructions in Appendix A.2).
• In the utility, locate the following error message:
For v4.2, the error will appear as follows:
gNU-compiled cases might not work due to non-English decimal number formats
For v4.3 and later, the error will appear as follows:
GFortran-compiled cases might not work due to non-English decimal number formats
• Right-click on the error, and a “Solution” dialog box will display. Follow the instructions in the dialog box to
resolve the issue. Note the following:
If prompted to modify the Control Panel, see instructions below for more details, below (for v4.2.1 and
v4.3.0).
Page 242
PSCAD - Resolving Launching, Compiling, and Running Issues
• If prompted to Modify the Control Panel (Solution 2): Perform the following:
In the Windows Control Panel, go to the “Region and Language” dialog, and select “Additional settings”:
Page 243
PSCAD - Resolving Launching, Compiling, and Running Issues
Problem
The following error message is displayed when performing a multi-run using GFortran 4.2:
Solution
There is a bug in GFortran 4.2, in which GFortran does not release the “handles” when running a multi-run simulation, and
it crashes the simulation. The following are work-arounds:
Page 244
PSCAD - Resolving Launching, Compiling, and Running Issues
Page 245
PSCAD - Resolving Launching, Compiling, and Running Issues
Problem
The following messages are displayed in the Runtime Messages:
PSCAD
OK
Cause
Security software (anti-virus software or firewall) is blocking PSCAD from communicating with the simulation over the
dedicated TCP ports, so the run fails. For example, McAfee has been known to cause this issue.
Applicability
This is applicable to using any compiler for compiling the PSCAD case.
Solution
PSCAD must be able to access the dedicated TCP ports: 30,000 to 30,999
• Obtain assistance from your IT staff to allow PSCAD to communicate on the above TCP ports.
• Turn off your anti-virus or firewall software.
Page 246
PSCAD - Resolving Launching, Compiling, and Running Issues
8.6 Receiving Error “connection function failed with error: 10013” Runtime Error Message
Problem
The following message displays in the Runtime Messages pane:
Cause 1
Some aggressive security software (anti-virus or firewall) is preventing the simulation from being run.
Verification 1
To verify the above cause, disable all security software, then retry running the case.
If the case runs alright, then one or more security products is blocking the run.
To test which product/s are causing this, enable one product at a time and test whether the run is blocked with that
product.
Solution 1
Try one of the following solutions:
• Place all of your PSCAD projects into one designated folder, and whitelist that folder in the offending protection
software.
• Whitelist PSCAD simulations and activities in the offending protection software. A listing of requirements may
help with this: From this page, select the System Specifications and Requirements link pertaining to your PSCAD
branch, display the posted requirements document, and review Chart 2.
• Disable your protection product when running PSCAD.
Cause 2
The PSCAD becomes blocked if WIFI and Ethernet are both enabled.
Solution 2
Turn off your WIFI, and retry running the case.
Page 247
PSCAD - Resolving Launching, Compiling, and Running Issues
Problem
The following message displays in the Runtime Messages pane:
Cause
This error is most likely caused by ‘floating’ parts of circuit, i.e in which there is no ground reference in that part of circuit.
For example, it may occur in the delta side of a three-phase transformer.
Solution
Ensure all circuits are grounded.
Page 248
PSCAD - Resolving Launching, Compiling, and Running Issues
8.8 Unable to Run any PSCAD Cases – The Regional Language Issue
Problem
When running any case, the run fails, and the following Build Messages display:
EMTDC Simulation Run
Communications: Command: vdiv.exe -v4 localhost 57153
Communications: Connection established.
Non-standard Messages: Current locale = [SOME COUNTRY]
The issue you have encountered is directly due to your computer’s Language and Region (also known as locale) settings, as
some regions do not handle decimal numbers in the traditional English (USA) manner.
To date, some users with Dutch, German, Finnish, or Spanish language settings have experienced any or all of the following
issues:
• Runtime errors or crashes due to invalid reading of the map file,
• Runtime errors due to invalid reading of the snapshot file,
• Runtime results are incorrect if using GFortran, but correct if using Intel or Compaq, and
• Runtime’s Finish time is adjusted to some other value
• Undefined, zero, or negative finish time
• Runtime’s Finish time is adjusted to some other value
• Unable to enter floating point numbers.
Affected Versions
This issue affects PSCAD X4 (4.4.0) only when using the GFortran compiler if the Use English (USA) number format is not
selected
This issue affects PSCAD X4 (4.3.1) only when using the GFortran compiler if the Use English (USA) locale is not selected
This issue affects PSCAD X4 (4.3.0) only when using the GFortran compiler.
This issue affects PSCAD 4.2.1 only when using the GNU compiler.
Page 249
PSCAD - Resolving Launching, Compiling, and Running Issues
Page 250
PSCAD - Resolving Launching, Compiling, and Running Issues
Page 251
PSCAD - Resolving Launching, Compiling, and Running Issues
• Click on Additional Settings (Note that we tested this with Dutch settings).
Page 252
PSCAD - Resolving Launching, Compiling, and Running Issues
• Save your settings and PSCAD should work fine with different language settings.
If Unable to Resolve Your PSCAD Issues
If you have any further PSCAD issues, then please download the latest FortranMedic utility and send in the generated log
file as per Appendix A.2 and Appendix A.3.
Page 253
PSCAD - Resolving Launching, Compiling, and Running Issues
Problem
When trying to run a PSCAD case, an error similar to the following is displayed:
Process
Note
See Section 7.4 for similar issue.
Problem (1)
Anti-virus software is blocking the execution of files with double extensions (e.g. *.mak.bat), which are present in PSCAD
v4.6.3 Update 4 and older.
Specifically, McAfee released an update that detected and blocked files with double extensions in June 2018.
Solution (1.a)
Continue to use McAfee, but update your PSCAD software to v4.6.3 Update 5 or newer.
PSCAD v4.6.3 Update 5 and newer has replaced the double file extensions (*.mak.bat) with a single file extension
(*_mak.bat). Further details on this update are listed in this article.
Solution (1.b)
Continue to use McAfee, but somehow configure the anti-virus software to not block the PSCAD files. For example, confine
all PSCAD project files to a particular, local folder, (e.g. C:\PSCAD), and configure your protection software to allow all
executable files to run from that folder without restriction. This would be analogous to giving a ‘sandbox folder’ to PSCAD
which would not be monitored by the protection software.
Solution (1.c)
Switch to a different anti-virus software. Some examples of protection software that some customers switched to are
Microsoft Security Essentials, Malwarebytes, and Windows Defender.
Solution (1.d)
Turn off anti-virus software.
Problem (2)
User was trying to run a project from a remote drive.
Solution (2)
Move the project to a local drive.
Page 254
PSCAD - Resolving Launching, Compiling, and Running Issues
8.10 Receiving a “The simulation process has stopped unexpectedly. Please review runtime messages for
details” Error Message
Problem
When running a PSCAD project, the simulation appears to build and to start to run. However, the run stops, and the
following build message displays:
The simulation process has stopped unexpectedly. Please review runtime messages for details.
Issue (1)
In addition to the information listed above, in “Problem”, when the Fortran Medic is run, the following error is listed (see
Appendix A.5, Item 45 for details):
Network Information
…
Error: NotConnected (10057), A request to send or receive data was disallowed because the socket is not connected and (when s ending on a datagram
socket using a sendto call) no address was supplied.
Cause (1)
PSCAD and the EMTDC are not able to communicate over ports 30,000 to 40,000.
Consequently, PSCAD cannot run the sumulations.
Solution (1)
Enable communication between PSCAD and the EMTDC over ports 30,000 to 40,000.
Issue (2)
In addition to the information listed above, in “Problem”:
• Anti-virus is turned off, so protection software is not blocking the run.
• The PSCAD project contains c-code, however, the edition of Visual Studio that is installed does not have a c-
compiler.
• There are no Runtime errors; the Runtime pane simply displays normal messages, similar to the following:
Cause (2)
The project contains c-code, but cannot be compiled because the compiling software (Visual Studio) does not have a c-
compiler.
Solution (1)
Install an edition of Visual Studio that has a C-compiler (see this article for further information on this).
Page 255
PSCAD - Resolving Launching, Compiling, and Running Issues
Issue (3)
In addition to the information listed above, in “Problem”:
• You have one compiler version installed (e.g. Intel 15), and two versions of Visual Studio (e.g. VS 2013 and VS
2015).
• You are trying to set up the capability to switch between using the two versions of Visual Studio (e.g. VS 2013
and VS 2015) with that same compiler (e.g. Intel 15). This is being done by toggling the PSCAD configuration
between the two, as per instructions in this article.
• You find that the build/run works alright when you have configured PSCAD to use the newer version of Visual
Studio (e.g. VS 2015).
• You find that when PSCAD is configured to use the older version of Visual Studio (e.g. VS 2013), the build
works, but the run fails, and the error related to “The simulation process has stopped…” is displayed.
As shown below, the Runtime Messages do not specify the cause:
• The build messages show evidence that the newer Visual Studio (e.g. VS 2015 = version “14.0”) was being
called by Intel, even though PSCAD is configured to use the older version of Visual Studio (e.g. VS 2013 =
version “12.0”).
Cause
Regardless whether PSCAD was configured to use VS 2013 or VS 2015, Intel will always pick the most recently supported
version.
In this case, Intel 15 is supported with both VS 2013 and VS 2015, so Intel picks VS 2015.
Solution
Install a second version of Intel 15, one that is not supported with VS 2015. For example, Intel 15 Update 3 (version
15.0.3.208):
• Intel 15 Update 3 will use VS 2013 (and will not detect VS 2015), and
Page 256
PSCAD - Resolving Launching, Compiling, and Running Issues
8.11 Otaining Different Simulation Results for a Case Compiled with GFortran and Intel
Problem
You are getting different simulation results between a case compiled with a GFortran compiler and the same case compiled
with the Intel compiler.
Cause
The differences in simulation results may be due to un-initialised variables used in the code of one of your custom
components. Gfortran will set un-initialised variables to a value of Zero when the simulation starts. Intel will not set
uninitialized variables to anything, and they will be set to randum numbers by default.
Solution
Try removing/replacing parts of your network until the results match, then you should be able to identify which component
is causing problems. Once you have the component isolated, look at its custom code and see if there are any problems.
[case 367
Problem
When attempting to run a PSCAD project, the following error displays:
NMAKE : fatal error U‘077: 'cas’_name' : return‘code '0xc0’00005'
Cause
Anti-virus protections software is blocking the executable files from running.
Solution (1)
Whitelist the folder containing the PSCAD case.
Solution (2)
Whitelist a sandbox folder, then move any PSCAD case files into that folder, and always run any PSCAD cases from that
sandbox folder.
Solution (3)
Turn off the anti-virus software when running PSCAD cases.
Page 257
PSCAD - Resolving Launching, Compiling, and Running Issues
Problem
If a user’s laptop is connected to its docking station, and they have a wired network connection to their docking station, and
they are connected to their corporate VPN, then PSCAD is unable to run cases.
The following dialog might display:
PSCAD
OK
However, user can successfully run a simulation regardless whether their laptop is connected to their corporate VPN or not,
when either of the following is true:
• They connect to and use their WiFi network, or
• They unplug the laptop from their docking station and plug the network cable directly into their laptop network
jack.
Solution
• If possible, connect your laptop to a WiFi network, or
• Unplug the laptop from its docking station and plug the network cable directly into the laptop’s network jack, or
• Contact your IT staff for resolution.
Page 258
PSCAD - Resolving Launching, Compiling, and Running Issues
8.14 Running PSCAD Cases Prompts You for Windows Administrator Rights to Execute
Problem
PSCAD is launched with normal user rights, and a 32-bit GFortran 4.6.2 or 32-bit GFortran 8.1 compiler, or Intel compiler is
successfully used to compile a PSCAD case. However, when attempting to run the case, the following occurs:
1. A noticeable delay of no activity
2. The compiled case displays a prompt asking you for Windows administrator privileges in order to execute
3. After agreeing to the above prompt, you receive 2 or more notices of missing DLLs, similar to those below:
• Code execution cannot proceed because libstdc++-6.dll was not found
• Code execution cannot proceed because libgfortran-3.dll was not found
• Code execution cannot proceed because libquadmath-0.dll was not found
4. You then receive a notice that the case has unexpectedly aborted
Affected Platforms
This issue affects PSCAD running on Windows 7, and up to and including, Windows 10
Cause
If the filename of a 32-bit executable includes keywords like “install”, “setup”, “update” etc. then that file will automatically
request administrator privileges when it executes. This is standard Windows Installer Detection Technology behaviour
which is built into the Windows file behaviour.
Solution
Ensure that your PSCAD case filename does not include the following character sequences;
“install”, “setup”, “update”
For example, the following files will be affected by this issue:
UpdatedSystem.pscx
hassetUplandsIntertie.pscx
Page 259
PSCAD - Resolving Launching, Compiling, and Running Issues
Problem
When running a PSCAD project using GFortran compiler, the run fails, with Runtime message similar to the following:
Initializing Simulation Run
Executing > "C:\...”*.bat"
C:\...\*.gf42“call "C:\...gf42va”s.bat"
Access is denied.
Or when running a PSCAD project using the Intel Fortran compiler, the run fails, with Runtime messages similar to the
following:
Initializing Simulation Run
Executing > "C:\...\\"*.bat"
C:\...\*.if15_x86>call "C:\...\ifortvars.bat" ia32
Solution 3
Work with your IT team to test which of your anti-virus software is blocking the *.bat files:
• Disable all of your anti-virus software, then try running the project.
• If the project runs, then one or more anti-virus software must be blocking the *.bat files.
• If so, enable each anti-virus software one at a time, running the PSCAD project each time, to determine the
offending anti-virus software. Once the offending anti-virus software has been identified, either configure it to
allow you to run the PSCAD *.bat files, or whitelist a folder that will never be monitored by the anti-virus software,
and always save and load PSCAD projects from that folder.
Page 260
PSCAD - Resolving Launching, Compiling, and Running Issues
8.16 Receiving Runtime Error “The simulation is no longer responding and may have an unexpected
disconnect”
Problem
When running a PSCAD project, the run fails, with Runtime message similar to the following:
PSCAD
Cause
This message has been associated with TrendMicro protection software blocking the PSCAD run.
Solution
Request help from your IT team to whitelist a folder for TrendMicro to ignore, and run your PSCAD projects from there.
For example, C:\PSCADProjects\
Other Causes / Solutions
Refer to the following sections for further possible causes / solutions: Sections 7.4, 7.79, 7.114.
Page 261
PSCAD - Resolving Launching, Compiling, and Running Issues
8.17 Simulation Builds and Starts to Run, but the Run Stalls
Problem
After a PSCAD project has been built, the run starts, but then the run stalls with no error messages.
This occurs for large, complex projects with parallel processes.
Small projects without parallel processes do not encounter this issue.
Cause
Overly sensitive protection software may be blocking the run.
It may be that large, complex projects with parallel processes are viewed as suspicious activity by the protection software.
Whereas, a small project without parallel processes are not considered as a threat.
Affected Versions
Occurs in PSCAD version 5.0.1 Update 3 and older.
Solution 1 – Keep Stopping and Restarting the Run in PSCAD
When the Run stalls, Stop and restart the run, repeat.
The run may eventually creep closer and closer to completion with each run, until the entire run is complete.
Solution 2 – Whitelist the PSCAD Project Folder
Ask your IT Team to configure your protection software to whitelist the PSCAD project folder.
This will not work for every protection software brand. Some protection software will still block the simulations in a
whitelisted folder.
Solution 3 – Update your PSCAD When this Issue has been Resolved
In an upcoming version of PSCAD, this issue will be resolved. To be determined.
Solution 4 – Disconnect Computer from your Network
The protection software might only be used when a computer is connected over the Network.
Disconnecting your computer from the Network might allow PSCAD projects to run without activating a response from the
protection software.
Continued…
Page 262
PSCAD - Resolving Launching, Compiling, and Running Issues
Continued…
Page 263
PSCAD - Resolving Launching, Compiling, and Running Issues
b. The following example shows how open a Command Prompt at the temporary folder:
c. The following example shows how to enter the project executable file in the Command Prompt to start the run.
Note:
---For PSCAD v4.6.3 Update 4 and older, this will be in format: <projectfilename>.mak.bat
---For PSCAD v4.6.3 Update 5 and newer, this will be in format: <projectfilename>_mak.bat
Page 264
PSCAD - Resolving Launching, Compiling, and Running Issues
8.18 A Project That Runs in v5.0.0 Fails to Build in v5.0.1 with Error “Abnormal termination of EMTDC by
DSLINT” and “Status code = 2000”
Problem
A customer’s custom project successfully builds in PSCAD v5.0.0, but when run in PSCAD v5.0.1, the run fails, with similar
build messages:
Linking objects and libraries into binary ‘*.exe’
Finished compiling with PSCAD Version 5.0.1
Invalid node number on receiving end of Line/Cable ‘*’
Non-standard Messages: Abnormal program termination.
EMTDC Runtime Error: abnormally terminated
ERROR: Abnormal termination of EMTDC by DSLINT
Simulation stopped.
And messages similar to the following are reported in the Runtime Pane:
Invalid node number on receiving end of Line/‘a’le '*'
Non-standard Messages: Abnormal program termination.
EMTDC Runtime Error: abnormally terminated
=========================================================
ERROR: Abnormal termination of EMTDC by DSLINT
=========================================================
Terminating connection.
Simulation stopped.
Simulation has ended. Status code = 2000
Cause
This error occurred in one customer’s project. The error is directly related to mutually coupled t-lines, and how
PSCAD interprets the end-point connections in terms of subsystem number.
Between PSCAD v5.0.0 and v5.0.1, it was discovered that an over-simplification in the algorithm for organizing
mutually coupled t-line endpoints could wind up combining multiple subsystems into a single subsystem. The
compiler was modified to ensure this situation did not occur.
However, in this one PSCAD project, there were elements that were not considered by this new compiler
update. Consequently, this one project runs in v5.0.0 and not in v5.0.1.
Solution
Update to PSCAD v5.0.2 or newer.
Page 265
PSCAD - Resolving Launching, Compiling, and Running Issues
8.19 Receiving Error “Unable to assign data port for a simulation process. Aborting launch.”
Problem
When attempting to run a project, the run fails, with the following error:
Cause
During a PSCAD project run, communication between PSCAD and the EMTDC occurs over a specific port range (30000 to
40000). If that port range is unavailable for some reason, the run will fail.
Possible causes that the port range is unavailable may be:
• -Restriction for the port range is specifically imposed on your machine or over VPN.
• -PSCAD has already consumed all of the ports. For example, 10 instances of PSCAD open and are running
projects, which are using all 10 ports.
When the 11th instance of PSCAD tries to use a port, all ports are unavailable and in use, so the run fails.
Solution
• -Ensure that the port range (30000 to 40000) is not blocked on your machine.
• -If you are running multiple cases simultaneously, do not launch multiple instances of PSCAD.
Instead, load the multiple projects into a maximum of 10 instances of PSCAD, and run the projects
simultaneously.
Refer to High Performance Computing.
Page 266
PSCAD - Resolving Launching, Compiling, and Running Issues
Problem
When attempting to run a project, the project builds and runs, but the plots do not become populated.
Cause
Antivirus or firewall may be blocking the data from transferring between the simulation and PSCAD. This is a common
problem because they are separate programs and have to exchange data between each other.
Solution
Disable all of your protection software, and test whether the PSCAD run.
If the plots are populated, one or more of your protection software is blocking the plotting.
Enable one protection software at a time, testing a PSCAD run each time, until you determine which product is causing this
issue.
Once you have determined the product, whitelist PSCAD activities. See Appendix F for information on what activities need
to be whitelisted in PSCAD.
Page 267
PSCAD - Resolving Launching, Compiling, and Running Issues
Problem
When trying to run a PSCAD project with GFortran, the build completes, but the run does not proceed. There are
no error messages or any kind of indication of the cause.
Cause 1
Some sort of protection software (e.g. TrendMicro) is blocking the execution of the temporary executable file.
Solution 1
Whitelist PSCAD activities in your protection software, or disable the protection software
Cause 2
Unknown.
Solution 2
Restart your machine.
Page 268
PSCAD - Resolving Launching, Compiling, and Running Issues
8.22 Receiving the Runtime Error: “This program is blocked by group policy”
Problem
When trying to run a PSCAD project with GFortran, the build completes, but the run fails, with an error similar to
the following:
Initializing Simulation Run
Executing > …
This program is blocked by group policy. For more information, contact your system administrator.
The simulation process for ‘*’ has stopped unexpectedly.
Please review runtime messages for details.
Cause
The temporary executable file that was created during the project build is being blocked from running by your
Group Policy/protection software.
Solution
Whitelist PSCAD activities in your protection software, or disable the protection software.
For more information, please refer to the Usage requirements chart in the corresponding document on this
page.
Page 269
PSCAD - Resolving Launching, Compiling, and Running Issues
8.23 The Run Fails when Initializing a Large Number of PGBs in PSCAD v5.0.2
Problem
When trying to initialize a large number of PGBs in PSCAD v5.0.2, the run fails.
Cause
There is an issue with initializing a large number of PGBs in PSCAD v5.0.2.
Solution
Download and install either PSCAD v5.0.2 Hot Fix 1 or newer.
If the installer for the above software is not available in your MyCentre user account, please contact our Sales
Desk.
Page 270
PSCAD - Resolving Launching, Compiling, and Running Issues
Page 271
PSCAD - Resolving Launching, Compiling, and Running Issues
Page 272
PSCAD - Resolving Launching, Compiling, and Running Issues
Problem
When compiling a PSCAD project, build messages similar to any of the following display:
Test_Case Test_Case.mak(167) : fatal error U1035: syntax error : expected ‘:’ or ‘=’ separator
Test-Case Unable to generate a simulation executable for namespace ‘Test_Case’
Cause
The errors are most likely a result of FORTRAN code errors in custom models.
Solution
There are three possible solutions:
• If these errors only show up in newer FORTRAN versions, then fix the programming errors, if wanting to use the
models with newer FORTRAN versions.
• If these errors only show up in newer FORTRAN versions, then just compile using the FORTRAN versions in which
these issues do not result in errors (e.g. earlier versions).
• Fix these errors so that they work in ALL FORTRAN versions.
Page 273
PSCAD - Resolving Launching, Compiling, and Running Issues
A.1 Overview
The Fortran Medic utility is our standard mechanism for gathering and displaying information about a user’s machine that is
relevant to installing, launching, licensing, and running PSCAD (see Appendix A.2).
This utility does not perform any reporting back to us, other than allowing a user to generate a text log file and forward it to
our Support Desk to assist with troubleshooting (see Appendix A.3).
This utility does not modify anything on a machine without a user’s explicit permission, which is obtained by clicking on the
green or red arrows and confirming the recommended action (see Appendix A.4).
Many of the issues that may be detected by the utility are related to software installation and compatiblity, integration of
Intel Fortran compilers with Microsoft Visual Studio, and protection software preventing PSCAD usage (see Appendix A.5 for
some of the more common issues).
Appendix A.6 lists some of the functions in the Fortran Medic Utility.
To run the Medic without Windows Administrator privileges, select “No” when prompted by the Windows User
Account Control. (The Medic will not be able to be used to make any changes to your machine)
To run the Medic with Windows Administrator privileges, select “Yes” when prompted by the Windows User
Account Control. (The Medic will be able to be used to make changes to your machine)
c. When the Medic opens, click on the “Actions” menu and select “Start”. The utility will retrieve information about
your computer.
Page 274
PSCAD - Resolving Launching, Compiling, and Running Issues
c. Right-click on any green arrow to optionally perform additional actions only if instructed by [email protected].
Specific errors within this utility are listed in Appendix A.5.
Page 275
PSCAD - Resolving Launching, Compiling, and Running Issues
The following table lists common errors within the utility along with solutions:
Error Solution
2. Intel Fortran (xx.x.xxx) This error is only related to using PSCAD v4.2.1 and the specified version of Intel.
Conflicts The error is indicating that PSCAD v4.2.1 will not recognize this compiler due to the location of the Intel
Missing expected file executable file, “ifort.exe”, here:
C:\Program Files (x86)\IntelSWTools\compilers_and_libraries_2020.0.166\windows\bin\intel64_ia32\ifort.exe
File: C:\Program Files (x86)\Intel…\Ifort.exe
PSCAD v4.2.1 requires that this file be copied to here:
PSCAD V4 will not recognize this compiler
C:\Program Files (x86)\IntelSWTools\compilers_and_libraries_2020.0.166\windows\bin\Ifort.exe
The Fortran Medic utility can be used to copy the file to the required location, if you right-click on the
error in the utility, and select the option to proceed when prompted. Or you can copy this file over
manually.
5. Visual Studio [VS2005] You are running an older version of the Fortran Medic, and the following is occurring:
Installation Folders: • This version does not correctly detect Visual Studio Premier Partner (Shell Edition).
VS folder: C:\Program Files (x86)\Microsoft Visual Studio 8\ • This version interprets this information as an incomplete installation of the full (commercial) Visual
Folder exists Studio.
VC folder: The solution is to:
Visual Studio VS2005 is not installed. Folder not specified. • Download and re-run our latest version of the Fortran Medic utility.
• Check the installation of Visual Studio.
Page 276
PSCAD - Resolving Launching, Compiling, and Running Issues
Error Solution
8. This variable is normally added to the Windows Environment during the installation of the product.
If the variable is missing, for some machines, PSCAD is unable to detect and use this software during a
Intel Fortran (<version>) build.
Environment [Actual]: Right-clicking on the error and selecting “OK” should result in the Fortran Medic utility adding the
IFORT_COMPILERXX: Variable to the Environment.
Environment variable does not exist Or select “Cancel” to leave the environment value unmodified.
(Refer to this article for further details on adding the variable to the Environment, either manually or
automatically with the Fortran Medic utility)
See Sections 7.123 and 7.119 for further information.
Page 277
PSCAD - Resolving Launching, Compiling, and Running Issues
Error Solution
11. Intel Fortran (*) Right-clicking on the message displays a dialog with the solution steps:
Conflicts
INTEL_LICENSE_FILE – Duplicate path segments may cause this compiler to fail
To fix this issue, the FortranMedic must replace an environment variable value.
Name: INTEL_LICENSE_FILE
Type: Machine
Required value: C:\Program Files (x86)\Common Files\Intel\Licenses
12. “PATH (Machine) Environment Variable” Right-clicking on the error displays the following dialog:
Conflicts
The PATH variable is excessively long and you may lose your shortcuts to
Notepad and other system programs. In order to resolve this issue, you need to use the FortranMedic to remove
unnecessary paths from the PATH environment variable.
Page 278
PSCAD - Resolving Launching, Compiling, and Running Issues
Error Solution
16. Detecting Intel Compilers Right-clicking on the error displays a message similar to the following:
Intel Fortran 15.0.0 to 15.0.221 might not work due to Visual Studio 2015 being
installed.
Cases compiled with Intel Fortran 15.0.221 should compile and link, but
MAY fail to run due to Visual Studio 2015 being installed.
To resolve this, you need to:
1. Use Intel Fortran 16.0 or better, or
2. Rename your Visual Studio 2015 installation folder, or,
3. Uninstall Visual Studio 2015.
17 Visual Studio [*] Compiling might not work. The Medic tool can detect and in some cases repair this missing variable for
Required Environment and Registry Values the following Visual Studio releases:
VS90COMNTOOLS (environment) Can repair it (right-click on the error,
Visual Studio edition / version Can detect it
and select the option to repair it)
does not exist
2008 Professional Edition ✓ ✓
2008 Premier Partner (Shell) Edition ✓ X
2010 Professional Edition ✓ ✓
2010 Premier Partner (Shell) Edition ✓ X
2012 Professional Edition ✓ X
2013 Professional Edition ✓ X
2013 Premier Partner (Shell) Edition ✓ X
2015 Professional Edition ✓ ✓
2015 Premier Partner (Shell) Edition ✓ X
2015 Community Edition ✓ ✓
2017 Professional Edition ✓ ✓
2017 Community Edition ✓ ✓
2019 Professional Edition ✓ ✓
2019 Community Edition ✓ ✓
2022 Professional Edition ✓ ✓
2022 Community Edition ✓ ✓
Page 279
PSCAD - Resolving Launching, Compiling, and Running Issues
Error Solution
18. MyUpdater These messages indicate that a utility has been installed on this machine (MyUpdater), but has never
Products file been logged into. Therefore the corresponding user file has not been created.
C:\Users\[Your Windows Username]\AppData\Local\Manitoba HVDC Research This message is intended to assist users experiencing an issue with logging in to MyUpdater due to
Centre\UpdateClient\Products.xml their computer or network restrictions.
File does not exist For further assistance on this a MyUpdater login issue, please contact our Support Desk
([email protected]).
Or
Products file
C:\Users\[Your Windows Username]\AppData\Local\Manitoba HVDC Research
Centre\UpdateClient\Products.xml
File exists
File does not contain a MyCentre identity
19. * Environment Variable Multiple unecessary segments should be removed because they might cause performance issues on
your computer.
Listing: [ length = * characters ] Right-click on each duplicated segment, and select “OK” when prompted:
…
Process Environment Variable Segment
Actions
Conflicts For the following User environment variable with * duplicate:
Duplicated segments Name: PATH
Value: *
C:\*** I want to do the following:
Keep the first occurrence only (with high priority)
Keep the last occurrence only (with low priority)
The Fortran Medic utility will retain only the first occurrence of this segment, and delete all duplicates.
The following message will display following a successful deletion of duplicates:
Page 280
PSCAD - Resolving Launching, Compiling, and Running Issues
Error Solution
20. Conflicts For example, a program called “Single Sign On Engine” is suspected of halting PSCAD simulations. See
Identity and access management software can affect operation of PSCAD Section 7.37 for details.
Double-clicking on this error displays the following dialog box:
21. One of the following errors displays: Double-clicking on one of the errors displays a dialog box similar to the one shown below. See Section
7.38 for more information.
GFortran (some_version)
...
Git and Mingw appear to be installed, and may cause PSCAD to fail when
Conflicts compiling cases with Intel Fortran.
Git version xxx appears to be installed and is in the PATH -
GFortran might not work To resolve this issue, you need to:
1. rename the Git folder,
2. remove Git and Mingw, or
( or) 3. Install PSCAD and Intel FORTRAN on another machine.
Intel (some_version)
…
Conflicts
Git/mingw may cause PSCAD to fail when compiling with Intel FORTRAN
Page 281
PSCAD - Resolving Launching, Compiling, and Running Issues
Error Solution
23. Installed PSCAD versions Uninstall then re-install the product, then re-start the Fortran Medic utility to ensure the error is
Conflicts cleared.
EMTDC files that are specific to VS 2010 and VS 2015 appear to be missing or
corrupted.
Page 282
PSCAD - Resolving Launching, Compiling, and Running Issues
Error Solution
Select “OK” and the Medic will fix the environment variable.
Refer to Section 7.47 for details.
Page 283
PSCAD - Resolving Launching, Compiling, and Running Issues
Error Solution
30. Machine Info Right-clicking on the error displays the following message:
Echo.vbe
C:\Windows\echo.vbe
The presence of echo.vbe file(s) can cause PSCAD compiles to
fail and unrelated dialogs to appear when attempting to
compile:
To resolve this issue, you need to:
- rename all echo.vbe file, then try again, or
- delete all echo.vbe files, then try again.
31. Automation Library The BuildTime.txt file is not detected by the Fortran Medic.
Install folder: C:\Program Files (x86)\PSCAD\Automation (see Appendix A.6 Item 3 to see the text when this file is detected)
Publish date: unknown
32. PATH (Machine) Environment Variable Right-click on each message and select the option to create/add that value.
Conflicts See Section 7.57 for more details.
The PATH environment variable is missing the
\Windows\System32 path segment.
Page 284
PSCAD - Resolving Launching, Compiling, and Running Issues
Error Solution
34. Installed PSCAD versions Visual C++ 2015 Redistributable version (x64/x86) 2015 version 14.0.23506 or better must be installed
… for this version of PSCAD.
PSCAD * requires Visual C++ 2015 Redistributable (x64/x86) This software may be installed as per this article.
version 14.0.23506 or better
Page 285
PSCAD - Resolving Launching, Compiling, and Running Issues
Error Solution
Can occur if the “Decimal symbol” and “Digit grouping symbol” in the Windows Region and Language
settings are set to the same character. These should not be set to the same symbol.
Page 286
PSCAD - Resolving Launching, Compiling, and Running Issues
Error Solution
(Or)
(or)
GFortran ***
… msys appears to be installed and is in the PATH, which can cause compiles using
Conflicts GFortran 4.2.1 to fail.
GFortran 4.2.1 compiles mi–ht fail - msys appears to be installed and is in To resolve this issue, you need to:
the PATH
1. use this utility to remove msys from the PATH, or
2. rename the msys installation folder when using GFortran 4.2.1, or
3. uninstall the msys application.
Note
This issue is applicable also to GFortran 4.6.2
Page 287
PSCAD - Resolving Launching, Compiling, and Running Issues
Error Solution
41. PATH (Machine) Environment Variable Right-clicking on this conflicts results in a dialog similar to the following being displayed:
…
Conflicts
Segments containing a ‘&’ character
<some path with ‘&’ character>
42. Network Information Right-clicking on this conflict results in a dialog similar to the following. Configure your machine as
… directed.
Page 288
PSCAD - Resolving Launching, Compiling, and Running Issues
Error Solution
43. Licensing Existing certificate licensing and the MyUpdater utility require that one of the following protocols be
enabled: SSL 3.0, TLS 1.0
…
This computer is not configured to support a required protocol. If the Medic detects that neither protocols are enable, this will be reported as shown in the left
column.
Right-clicking on the error displays the screen as shown below. Select OK to enable the TLS 1.0 protocol
for clients on this machine, to allow MyUpdater activities and certificate licensing:
As a result:
- the MyUpdater will not be able to display your authorized products
- PSCAD Free, PSCAD v4.5.4 and better, will not be able to use certificate
licensing
To resolve this issue, this utility needs to create/update the following
registry value:
Folder: HKEY_LOCAL_MACHINE (Default registry)
Key:
SYSTEM\CurrentControlSet\Control\SecurityProviders\SCHANNEL\Protoc
ols\TLS 1.0\Client
Name: Enabled
Value: 1
Select OK to enable the TLS 1.0 protocol for clients on this machine
Select Cancel to leave the registry value unmodified
44. Folder Info Windows short 8.3 pathnames has been disabled, which can affect the compiling of PSCAD projects.
Please refer to suggested solutions in Sections 7.62, 7.78 and 7.104.
… Note: The Medic will dipslay the following if 8.3 Pathnames are generated:
Conflicts
Windows is not generating short (8.3) pathnames
8.3 filename creating is disabled on all volumes
45. Network Information It has been detected that access to ports 30,000 to 40,000 is not allowed, therefore PSCAD cannot
communicate with the EMTDC in order to build and run the simulation.
…
Local socket connections: Communication over ports 30,000 to 40,000 must be enabled to run PSCAD simulations.
Error: NotConnected (10057), A request to send or receive data was Additional system requirements are as listed in this article.
disallowed because the socket is not connected and (when sending on a
datagram socket using a sendto call) no address was supplied See Section 8.10 for further information on this issue.
Page 289
PSCAD - Resolving Launching, Compiling, and Running Issues
Error Solution
46. Network Information It has been detected that PSCAD cannot communicate with external machines.
… This situation is only an issue if a user is trying to use high performance computing features in PSCAD,
External socket connections wherein multiple simulations are being launched in parallel over multiple machines.
Error: HostNotFound (11001), No such host is known This situation is not an issue if a user is simply running PSCAD simulations (singularly or in parallel) on
his own machine.
47. Visual Studio 2017 ProfIional This error can be disregarded.
This error only occurs if Visual Studio has been installed, but never launched.
...
Private registry:
privateregistry.bin
Folder does not exist
48. PSCAD settings If an invalid IP address was entered for PSCAD lock-based licensing, in the Licensing tab of the System
Settings dialog, then an error will display in the Medic results as shown in the left column. Right-
… clicking on this error will display an error dialog similar to the following:
LmgrHost (32-bit):
LmgrHost (64-bit): XX.XXX.XX.XX 2053 The specified lock-based License Manager IP address is
incorrect and PSCAD might not be able to acquire a lock-
An invalid IP address was specified based license.
Examples of invalid IP addresses include entering fewer or more than four digits (e.g. x.x.x.x), or when
any of the four digits exceed 255.
49. Network Information The detected conflict means that all outbound traffic on this machine is actually being routed to port
9000, or some other port, where most likely some service is listening on.
…
That appears, from prior research, to be a favorite port used by malicious software to sniff all traffic
Conflicts for, say, log in credentials, credit card info, etc, before possibly forwarding it on to the intended
Your default proxy may be hijacked. website or recipient, or maybe not, or maybe even streaming a copy of everything off to some remote
hacker server for later analysis.
Your system proxy may be hijacked.
It is recommended that the customer’s IT Department look at the issue and resolve it if required.
Page 290
PSCAD - Resolving Launching, Compiling, and Running Issues
50. User attempts to perform any changes to their machine using the Fortran
Medic utility, and receives an error message instead. A dialog similar to the following displays:
The Fortran Medic utility was not launched with Windows Administrator privileges, therefore the
Medic cannot make any changes to this machine.
To allow the Medic to make changes to this machine, close the Medic, and relaunch the Medic with
Windows Administrator privileges as specified in Appendix. A.2.
51. Conflicts For the system or user PATH and LIB environment variables, the Medic can correct path segments
containing any of the following:
Segments with unbalanced or mid-string quotes
• Contain an odd # of quotes (i.e. have unbalanced quotes), or
<some segment with unbalanced or mid-string quotes> • Contain any internal quotes (i.e. mid-string quotes),
The Medic can now repair them by removing all quotes in/bracketing that segment. Simply right-click
on the error as listed in the left column, and select OK in the “Confirm Environment Changes” dialog:
Page 291
PSCAD - Resolving Launching, Compiling, and Running Issues
52. Visual Studio 201X Professional (###) Right-clicking on the message in the left column displays a dialog similar to the following:
…
VS201XINSTALLDIR
Confirm Environment Changes
Path not set
The VS201XINSTALLDIR environment variable has not been set. This may
result in Intel FORTRAN failing to compile PCAD cases.
To fix this issue, the Medic must ensure that the following environment
variable exists and has the specified value.
Name: VS201XINSTALLDIR
Type: Machine
Required value: C:\Programs Files (x86)\Microsoft Visual Studio XX\
Select OK to create the environment variable
Select Cancel to leave the environment value unmodified
53. Detected Intel Compilers This error can be disregarded. This error is not associated with any issues when compiling PSCAD
projects.
…
Intel Fortran (*)
…
Conflicts
LIB has 64-bit library path
C:\Program Files (x86)\...intel64
Page 292
PSCAD - Resolving Launching, Compiling, and Running Issues
(and/or)
(and/or)
55. PSCAD Right-clicking on the conflict displays the informational dialog below.
The tline.exe file must somehow be unblocked from being launched.
… This file is located in the following path:
TLine.exe (version = unknown exit code = 0xFFFFFFFF) C:\Program Files (x86)\<version>\bin\win\
Tline.exe did not launch or execute as expected
…
Conflicts
Tline.exe did not launch or execute as expected To fix this issue, you need to:
1. disable your firewall, or
2. disable or update your anti-virus software, or
3. manually configure your anti-virus software to allow tline.exe to run.
Page 293
PSCAD - Resolving Launching, Compiling, and Running Issues
56. Installed PSCAD versions The medic has detected that multiple required files are missing from the PSCAD installation.
See Section 7.97 for more details.
…
РSCAD Free [Publish date: *]
Dockable pane settings
Installed by: UpdateClient
Install folder: C:\Program Files (x86)\PSCADFree
Folder exists
App folder: C:\Program Files (x86)\PSCADFree\bin\win
master.pslx (version = unknown, revised = unknown)
File does not exist
fortran_compilers.xml (state: missing )
matlab_versions.xml (state: not supported )
TLine.exe (version = unknown)
File does not exist
MFCMIF80.dll
File does not exist
ZSLib2.dll
File does not exist
User profile file: C:\Users\support\AppData\Local\Manitoba HVDC Research
Centre\PSCAD\user_profile_free.xml
File exists
private_env: true = Private to process only
english_locale: true = English (United States)
Certificate behaviour on exit: <missing>
Unable to return a certificate on exit: <missing>
Conflicts
The PSCAD Free fortran_compilers.xml file is missing and will not detect any
Fortran compilers
File: C:\Program Files (x86)\PSCADFree\fortran_compilers.xml
Tline.exe is missing or appears to be empty
ZSLib2.dll is missing or appears to be empty
Internet Explorer 11 may cause PSCAD to not to display the Start Page.
Page 294
PSCAD - Resolving Launching, Compiling, and Running Issues
57. PSCAD Configuration Right-clicking on the error displays the following dialog:
PSCAD V4 file extensions
Warning
Status: <not configured or not found>
PSCAD X4 file extensions Double-clicking on any psc or psl files will not launch PSCAD.
Status: <not configured or not found>
To resolve this issue, you need to install an official release of PSCAD 4.2.1
or PSCAD X4, such as the latest release of PSCAD 4.5 or PSCAD 4.6.
This error indicates that PSCAD cannot be launched from the PSCAD model file. PSCAD would have to
be launched by the Windows shortcut or from within the PSCAD program files (…bin\win…).
The solution is to obtain and install a supported version of PSCAD, as specified in the above dialog.
Please send your request, along with your license number, to [email protected].
58. Intel Fortran ([Version] x86) Right-clicking on the error in the left view will display the following dialog.
Selecting OK will replace the non-ascii symbol “®” in the regIry with “(R)”.
… For more information, refer to Section 7.103 for details.
SOFTWARE\Intel\Compilers\Fortran\[Version]\IA32 | Display String
Intel® Fortran Compiler Classic [Version] [IA-32] Confirm Registry Action
…
The registry DisplayString text contains a ‘®’ symbol which can, in some cases,
cause the following error:
Intel Fortran ([Version] x64)
… Fatal error U1033: syntax error : ‘Fortran’ unexpected
SOFTWARE\Intel\Compilers\Fortran\[Version]\EM64T_NATIVE | Display String
To resolve this issue, this utility needs to create/update the following registry
Intel® Fortran Compiler Classic [Version] [IIntel(R) 64]
value:
Page 295
PSCAD - Resolving Launching, Compiling, and Running Issues
59. Visual SIudio XX Right-clicking on the red arrow in the left view will display the dialog below.
The Medic displays the cause, but cannot be used to resolve the matter. Refer to Section 7.106 for
... more information on this issue, and how to resolve it.
Detected folders and files:
...
Warning
Visual C++ folder:
C:\Program Files\Microsoft Visual Studio\XXXX\...\VC The current installation of
Folder does not exist Visual Studio XX
Visual C/C++ Tools folder: appears to be missing the following workload
C:\Program Files\Microsoft Visual Studio\XXXX\...\VC\Tools\MSVC Desktop development with C++
Folder does not exist without which you will not be able to use Intel FORTRAN to compile and run
PSCAD cases.
60. Environment (System) This error indicates that a variable was detected in the Environment, and that variable is referencing an
Intel Fortran compiler that is not currently installed.
...
Conflicts: Perhaps that product was previously installed, but was since uninstalled.
Variable(s) which reference non-existent folders or files
IFORT_COMPILER21 The presence of this error could potentially cause an issue if using an Intel compiler for your PSCAD
projects.
61. Intel Fortran <version> One of the PSCAD Program Files is outdated, and needs to be replaced.
See Section 7.85 for further details.
...
Conflicts:
Unable to locate the ifortvars.bat file
PSCAD <version> will not be able to use Intel <version> within an
isolated compiler environment
Page 296
PSCAD - Resolving Launching, Compiling, and Running Issues
62. Visual Studio <Some Version> <Some Edition> If you right-clicking on the “Path may be incorrect" error as shown in the left pane, a dialog similar to
the following will display:
…
VS2017INSTALLDIR <Or Some Other Environment Variable>
C:\Program Files (x86)\Microsoft Visual Studio\2017\Enterprise
<Or Some Other Path> The VS2017INSTALLDIR environment variable value appears to be incorrect.
Path may be incorrect
Current value:
C:\Program Files (x86)\Microsoft Visual Studio\2017\Enterprise
Expected value:
C:\Program Files (x86)\Microsoft Visual Studio\2017\Professional
To fix this issue, the Medic must ensure that the following environment
variable exists and has the specified value.
Name: VS2017INSTALLDIR
Type: Machine
Required value: C:\Program Files (x86)\Microsoft Visual
Studio\2017\Professional
Page 297
PSCAD - Resolving Launching, Compiling, and Running Issues
64. PSCAD v5.0.1 (x64)… If the Medic detects that PSCAD is installed, the medic will determine whether the most recent
… matlab_versions.xml program file is installed. If not, a conflict will display similar to the text in the left
column. Right-clicking on the conflict will display the notice shown below.
Conflicts
It is recommended to update this file if you will be performing MATLAB|PSCAD co-simulation.
The PSCAD 5.0.1 matlab_versions.xml file is outdated and will not detect
newer versions of Matlab
Confirm File Update
To fix this issue, The FortranMedic must update the
matlab_compilers.xml file.
65. Intel Fortran (19.2.46319, x86) Missing folders for each of these filepaths indicate the possibility (not confirmation) that the Intel
… Fortran Compiling software might either not be fully installed, or else different versions of the two
toolkits (Base and HPC) might be mixed.
oneAPI\xxx\Latest: folders
See Section 7.120 for more information.
C:\Program Files (x86)\Intel\oneAPI\compiler\latest
Folder is missing
C:\Program Files (x86)\Intel\oneAPI\mpi\latest
Folder is missing
C:\Program Files (x86)\Intel\oneAPI\tbb\latest
Folder is missing
C:\Program Files (x86)\Intel\oneAPI\compiler_ide\latest
Folder is missing
C:\Program Files (x86)\Intel\oneAPI\debugger\latest
Folder is missing
C:\Program Files (x86)\Intel\oneAPI\dev-utilities\latest
Folder is missing
C:\Program Files (x86)\Intel\oneAPI\licensing\latest
Folder is missing
Page 298
PSCAD - Resolving Launching, Compiling, and Running Issues
66. If the error to the left displays in the Fortran Medic results, right-clicking on the error will result in a
Missing: C:\Program Files (x86)\...\emtdc.cfg
message similar to the following;
67 Visual Studio 2022… Right-clicking on the error shown to the left will display a message similar to the following:
…
Page 299
PSCAD - Resolving Launching, Compiling, and Running Issues
1. PSCAD Configuration…
Refer to Section 7.36 for instructions on this configuration.
Installed PSCAD versions…
Note
PSCAD X4 Release (4.6.X)
Applicable to PSCAD v4.6.1 and later.
Configure EMTDC for Visual Studio 2015 and later
If you proceed with this action, PSCAD will be configured to use Visual Studio 2010, 2012, or 2013.
Warnings
1. If you proceed, PSCAD will no longer be configured to use Visual Studio 2015 and newer.
2. It is not recommended to ever use Visual Studio 2012 as it can interfere with other Visual Studio
installations even after the uninstallation of Visual Studio 2012.
Notes
1. This function does not install Visual Studio 2010, 2012, or 2013, it merely configures PSCAD to be able to
use these versions.
2. Configuration with Visual Studio 2015 and newer may be restored as per Section 7.36.
Page 300
PSCAD - Resolving Launching, Compiling, and Running Issues
Function Action
3. Automation Library The BuildTime.txt file has been properly detected by the Fortran Medic.
Install folder: C:\Program Files (x86)\PSCAD\Automation (see Appendix A.5 Item 31 to see the text when this file is not detected)
Publish date: YR.MO.DAY…
A newer version of the PSCAD v4.6.3 master library and/or EMTDC runtime libraries are
available.
Important Note
If you previously used the Medic utility to configure PSCAD v4.6.3 EMTDC to use ‘Visual
Studio 2015 and later’, then after applying this update, you will need to:
- launch the Medic utility and run it,
- scroll down to PSCAD 4.6.3 (x64),
- right click on ‘Configure EMTDC for Visual Studio 2015 and later’ if required
- then rerun the Medic,
- scroll down to PSCAD 4.6.3 (x86)
- ensure it is also configured for ‘Visual 2015 or later’
6. PSCAD X4 Release (4.6.3…) The dockable pane default settings may be restored using the Medic. Close all instances of PSCAD, then in
Dockable pane settings the Medic, right-click on “Docable pane settings”, and select the option to restore the pane settings.
Page 301
PSCAD - Resolving Launching, Compiling, and Running Issues
Page 302
PSCAD - Resolving Launching, Compiling, and Running Issues
Function Action
7. Environment (System) If the LIBRARY_PATH is set, GFortran Compilers might fail to build or the project to run.
…
LIBRARY_PATH=[some path] Right-clicking on the switch for the LIBRARY_PATH will display the dialog below.
If OK is then selected, the environment variable assigned to LIBRARY_PATH will be removed.
Warning – Proceeding with this deletion may affect other software.
Name: LIBRARY_PATH
Type: Machine
Deleting this environment variable may seriously affect the operation of the
Intel Visual compiler, if it was configured for use with an Intel compiler.
Page 303
PSCAD - Resolving Launching, Compiling, and Running Issues
8. PSCAD <version>… This feature will allow the user to select which PSCAD version will be launched from a project file.
Installed by..
Follow the steps in Appendix A.2 to download and run the newest version of the utility.
Note Ensure to select Yes when prompted by the Windows User Account Control, to elevate the Utility, so it
can make changes.
In the utility results, scroll down to your desired PSCAD version in, right-click on the green arrow beside the
PSCAD version to be selected (see left pane).
The following dialog will display.
Select “Yes” to set this version as the one that will be launched from a project file.
Page 304
PSCAD - Resolving Launching, Compiling, and Running Issues
Page 305
PSCAD - Resolving Launching, Compiling, and Running Issues
Page 306
PSCAD - Resolving Launching, Compiling, and Running Issues
Page 307
PSCAD - Resolving Launching, Compiling, and Running Issues
In many issues in this document, there is reference to a folder that may be hidden, called appdata. If this folder
is hidden, it may be displayed as shown:
Page 308
PSCAD - Resolving Launching, Compiling, and Running Issues
The following is a description of the files that are created and used when a PSCAD case is compiled:
• When a PSCAD case is compiled, for example
C:\Users\Public\Documents\PSCAD\4.6.2\Examples\tutorial\vdiv.pscx
• PSCAD will create a working folder called <case name>.<compilerID>.
• For GFortran 4.2, this would be vdiv.gf42, as shown in the screenshot below. For GFortran 4.6.2, the folder is called
.gf46, for Intel, it is called if9, if12, or if15.
• The vdiv.mak.bat file is created by PSCAD, and has the following format: <case name>mak.bat, and is executed by
PSCAD, which launches the GFortran 4.2 compiler for the entire case.
• The GFortran make/nmake file creates one batch file in the %USERPROFILE%\AppData\Local\Temp folder for each
.f file it needs to compile.
Page 309
PSCAD - Resolving Launching, Compiling, and Running Issues
• For GFortran 4.2 and GFortran 4.6.2, two or more batch files are created:
One is called BATCHnnnn.bat, and
One or more other randomly named batch files NOT ending with .bat
• For Intel, we observed two to three randomly named files, NOT ending in .bat
• Once the .f file has been compiled, all of the two to three batch files are deleted, and the next .f file is created.
• The contents of the BATCHnnnn.bat file created by GFortran typically looked like the following:
@echo off
• We were not able to capture any of the Intel batch files as they existed only very momentarily
As mentioned earlier, these temporary batch files are created by the third party GFortran or Intel Fortran compilers.
Continued…
Page 310
PSCAD - Resolving Launching, Compiling, and Running Issues
Licensing log file File Write C:\ProgramData\Manitoba HVDC Research Fixed May possibly be
Centre\LicenseManager\PscadLmgr.txt reconfigured
Certificate license Folder Read/Write For PSCAD v4.5 and v4.6: Fixed Fixed
(if used) C:\Users\Public\Documents\Manitoba HVDC Research
Centre\Licensing\Licenses
For PSCAD v5:
C:\Users\<user_id>\AppData\Local\Manitoba Hydro
International\Licensing\Licenses
Legacy License File Write Read/Write C:\Users\Public\Documents\Manitoba HVDC Research Fixed Fixed
(if self-licensing or Centre\LicenseManager\lmgr-hvdc
License Manager is used)
User settings File Write Read/Write C:\Users\USERID\AppData\Local\Manitoba HVDC Research Fixed May possibly be
Centre\PSCAD\ reconfigured
Page 311
PSCAD - Resolving Launching, Compiling, and Running Issues
Page 312
PSCAD - Resolving Launching, Compiling, and Running Issues
DOCUMENT TRACKING
Rev. Description Date
0 Initial 17/Jul/2014
Added new sections (7.3, 8.1, 8.2, A.5);
1 Updated sections (6.1, 10.1, A.4); 29/Jul/2014
Updated the document title
Changed section numbers (Section 6);
2 Added new sections (1.2, 2.1, 2.2, 8.3-8.11, 10.3); 08/Aug/2014
Updated sections (1.1, 6.1-6.3, 8.1)
Updated sections (3.1, 6.4, 8.1, 8.2, 8.5, 8.8, 8.9, 10.1, 10.2, A.2);
Deleted Section 8.4;
3 17/Oct/2014
Renumbered Sections 8.5-8.11 to 8.4-8.10;
Added new sections (7.4, 7.5, 8.11-8.14, 10.4-10.6)
Modified title; Reorganized content per following changes;
Moved all installation instructions to manual “Resolving Installation Issues”;
Moved MyCentre launching issue to manual “Update Client – Common Issues”;
4 28/Nov/2014
Section 4.2 moved from later section;
New Section 8 (moved issues from Section 7)
Added new Sections 4.1, 6.6, 7.15, 7.16, 10.1, 10.2
Deleted Section 6.6 (duplicated);
5 28/Jan/2015
Added new Sections 6.6, 6.7, 7.17, 7.18, 7.19
Update to Section 3.4, Section 9.2, Appendix A.5;
6 20/Apr/2015
Added new Section 7.20
7 Added new Sections 2.4, 4.3, 9.5 20/May/2015
Update to Sections 4.1, 7.8;
8 28/May 2015
Added New Sections 2.5, 2.6, 7.21, 7.22
9 Update to Section 7.8 07/Jul/2015
10 Added new Section 2.7, populated Section 3.3 28/Jul/2015
11 Added new Sections 5.1, 3.5, and 6.8 15/Sep/2015
Added new Cause #3 within Section 5.1;
12 Added new Section 3.6; 22/Sep/2015
Update to Appendix A.5
13 Added new Cause #3 within Section 7.3, Added new Section 3.7 and new Appendix C 24/Sep/2015
Added new Sections 3.8, 7.23, 7.24, and 7.25;
14 02/Nov/2015
Update to Section 3.6
15 Added new Section 2.8; Update to Sections 2.7, 3.4 and Appendix C 13/Nov/2015
PSCAD - Resolving Launching, Compiling, and Running Issues
New Sections 6.29, 7.93, 7.94, 7.95, 7.96, 7.97, 7.98, 7.99, 7.100, 7.101, 7.102, 7.103, 7.104,
7.105, 8.16, 8.17
Update to Sections 6.6, 7.57 Problem 2, 7.73, 7.76 and 7.89; Update to Solution 2 of Section
8.2, Added Issue 3 to Section 8.10;
79 11/July/2022
Added new Items 55, 56 and 57 to Appendix A.5;
Update to Item 39, Appendix A.5
New Sections 6.30, 7.106, 7.107, 7.108, 7.109, 7.110, 7.111, 7.112, 7.113, 8.18, 8.19, 8.20
New Solution 1 added to Section 7.103
New Items 58, 59, 60, 61, added to Appendix A.5
80 Update to Sections 6.29, 7.73, 7.84, 7.85, 7.90, 7.95, 8.15; 19/Oct/2022
Update to Appendix A.6 Item 5;
Deleted content from Sections 6.12 and 8.4, instead adding referrals to Section 7.95
PSCAD - Resolving Launching, Compiling, and Running Issues